Sunteți pe pagina 1din 161

2.3.

4 Unit 4:
We are going to give you some suggestions in this section on how to deal with portions of Unit-4. We
would be focusing on the following questions which generally one has in his/her mind when one thinks
about the easiest way to prepare for Unit-4.

Is this topic important keeping in mind CSIR-NET is in 3 months?

Which books to follow?

How many questions can I expect from this topic?

How to prepare in less time?

Topic-wise suggestion:

Is this topic important?

Being one of the core topics in Life Science, this Unit can be considered to be very important.
Especially with limited time to prepare, students should keep in mind that this specific Unit is
inter-related to units like Unit 2. So in depth preparation of this unit would prove to be beneficial
overall. Every year an average of 4-5 questions from this Unit can be expected spread over both
Part-B & Part-C, majorly in the later Section.

How shall I start?

Methodical approach is of utmost importance for the preparation of this Unit. Being
very exhaustive, the Unit demands to be initiated with topics which are inter-related to
the other sub-topics. This would help save time, and cut on wasting the same on
repetitive topics/ sub-topics. To begin with, I will suggest you to start with the topic of
Cell Signaling. This very topic covers up almost all the essentials of this Unit.
Which books to follow?

You can follow any standard Book for this Unit. For Additional reference, I will suggest the following
books:

Molecular Cell Biology (Lodish):


o This Book is considered sufficient for the preparation of this Unit specifically.
The book puts more emphasis is on complex systems and principles.
Cell and Molecular Biology by Gerald Karp:
o This book can be used to explore core concepts in considerable depth, and it
presents experimental detail which helps to explain and reinforce the concept being
explained. This book offers an exceedingly clear presentation and excellent art
program pertaining to the respective topics of the unit.
Prescotts Microbiology:
o This book can be referred specially for the preparation of the topics related to Host
and Parasite Interaction. It provides a balanced and comprehensive

introduction to all the major areas of microbiology.


Kuby Immunology:
o This textbook is very well written, well illustrated, efficiently organized, and the
study guides and self-tests at the end of each chapter are great study tools.

Time for this Unit:

This Unit consists of exhaustive Topics and sub topics like Cancer, Cell Signaling and Immunology.
Thus in this truncated time schedule, it demands stress and even a slightly longer duration to
prepare thoroughly. So In order to successfully complete this Unit, I would suggest spending 3hrs
a day for the next 2 months. A minimum of 2hrs each day is highly recommended for this Unit.

Prioritize the subtopics:


Preparing this Unit in such a short time requires us to prioritize the sub topics in a systematic
fashion. As told earlier, starting with topics that are larger and more exhaustive is recommended.
Try starting with topics like Cell Signaling followed by topics and sub topics like Cell
Communication. These topics are inter-related with other sub topics of the Unit too. Thus
helping saving a lot of valuable time. You can later on follow on with topics like Cancer and then
Immunology related parts.
Suggestions:
Do not take the risk of missing out any of the topics from this Unit.
Including the sub topics, this Unit renders questions from various parts in various sections.
On top of this, Host Parasite Interaction can also be referred from Unit 2, the topics being
inter-related.

Topic-wise suggestion:
Host Parasite Interaction:

From this section, important sub topics from where questions can be expected are
Toxins. Thorough study of endotoxins and exotoxins is recommended.

Cell Signaling:

Both Part-B and Part-C contains questions from this topic. So topic has to be dealt
carefully in order to score more in these sections. Both conceptual and theoretical
knowledge about this section is necessary.

Cellular Communication:
In this topic Cell adhesion is very important. Mostly questions from the above sub topic
are very common from this section, in both Part-B and Part-C.
Cancer:
The complete section is to be covered in order to score good marks. None of the sub-
topics from this topic can be ignored. It should be kept in mind that this section can be
considered as one whole topic on its own.
Innate & Adaptive Immune System:
Both Part-B and Part-C carries questions from this section. Part-B mostly consists of
questions from sub-topics related to the Cells of the Immune System and their
properties. In Part-C, questions from topics related to the Activation processes,
antibody engineering, antigen-antibody interactions, MHC molecules, antigen
processing & presentation, activation and differentiation of B & T cells, B and T cell
receptors, humoral & cell-mediated immune responses, primary & secondary immune
modulation, the complement system can be
expected
HOST-PARASITE INTERACTION
1. LAL test is used in
a. Quantifying endotoxin
b. Quantifying exotoxin
c. Detecting endotoxin
d. Detecting exotoxin.

2. Which of the following viruses is spread by way of nerves?


a. Rabies
b. Varicella-zoster
c. Herpes simplex
d. All of the above
e. None of the above.

3. Which of the following represents a passive mechanism by which a bacterium can penetrate a host?
a. Attacking the intestinal linings
b. Degrading carbohydrate-protein complexes
c. Disrupting the cell surface
d. Tissue damage caused by other organisms
e. All of the above

4. Expression of the virulence gene of Bordetella pertussis (the pathogen causing whooping cough) is
dependent upon
a. temperature.
b. osmolality.
c. available iron.
d. pH.
e. specific ions.

5. Which of the following is true for exotoxins?


a. Effect on host similar for all
b. Heat stable
c. Produce a fever
d. Strongly immunogenetic
e. Frequently encoded on plasmids

6. True enterotoxins have what effect on the intestinal mucosa?


a. Cause a change in pH
b. Elicit profuse fluid secretion
c. Increase temperature
d. Bind to mucosal cells
e. None of the above

7. Staphylococcal food poisoning is caused by:


a. Ingestion of preformed exotoxin
b. Colonization of a mucosal surface followed by the production of exotoxin
c. Colonization of a wound or abscess followed by local exotoxin production
d. All of the above
e. None of the above

8. All of the following diseases are treated by using toxoid except


a. Tetanus
b. Whooping cough
c. Syphilis
d. Pertussis.

9. All of the following are reportable sexually transmitted diseases EXCEPT ?


a. Gonorrhea.
b. AIDS.
c. Trichomoniasis.
d. Syphilis.
e. All of the above.

10. Most cases of complicated cystitis are caused by


a. E. coli.
b. Enterococcus.
c. Proteus.
d. Klebsiella.
e. Pseudomonas.

11. Which of the following is not a complication of gonorrhea?


a. Sterility
b. Pelvic inflammatory disease
c. Arthritis
d. Meningitis
e. None of these

12. Which of the following pairs are mismatched for syphilis?


a. incubation period - three weeks
b. chancre - indolent
c. antibodies produced, no outward signs - primary phase
d. flu like syndrome - secondary phase
e. None of these
13. Which of the following statements regarding neonatal syphilis is incorrect?
a. Transmitted through placenta during 5th month.
b. Early lesions resemble chancres.
c. 25% of the cases die in utero.
d. Late lesions resemble gummas.
e. None of these

14. Which of the following statements regarding the Rapid Plasma Reagin test for syphilis is TRUE?
a. Uses the Reiter strain of T. pallidum.
b. Uses the Nichol strain of T. pallidum.
c. Almost no biologically false positives.
d. Reacts with Wasserman antibodies of patient.
e. None of the above.

15. All of the following are symptoms of gonorrhea in males EXCEPT?


a. dysuria
b. purulent discharge
c. urethral stricture
d. chronic prostatitis
e. salpingitis

16. A major cause of septic arthritis in adults is?


a. Neisseria gonorrhoeae
b. Treponema pallidum
c. Chlamydia trachomatis
d. Herpes simplex
e. None of these.

17. Most cases of posttransfusion hepatitis are caused by


a. Hepatitis A virus.
b. Hepatitis B virus.
c. Hepatitis C virus.
d. All of these.
e. None of these; modern screening techniques have eliminated posttransfusion hepatitis.

18. All of the following statements about salmonellosis are true except
a. It is a food infection disease.
b. It requires a small infective dose.
c. main symptoms involve GI disturbance and septicemia
d. results in about two million food poisonings annually
e. It is often associated with poultry products.
19. Which of the following does not produce a gastrointestinal disease that includes production of an
exotoxin?
a. Escherichia coli
b. Vibrio cholerae
c. Shigella dysenteriae
d. Salmonella typhimurium
e. all of the above produce exotoxins

20. Treatment of gonorrhea includes all of the following EXCEPT?


a. ampicillin - for penicillin sensitive strains.
b. ceftriaxone - effective against incubating syphilis.
c. spectinomycin prevents post-gonococcal urethritis.
d. ceftriaxone - pharyngeal gonorrhea.
e. All of these are correct treatments.

21. Chemoprophylaxis for meningitis caused by Neisseria includes?


a. Penicillin G.
b. chloramphenicol.
c. streptomycin.
d. rifampin.
e. both a and b are correct.

22. Which of the following is diagnosed by the presence of flagellates in the patient's feces?
a. Balantidiasis
b. Giardiasis
c. Trichinosis
d. Trypanosoma
e. Plasmodium

23. Which of the following causes an acute infection in humans, which is transmitted by cats?
a. Giardia lamblia
b. Balantidium coli
c. Trypanosoma
d. Toxoplasma gondii
e. Entamoeba histolytica

24. Which of the following pairs regarding Helminth infestations is incorrect?


a. Platyhelminths - roundworm.
b. Nematodes - roundworm.
c. Tapeworm - flatworm.
d. Pinworm - roundworm.
e. Fluke - flatworm.

25. The most common cause of traveler's diarrhea is probably


a. Shigella spp.
b. Salmonella typhi.
c. Giardia lamblia.
d. Escherichia coli.
e. Entamoeba coli.

26. Which of the following can be transferred from an infected mother to her fetus across the placenta?
a. Mumps
b. Cytomegalovirus inclusion disease
c. Infectious hepatitis
d. Typhoid fever
e. Enteroinvasive E. coli

27. Treatment of superficial candidiasis includes?


a. chloroquine
b. metronidazole
c. tetracycline
d. griseofulvin (oral)
e. miconazole (topical), nystatin (oral)

28. Which of the following causes an infection of the liver?


a. Salmonella
b. Shigella
c. Hepatitis A virus
d. Vibrio
e. Balantidium

29. "Rice water stools" are characteristic of


a. Salmonellosis.
b. Cholera.
c. Bacillary dysentery.
d. Amoebic dysentery.
e. Tapeworm infestation.

30. Most gastrointestinal infections are treated with


a. Antitoxin.
b. Penicillin.
c. Fluid and electrolytes.
d. Quinacrine.
e. None of these.

31. Which of the following is treated with tetracycline?


a. Staphylococcal food poisoning
b. Vibrio cholerae
c. Infectious hepatitis
d. Travelers diarrhea
e. Shigellosis

32. All of the following are gram-negative rods that cause gastroenteritis except
a. Pseudomonas.
b. Escherichia.
c. Salmonella.
d. Shigella.
e. Vibrio.

33. Systemic mycoses include all of the following except?


a. Ringworm.
b. Blastomyces.
c. Coccidioidomycosis.
d. Histoplasmosis.
e. Cryptococcosis.

34. Treatment of Pseudomonas aeruginosa Urinary Tract Infections includes?


a. Tetracycline
b. Gentamicin
c. Carbenicillin
d. Gentamicin with carbenicillin
e. Erythromycin

35. A primary symptom of this disease is a spasmodic coughing accompanied by a rapid intake of air.
Complications include a collapsed lung and convulsions. This disease is?
a. Pertussis.
b. Legionnaire's disease.
c. Pseudomonas pyoderma.
d. Contagious meningitis.
e. Hepatitis E virus.

36. Which of the following is not a characteristic of P. aeruginosa?


a. Gram-positive rods
b. Aerobic
c. Green pigment
d. Resistant to antibiotics
e. None of these

37. Which of these is not caused by herpes virus?


a. Chickenpox
b. Shingles
c. fever blisters
d. Hepatitis C
e. None of these

38. Thrush and vaginitis are caused by


a. Herpes virus.
b. Chlamydia trachomatis.
c. Candida albicans.
d. Staphylococcus aureus.
e. Streptococcus pyogenes.

39. The greatest single cause of ophthalmia neonatorum in the world is


a. Neisseria gonorrhoeae.
b. Pseudomonas aeruginosa.
c. Chlamydia trachomatis.
d. Haemophilus influenzae.
e. Haemophilus aegyptius.

40. All of the following are true regarding Legionnaire's disease EXCEPT?
a. Transmission is by droplet.
b. An atypical pneumonia.
c. A gram negative rod.
d. Air conditioning units implicated in infection.
e. Prevention is by DPT vaccine.

41. The most common etiology for shigellosis in the northern USA is?
a. Shigella enteriditis.
b. Shigella flexneri.
c. Shigella sonnei.
d. Shigella dysenteriae.
e. Shigella coli

42. Transmission of shigellosis is primarily by?


a. poultry such as chickens.
b. pet turtles.
c. cats spreading it to humans.
d. mice spreading it to humans.
e. humans to humans.

43. The etiologic agent of chickenpox.


a. human herpes virus 1
b. human herpes virus 2
c. human herpes virus 3
d. human herpes virus 4
e. human herpes virus 5

44. Probably the most common cause of infantile diarrhea is?


a. Enteropathogenic E. coli
b. Enterotoxigenic E. coli
c. Enteroinvasive E. coli
d. Shigella dysenteriae
e. Proteus mirabilis

45. This organism ranks second to E. coli as a cause of uncomplicated cystitis.


a. Shigella
b. Salmonella
c. Proteus
d. Vibrio
e. Pseudomonas

46. The etiologic agent of fever blisters.


a. human herpes virus 1
b. human herpes virus 2
c. human herpes virus 3
d. human herpes virus 4
e. human herpes virus 5

47. This organism is a common cause of nosocomial pneumonia.


a. Streptococcus pneumoniae
b. Mycoplasma pneumoniae
c. Haemophilus influenzae
d. Klebsiella pneumoniae
e. Proteus mirabilis

48. This antibiotic is used for UTI caused by Proteus mirabilis.


a. Ampicillin.
b. Cefotaxime.
c. Penicillin.
d. Streptomycin.
e. Cefotaxime with Penicillin.

49. The major cause of meningitis in children under two years of age is?
a. Streptococcus pneumoniae
b. Staphylococcus aureus
c. Streptococcus pyogenes
d. Haemophilus influenzae
e. Yersinia pestis.

50. Erysiphe causes the disease


a. powdery mildews,
b. downy mildews
c. covered smut
d. late blight of potato

51. The coprophilic fungi inhabit


a. dung substratum
b. dead wood
c. decaying leaves
d. food articles.

52. Mycotrophy is the symbiosis of a fungus with


a. bacteria
b. algae
c. bryophytes
d. other fungi

53. Macrocyclic rust is the name given to some fungi,


a. which produces bigger spores
b. where all the five spore stages are produced
c. which completes its life-cycle on a single host
d. which selects many hosts to complete its life-cycle

54. The obligate parasitic fungi absorb their nourishment from the host cells through
a. the surface
b. haustoria
c. appressoria
d. rhizoids

55. Biological specialization is a term used for fungus which :


a. can infect differential hosts,
b. shows host specialization
c. can grow in a variety of substrata
d. are biologically useful

56. White rust of crucifer is caused by:


a. Puccinia,
b. Utilago
c. Cystopus
d. Peziza

57. Coprophilous fungi are growing in


a. grasses,
b. dung,
c. animals,
d. wood

58. A haustorium of a fungus is meant for


a. fixing up to the mycelium to the host,
b. increasing the spread of the disease
c. reproduction of the fungus
d. absorbing nourishment from the host

59. The sexual reproduction of Puccinia graminis is of the type known as


a. somatogamy
b. dikaryotization
c. spermatisation
d. automixis

60. Haustoria are produced in the case of mycelium which is :


a. both intracellular and endoparasitic,
b. ectoparasite,
c. both intercellular and endoparasite
d. either ectoparasitic or intercellular

61. Perfect stage of fungus means:


a. when the fungus is perfectly healthy
b. when it reproduces asexually
c. when it reproduces sexually,
d. when it forms perfect resting spores

62. Early blight of potato is caused by


a. Albugo candida,
b. Phytophthora infestans
c. Alternaria solani

63. Tikka disease of groundnut is caused by


a. Aspergillus
b. Puccinia
c. Cercospora
d. Fusarium

64. The name smut diseases is given to those produced by Ustilago because
a. its mycelium is black in colour
b. it parasitizes cereals
c. the host becomes completely black,
d. the fungus produces black sooty spore masses

65. White rust of crucifers is a pseudo-rust because


a. the disease is not caused by basidiomycetous members
b. the colour of the pustule is not red
c. the disease is seen on crucifers
d. the disease is not seen on wheat

66. Wilt of arhar is caused by


a. Pythium
b. Alternaria
c. Colletotrichum
d. Fusarium

67. The whip smut of sugarcane is caused by


a. Ustilago maydis
b. Ustilago hordei
c. Ustilago scitaminea
d. Ustilago nuda
68. Downy mildews are caused by the members of
a. Erysiphales
b. Taphrinales
c. Ustilaginales
d. Peronosporales

69. The rusts are caused by


a. Ustilaginales
b. Peronosporales
c. Uredinales
d. Erysiphales

70. Fungi occurring on wood are:


a. epibiotic
b. eucarpic
c. epixylic
d. epigean

71. An organism which is normally a saprophyte, but can also become a parasite is called
a. facultative saprophyte
b. partial saprophyte
c. facultative parasite
d. partial parasite

72. Which of the following diseases is caused by a fungus?


a. Tobacco mosaic
b. Turnip yellow mosaic
c. potato spindle tuber
d. black rust of wheat

73. Loose smut of wheat is caused by


a. Ustilago tritici
b. Cystopus
c. Puccinia
d. Aspergillus

74. Obligate parasites are those organisms which


a. are essentially saprophyte but can become parasite
b. are essentialy parasite but can also become saprophyte
c. live only in dead and decaying organic matter
d. live only in living hosts

75. When two host species are required for completion of parasitic fungi life-cycle, this condition is
described as
a. autoecism
b. autotrophic
c. heteroecism
d. heterokaryotic

76. Black rust of wheat is caused by


a. Puccinia graminis
b. Puccinia recondita
c. Puccinia striformis
d. Puccinia glumarum

77. Alternaria solani causes:


a. late blight of potato
b. wart of potato
c. early blight of potato
d. leaf curl of potato

78. Target board effect is caused by


a. Alternaria
b. Colletotrichum
c. Pyricularia
d. Helminthosporium.
e.
79. Mad cow disease (Jacod-Creutzfelt disease) was caused by certain
a. Bacterium
b. Virus
c. Viriod
d. Prion

80. Cancer is induced by certain viruses called


a. Pox viruses
b. Adeno viruses
c. Oncogenic viruses
d. Herpes viruses
e. Muco viruses.
81. Rheumatic heart disease is due to repeated infections of
a. Staphylococcal bacteria
b. Streptococcal bacteria
c. Pneumococcal bacteria
d. All of these.

82. Which of the following statements is correct?


a. Poliomyelitis is a viral disease, whereas measles is due to bacteria.
b. Poliomyelitis is a bacterial disease, and measles is produced by a virus.
c. Poliomyelitis and measles are viral disease.
d. Both poliomyelitis and measles are bacterial disease.
e. Both are cancerous.

83. Diphtheria and polio can be prevented by


a. Using proper sanitation facilities
b. Drinking boiled water
c. Immunization with toxoids/vacancies
d. Proper disposal of wastes
e. Taking healthy food.

84. Agrobacteirum tumefaciens produces crown gall diseases in


a. Gymnosperms
b. Monocotyledons
c. Dicotyledons
d. Pteridophytes
e. Angiosperms.

85. Ringworms is and infection caused by


a. A nematode worm
b. Bacteria
c. Dermatophytons
d. Viruses
e. Mycoses.

86. Tetrodotoxin is a product of


a. The house lizard
b. A snake
c. The puffer fish
d. Cuttle fish
e. A house flies.
87. Saxitoxin is produced by
a. Some marine algae
b. Marine bacteria
c. A marine dinoflagellate
d. Clams and Mussels
e. Marine fungus.

88. Which disease is arthropod borne


a. Mastitis
b. Foot and Mouth disease
c. Salmonella
d. Rinderpet
e. Blue tongue.

89. Salmonellosis is caused by


a. Salmonella Dublin
b. S. typhimurium
c. Both (a) and (b)
d. None of these.

90. Which is not a protozoan disease


a. Coccidiosis
b. Babesiosis
c. Theileriasis
d. Rinderpet

91. Which disease is called as splenic fever?


a. Anthrax
b. Tuberculosis
c. Mastitis
d. Brucellosis.

92. Which of the following extracellular enzymes produced by Group A streptococci is called "spreading
factor," an enzyme important in skin and soft tissue infection?
a. Streptokinase
b. Hyaluronidase
c. M Protein
d. Deoxyribonuclease C
e. None of the above
93. Which of the following statements about Group B streptococci (Streptococcus agalactiae) are not
correct?
a. They are important causes of toxic strep syndrome.
b. They are frequent colonizers of the female genital tract.
c. Screening for this pathogen during pregnancy has reduced the incidence of neonatal sepsis.
d. These organisms are b-hemolytic.
e. They are important causes of urinary tract infections and bacteremia in elderly and diabetic
adults.

94. What is the O antigen of Enterobacteriaceae?


a. Cell surface polysaccharide
b. A channel controlling substance taken into the organism
c. A flagellar protein
d. A peptidoglycan matrix important for cellular rigidity
e. Cell wall lipopolysaccharide

95. Which of the following virulence factors of E. coli is important for attachment to host epithelial cells
in the pathogenesis of urinary tract infections?
a. Aerobactin
b. Alpha hemolysin
c. Urease
d. K1 antigen
e. Pili

96. Which of the following statements regarding Enterotoxigenic E. coli are CORRECT?
a. They are important causes of traveler's diarrhea.
b. Transmission occurs from ingestion of contaminated food and water.
c. Disease is caused by production of one or both of two types of enterotoxins.
d. None of the above are correct.
e. All of the above are correct.

97. Characteristics of a bacterial capsule include:


a. All bacteria have one
b. It is composed of peptidoglycan
c. It is an important mechanism for protecting a bacteria against ingestion by PMNs
d. It is what causes the gram stain reaction

98. Which of the following bacterial pathogenic enzymes does not facilitate degradation of tissue?
a. Streptokinase
b. Coagulase
c. Hyaluronidase
d. Collagenase
e. All are correct

99. What is not an example of parental portal entry?


a. Needle stick
b. Mosquito bite
c. mucous membrane
d. All are correct.

100. What does the A in AB toxin stand for?


a. Active
b. Agglutination
c. Adhesion
d. Accumulation
e. None of these is correct.

101. Which of the following microorganisms produces endotoxin?


a. E. coli
b. Staphylococcus spec.
c. Streptococcus spec.
d. C. albicans
e. All of these.

102. Which bacteriu m produces the hemolysin streptolysin?


a. S. pyogenes
b. S. aureus
c. C. diphteriae
d. C. botulinum
e. C. tetani

103. Which bacterial enzyme digests fibrin clots?


a. Kinase
b. Coagulase
c. Hyaluronidase
d. Collagenase
e. None of the above.

104. With which of the following bacteria is an infant colonized first after birth?
a. Lactobacilli
b. E. coli
c. Streptococci
d. All of the listed
e. None of the listed

105. Which port of entry requires the highest dose of B. anthracis endospores for infection?
a. Skin
b. respiratory tract
c. eyes
d. gastrointestinal tract

106. Which bacterium can be the source of a neurotoxin that belongs to the AB toxin family and
causes a flaccid paralysis?
a. C. diphteriae
b. C. botulinum
c. C. tetani
d. V. cholera
e. S. aureus

107. Streptococcus pyogenes uses all of the following enzymes to spread throughout the body except
a. DNAse
b. Strepokinase
c. Hyaluronidase
d. Coagulase
e. All of these enzymes are used.

108. Localized Impetigo is typically caused by:


a. Staphylococcus aureus
b. Pseudomonas aeruginosa
c. Varicella Zoster virus
d. Streptococcus pyogenes
e. Propionibacterium acnes

109. Which of the following are involved in bacterial infections of the eye?
a. Haemophilus influenza
b. Acanthamoeba
c. Herpes Simplex type 1
d. None of these
e. All of these.
110. Which of the following bacteria may lead to an infection with blue-green pus?
a. Streptococcus pyogenes
b. Propionibacterium acnes
c. Staphylococcus aureus
d. Pseudomonas aeruginosa
e. Both C and D

111. Which of the following are part of the normal microbiota of the skin?
a. Staphylococci
b. Micrococci
c. Corynebacteria
d. Propioni bacteria
e. All of these.

112. Which bacteria may cause Ritters disease provided that they have been infected by a
phage?
a. Streptococcus pyogenes
b. Staphylococcus aureus
c. Pseudomonas aeruginosa
d. Clostridium perfringens
e. All of these.
113. Which of the following is responsible for causing skin lesions?
a. Streptococci
b. Pseudomonas
c. Staphylococci
d. A and C
e. All of these.

114. Which of the following microorganisms can be responsible for both corneal infection and
inversion of eyelashes?
a. C. trachomatis
b. Herpes simplex
c. N. gonorrhoeae
d. Acanthamoeba
e. None of these.

115. Which of the following is involved in bacterial infections of the eye?


a. Haemophilus influenza
b. Acanthamoeba
c. Herpes Simplex type 1
d. None of these.
e. All of these.

116. Three family members developed double vision, swallowing difficulty, and speech
difficulty 2 days after eating dinner together. Which microorganism listed below produces an
exotoxin that is very specific in blocking transmission of cholinergic nerves?
a. Bacillus anthracis
b. Bacillus cereus
c. Clostridium botulinum
d. Clostridium difficile.

117. Below are the events of pathogenesis seen during Vibrio cholerae infection?
I. Reduction of A subunit to A1 and A2; Inhibition of the intrinsic GTPase activity.
II. Binding of cholera toxin to GM1 gangloside of the epithelial cells of the intestinal
tract.
III. Increase in cAMP and efflux of electrolyte and water into the lumen.
IV. Expression of more GM1-Like ganglioside by the action of neuraminidase.
V. Internalization of toxin by endosomal pathway and retrograde movement of toxin.
Which is the correct order?
a. I, II, III, IV, V
b. I, II, III, V, IV
c. II, IV, V, I, III
d. II, V, IV, I, III

118. Match the following


Toxin Mode of Action
P. Shiga Toxin I. ADP-ribosylation
of Gi
Q. Pertussis II. Cleaves SNAREs and
Toxin prevent the release of
neurotransmitter
R. Tetanus Toxin III. cleaves certain
MAP kinase kinases
S.Anthrax Toxin IV. Inhibition of EF-1
a. P- IV, Q-I, R-II, S-III.
b. P-IV, Q-I, R-III, S-II
c. P-I, Q-IV, R-II, S-III
d. P-I, Q-IV, R-III, S-II

119. Pathology of malaria may include which of the following is/are true
A. Anaemia due to red cell destruction.
B. Red blood cell sequestration.
C. Fever correlated with synchronous parasite release.
D. Recurrent disease due to cryptic infections in the liver
E. Cerebral inflammation due to over production of TNF
a.A, B and C
b.B, C and D
c. A, B,C and E
d.A, B, C, D and E.

120. The Epstein-Barr virus has a proven positive association with the following conditions,
EXCEPT
A. carcinoma of the cervix
B. infectious mononucleosis
C. human T cell lymphoma
D. Burkitts lymphoma
E. Undifferentiated nasopharyngeal carcinoma
a. A and C
b. B only
c. D and E
d. A and B.

121. It is extremely difficult to eradicate staphylococcal infection in the presence of a foreign


body. Thus, infected artificial joints and infected intravenous lines often must be removed in
conjunction with optimal intravenous anti-staphylococcal therapy with oxacillin or nafcillin
(sometimes in combination with rifampin) in order to eradicate such staphylococcal infections
(and these attempts are not always successful). Staphylococcal infection of bone may persist for
many years despite what should be optimal antimicrobial therapy. The most likely reason for this
tenacity of staphylococcal osteomyelitis is:
a. The Staphylococcal pilus gene is up-regulated by oxacillin
b. Fragments of dead bone, called "involucrum" may act as a foreign body, and may have to
be removed for optimal eradication of the staphylococcal infection.
c. Staphylococci may be perceived by the patient's immune system as a positive factor,
because the staphylococci produce an extracellular enzyme, obtundokinase, which
makes the patient more alert.
d. Staphylococci are such an integral part of our normal flora that they are treated as "self"
rather than "non-self", and no immune reaction to them occurs.

122. Which infection cycle is listed in the correct order?


a. Portals of entry, damage to cells, penetration and evasion, exit
b. Portals of entry, exit, damage to cells, penetration and evasion
c. Portals of entry, penetration and evasion, damage to cell, exit
d. Penetration and evasion, portals of entry, damage to cells, exit
e. None of the above is correct.

123. What exotoxin(s) can S. aureus produce?


a. TSST (hyperstimulates immune system)
b. Enterotoxin (similar to V. cholerae toxin)
c. Neurotoxin (AB toxin that causes a flaccid paralysis)
d. Both A) and B)
e. All A), B), and C)

124. Which of the following is mismatched?


a. Toxin & poisonous substance
b. Toxoid & inactivated toxin
c. Toxemia & presence of toxin in the blood
d. Toxigenicity & ability to induce toxic reaction in the host
e. All of the above are correct.

125. Which of the following microbial skin diseases is mismatched?


a. Parasitic & scabies .
b. Viral & shingles.
c. Fungal & candidiasis.
d. Bacterial & roseola.

126. All of the following are characteristics of the 5th disease except:
a. Caused by human parvovirus B19
b. Presence of Kopliks spots
c. Mild flu like symptoms
d. Distinct skin rash that appears liked slapped face
e. Rash fades away slowly.

127. Which statement is false regarding Corynebacterium diphteriae?


a. Prevented by immunization with diphteria toxoid
b. Infection may spread systemically resulting in kidney and heart failure.
c. Only lysogenized bacteria can produce the toxin.
d. Diphtheria membrane in the throat is composed of fibrin, dead tissue, and bacteria.
e. The bacteria are gram+ pleomorphic rods.

128. Which of the following statements is true regarding Bordetella pertussis?


a. Has as main pathogenic factor a thick capsule.
b. May secrete tracheal cytotoxin which locally damages ciliated respiratory cells.
c. The disease can be prevented with an acellular cell fragment vaccine.
d. The disease it causes has three stages: catarrhal, paroxysmal, and convalescent.
e. All the above statements are true

129. What are important factors in the pathogenesis of hemolytic uremic syndrome?
a. EHEC O157:H7 produces Shiga-like toxin that is absorbed and transported to the small
blood vessels.
b. EHEC O157:H7 injects proteins into the host cell via a type III secretion apparatus.
c. EHEC O157:H7 produces urease.
d. All of these.
e. None of these.

130. One of your patients is very sick. She has recently developed mucoid diarrhea, and a biopsy
shows signs of bacteria attaching via pili bundles to enterocytes. The bacteria test positive for E. coli.
Which strain designation would you choose in this case of diarrhea?
a. Enteroinvasive
b. Enteroaggregative
c. Enteropathogenic
d. Enterotoxigenic
e. Enterohemorrhagic
ANSWER KEYS:

1.A
2.D
3.D
4.A
5.E
6.B
7.A
8.C
9.C
10.C
11.D
12.C
13.B
14.D
15.E
16.A
17.C
18.B
19.D
20.C
21.D
22.B
23.D
24.A
25.D
26.B
27.E
28.C
29.B
30.C
31.B
32.A
33.A
34.C
35.A
36.A
37.D
38.C
39.C
40.E
41.C
42.E
43.C
44.A
45.C
46.A
47.D
48.A
49.D
50.A
51.A
52.D
53.B
54.B
55.B
56.C
57.B
58.D
59.B
60.D
61.D
62.B
63.C
64.D
65.A
66.D
67.C
68.D
69.C
70.C
71.C
72.D
73.A
74.D
75.C
76.A
77.C
78.A
79.D
80.C
81.B
82.C
83.C
84.C
85.C
86.C
87.C
88.E
89.C
90.D
91.A
92.B
93.A
94.E
95.E
96.E
97.C
98.A
99.D
100.A
101.E
102.A
103.E
104.A
105.A
106.B
107.E
108.A
109.A
110.D
111.E
112.B
113.E
114.A
115.E
116.C
117.C
118.A
119.D
120.A
121.B
122.C
123.E
124.E
125.D
126.E
127.B
128.E
129.A
130.C
Cell Signaling and Cellular Communication

1. Calmodulin is
a. A non-specific kinase
b. A protein that binds calcium
c. A second messenger
d. An activator of nitric oxide synthase
e. A protein channel that facilitates the influx of calcium

2. A cell secretes growth factors that have a slow diffusion rate, and interacting only
with other cells in the immediate area is an example of _______________ signaling
a. exocrine
b. synaptic
c. paracrine
d. cell contact

3. Which of the following is stored in cells of the immune system and released during an
allergic reaction?
a. nitric oxide
b. a growth factor
c. histamine
d. a prostaglandin
e. a neurotransmitter

4. Which of the following statements concerning receptors is FALSE?


a. They are found inside the cell and on the cell surface.
b. They are proteins or glycoproteins.
c. They are also called ligands.
d. They are highly selective.
e. They are activated by binding to a signaling molecule.

5. A receptor on the cell surface usually has several domains. The function of the
external domain is:
a. transmitting the signal to the inside of the cell.
b. holding the receptor within the membrane.
c. attaching the receptor to the DNA.
d. functioning as an enzyme.
e. binding the signaling molecule.

6. The toxin that impairs the function of G protein is


a. Cholera Toxin
b. Bungarotoxin
c. Saxitoxin
d. Diphtheria Toxin

7. Identify the correct order in which the extracellular signals are transmitted
a. Adenylate cyclase cAMP - Protein kinase A
b. Protein Kinase A Adenylate Cyclase cAMP
c. Protein Kinase cAMP adenylate cyclase
d. cAMP Adenylate cyclase Protein Kinase A
8. Which of the following hormones passes through the cell membrane, binds to its
intracellular receptors and activate it?
a. Estrogen
b. Norepinephrine
c. Acetylcholine
d. Glucagon
9. Which of these hormones uses cAMP as a second messenger?
a. testosterone
b. cortisol
c. insulin
d. epinephrine
10. The three most common cell junctions in animals are:
a. adhesive junctions, tight junctions, gap junctions
b. adhesive junctions, tight junctions, plasmodesmata
c. tight junctions, plasmodesmata, adhesive junctions
d. gap junctions, tight junctions, plasmodesmata.
11. The reason urine does not seep out of the bladder is due to the role of:
a. adherens junctions
b. hemidesmosomes
c. gap junctions
d. tight junctions
12. Desmosomes and hemidesmosomes are anchored in the cytoplasm by:
a. desmocollins
b. actin fibers
c. collagen
d. tonofilaments.
13. Gap junctions consist of channels formed by:
a. hydrophilic, connexons
b. hydrophobic, cadherins
c. hydrophobic, connexons
d. hydrophilic, cadherin.
14. Which receptor is an ion channel?
a. Acetylcholine receptor
b. EGF receptor
c. Corticosteroid receptor
d. GH receptor
15. A mutant G alpha protein with increased GTPase activity would
a. Not bind to GTP
b. Not bind to GDP
c. Show increased signaling
d. Show decreased signaling
16. Parkinsons disease is characterized by cell death
a. Substantia nigra
b. Motor cortex
c. Frontal cortex
d. Spinal cord

17. Under which of the following situations would receptor down-regulation most likely
occur?
a. The concentration of a neurotransmitter is too low.
b. The concentration of a hormone is too high.
c. The number of receptors in the plasma membrane is too low.
d. The number of G proteins is too high.
e. The cell is unable to manufacture cyclic AMP.

18. As a result of receptor up-regulation:


a. the number of genes that code for a receptor increases.
b. the sensitivity of a cell to a hormone decreases.
c. the number of receptors decreases.
d. the concentration of hormone molecules in the blood increases.
e. a hormones signal is amplified.

19. In contrast to a G protein-linked receptor and an enzyme-linked receptor, a channel-


linked receptor:
a. is located on the cell surface.
b. is composed of seven alpha helices.
c. couples signaling molecules to signal transduction pathways.
d. is a ligand-gated channel.
e. functions as a tyrosine kinase.

20. When acetylcholine binds its receptor on the surface of a muscle cell, which of the
following happens next?
a. G protein is activated.
b. Tyrosine kinase is activated.
c. Tyrosine is phosphorylated.
d. A neurotransmitter crosses the synapse.
e. A sodium gate opens.

21. The outer part of a G protein receptor binds ______, and its inner part binds_______.
a. the signaling molecule; tyrosine kinase
b. the signaling molecule; a G protein
c. G protein; tyrosine kinase
d. G protein; an ion channel
e. an ion channel; a G protein

22. An example of a signaling molecule that binds with a receptor on the cell surface is:
a. insulin.
b. ecdysone.
c. Vitamin D.
d. Vitamin A.
e. nitric oxide.

23. Which of the following statements concerning intracellular receptors is FALSE?


a. Their ligands are hydrophobic.
b. Most are transcription factors.
c. Some are located in the cytosol.
d. After binding their ligand, they move out of the cell.
e. Some are bound to DNA.

24. When a receptor binds to its G protein, which of the following happens next?
a. The signaling molecule binds to the receptor.
b. The G protein activates an enzyme.
c. The three G protein subunits come together.
d. GTP is replaced by GDP.
e. GDP is replaced by GTP.

25. Which of the following statements concerning G protein is FALSE?


a. Its subunits can separate.
b. One subunit is a GTPase.
c. One subunit binds GTP.
d. One subunit binds GDP.
e. One subunit binds ATP

26. A molecule that is a first messenger but not a second messenger is:
a. cyclic AMP.
b. G protein.
c. adenylyl cyclase.
d. protein kinase.
e. acetylcholine.

27. An example of a second messenger is:


a. protein kinase A.
b. an ion channel.
c. cyclic AMP.
d. GABA.
e. insulin.
28. Adenylyl cyclase catalyzes the reaction in which:
a. a protein is phosphorylated.
b. ATP is converted to cAMP.
c. GTP is converted to GDP.
d. PIP2 is split.
e. calcium ions bind calmodulin.

29. In the signal transduction pathway involving cAMP, once cAMP is formed, it the
activates:
a. protein kinase A.
b. adenylyl cyclase.
c. G protein.
d. protein kinase C.
e. IP3.

30. A protein kinase catalyzes the reaction in which:


a. a protein loses a phosphate group.
b. a protein is phosphorylated.
c. G protein is activated.
d. ATP is converted to cAMP.
e. GTP is converted to GDP.

31. In the cAMP signal transduction pathway, once a protein kinase is activated which of
the following happens next?
a. G protein is activated.
b. Adenylyl cyclase is activated.
c. GDP is replaced by GTP.
d. cAMP is converted to ATP.
e. The protein kinase activates a cellular response.

32. Phospholipase C catalyzes the reaction in which:


a. IP3 is converted to DAG.
b. PIP2 is split.
c. a protein kinase is activated.
d. calcium ions are released from the ER.
e. DAG is converted to PIP2.

33. In the phospholipase C signal transduction pathway, which are second messengers?
a. phospholipase C and G protein
b. PIP2 and IP3
c. cAMP and IP3
d. phospholipase C and protein kinase A
e. IP3 and DAG

34. What activates protein kinase C?


a. IP3
b. DAG
c. cAMP
d. PIP2
e. calcium ions

35. In the IP3 pathway, what is the function of IP3?


a. activate protein kinase C
b. activate phospholipase C
c. phosphorylate a protein
d. bind calcium channels in the ER
e. activate adenylyl cyclase

36. Which of the following statements concerning calmodulin is FALSE?


a. It activates protein kinases.
b. It activates phosphatases.
c. It is a protein.
d. It is found in all eukaryotic cells studied.
e. It binds a maximum of three calcium ions.

37. Most enzyme-linked receptors are:


a. phospholipases.
b. adenylyl cyclases.
c. tyrosine kinases.
d. G proteins.
e. ion channels.

38. Which of the following statements concerning Ras proteins is FALSE?


a. They are inactivated when phosphorylated.
b. They are G proteins.
c. They are active when bound to GTP.
d. They are activated by tyrosine kinase receptors.
e. They include MAP kinases.

39. The main signaling cascade for cell division and differentiation is the _______cascade.
a. phospholipase C
b. MAP kinase
c. phosphoinositol
d. cAMP
e. calcium-calmodulin

40. You conduct an experiment in which you inactivate Ras proteins in fibroblast cells.
Which of the following is the most immediate consequence of this procedure?
a. The fibroblasts synthesized excess G protein.
b. The fibroblasts synthesized excess DAG.
c. The fibroblasts no longer responded to calmodulin.
d. The fibroblasts no longer synthesized DNA in response to growth factors.
e. The fibroblasts divided uncontrollably.

41. The function of scaffolding proteins is to:


a. phosphorylate proteins in a cascade.
b. dephosphorylate proteins in a cascade.
c. convert ATP and GTP to cAMP and cGMP, respectively.
d. regulate the MAP kinase pathway.
e. stimulate the release of calcium ions from the ER.

42. Choanoflagellates have been important for the study of cell communication evolution
because choanoflagellates have:
a. signal transduction pathways unlike those in other bacteria.
b. signaling molecules similar to those of other protists.
c. signal transduction pathways that interfere with the cells they infect.
d. protein kinases similar to those in animals.
e. G proteins different from those in prokaryotes.

43. As compared to peptide hormones, steroid hormones take more time to activate a
cellular response because
a. steroid hormones show non-specific binding with diverse sets of receptors
b. steroid hormones act through a receptor which is a transcription factor
c. cells that respond to steroid hormones are dormant in nature
d. peptide hormones are not transported through plasma while steroid
hormones are

44. Which amino acid is phosphorylated in bacterial proteins?


a. Arginine
b. Cysteine
c. Lysine
d. Histidine

45. Which enzyme is activated by phosphorylation?


a. Acetyl-CoA carboxylase
b. Fructose-1,6-bisphosphatase
c. Glycogen synthase
d. Pyruvate kinase
e. Fructose-2,6-bisphosphatase

46. All of the following statements about monomeric G proteins are true
EXCEPT:
a. They are regulated by GTP-GDP exchange proteins.
b. They are regulated by GTPase activating proteins.
c. They regulate enzymes that synthesize cGMP.
d. They regulate vesicle formation.
e. They regulate vesicle fusion.

47. All of the following are known to be part of a signal transduction


cascade EXCEPT
a. phosphorylation of fibronectin
b. dissociation of the components of a hetero- trimeric G-protein
c. enzymatic breakdown of phosphatidyl inositol bisphosphate (PIP2 )

d. elevation of intracellular [Ca2+]


e. activation of cGMP phosphodiesterase

48. The overall adenylate cyclase hormonal scheme may be divided into two parts. the
first part includes those steps involved in the activation of the adenylate cyclase; the
second part includes those steps after the activation of the adenylate cyclase. The
correct order of events involving the second part of the adenylate cyclase hormonal
scheme would include
a. Adenylate cyclase generates Insp3 from PIP2 ; Insp3 binds to regulatory
subunits of calmodulin-dependent protein kinases; catalytic and regulatory
subunits dissociate; the catalytic subunits phosphorylates proteins
b. Adenylate cyclase generates cAMP from ATP; cAMP binds to catalytic
subunits of cAMP-dependent protein kinases; catalytic and regulatory
subunits associates; the catalytic subunits phosphorylates proteins
c. Adenylate cyclase generates cAMP from ATP; cAMP binds to regulatory
subunits of cAMP-dependent protein kinases; catalytic and regulatory
subunits dissociate; the catalytic subunits phosphorylates proteins
d. Adenylate cyclase generates cGMP from GTP; cGMP binds to regulatory
subunits of cGMP-dependent protein kinases; catalytic and regulatory
subunits dissociate; the catalytic subunits phosphorylates proteins
e. Adenylate cyclase generates cGMP from GTP; cGMP binds to catalytic
subunits of cGMP-dependent protein kinases; catalytic and regulatory
subunits associates; the catalytic subunits phosphorylates proteins

49. Estrogen and testosterone are steroid hormones, and are most likely to bind to:
a. G-protein linked membrane receptor
b. membrane ion channel
c. enzyme linked membrane receptor
d. cytoplasmic receptors

50. You have recently identified a molecule that you believe to be a ligand associated with
a signal transduction mechanism. All you know about this ligand is that chemically it
is hydrophilic. As a result, you expect it to interact with its receptor
a. on the cytoplasmic side of the plasma membrane.
b. within the nucleus of the cell.
c. at the outer cell surface.
d. within the cytoplasm of the cell.
e. all of the above

51. Which G protein below is associated with the formation of inositol triphosphate?
a. Gt
b. Gp
c. Gs
d. Gi
e. None of the above

52. You are testing a drug that might be marketed as an anti-inflammatory agent. It
would be a good choice if it possessed which of the following responses?
a. stimulates the release of prostaglandins
b. blocks an enzyme associated with arachidonic acid production
c. blocks adenylate cyclase activity
d. stimulates the activation of platelets
e. all of the above

53. One of the ways that cells communicate with each other is by secretion of various
molecules. The secreted molecule is known as
a. a receptor molecule
b. an anticodon
c. a spectrin tetramer
d. an integrin
e. a signaling molecule

54. Which of the following proteins does activated Ras primarily target?
a. Rb
b. Adenylate cyclase
c. Cyclin D
d. Raf
e. Calmodulin

55. How does nitroglycerin helps relieve constriction of coronary arteries?


a. Nitroglycerin inhibits cGMP-specific phosphodiesterase
b. Nitroglycerin inhibits the production of nitric oxide
c. Nitroglycerin activates adenylyl cyclase
d. Nitroglycerin elicits release of nitric oxide

56. The signal from steroid hormone receptors is amplified by direct


a. activation of gene transcription by the ligand receptor complex.
b. activation of a protein kinase cascade by the ligand receptor complex.
c. stimulation of cAMP production by the ligand receptor complex.
d. both A and B
e. choices A, B, and C
57. Glycogen breakdown is promoted through which of the following signal transduction
mechanisms?
a. Calcium/InsP3 pathway
b. receptor-bound tyrosine kinase activity
c. cAMP-mediated action
d. All of the above
e. None of the above

58. When nitric oxide is used in the dilation of smooth muscle cells, which of the
following would interfere with the relaxation of the smooth muscle cells? Inhibitors
of
a. nitric oxide.
b. phospholipase C
c. guanylyl cyclase.
d. calmodulin.
e. all of the above

59. Which of the following statements about TGF- signaling is true?


a. Activated type I receptors phosphorylates particular receptor-mediated
Smads
b. TGF-type II receptors phosphorylate the adenylate cyclase upon ligand
binding
c. TGF- receptors are tyrosine kinase receptors
d. Smads target kinases to the mitochondria

60. With regard to hormones, responses occurring over the shortest distance would be
classified as
a. paracrine.
b. exocrine.
c. endocrine.
d. Choices A, B, and C all act over the same distance.

61. Which G protein below is associated with the synthesis of cAMP?


a. Gs
b. Gi
c. Gt
d. Gp
e. None of the above

62. Which of the following is the ligand for the ryanodine receptor channels?
a. acetylcholine
b. inositol-1,4,5-triphosphate
c. diacylglycerol
d. calcium
e. vitamin D
63. The endogenous GTPase activity of G-proteins serves to:
a. synthesize cGMP as a second messenger.
b. hydrolyze GTP returning the G protein to a pre-stimulated level of activity.
c. stimulate the activity of enzymes by producing energy.
d. synthesize GTP as an energy source.

64. Which of the following statements is true about aspirin?


a. Aspirin activates phospholipase A2
b. Aspirin inhibits cyclooxygenase activity
c. Aspirin activates cyclooxygenase activity
d. Aspirin inhibits phospholipase A2

65. Which of the following statements is true about sildenafil (Viagra)?


a. Viagra primarily inhibits nitric oxide production
b. Viagra primarily activates cGMP-specific phosphodiesterase
c. Viagra primarily activates guanylyl cyclase
d. Viagra primarily inhibits cGMP-specific phosphodiesterase
e. Viagra primarily activates adenylyl cyclase

66. In a fight-or-flight response, epinephrine released by the adrenal glands binds to a


membrane receptor on muscle cells. Which of the following events result from ligand
binding?
a. G-protein is activated by binding GTP and causing adenylyl cyclase to produce
cAMP.
b. Glucose produced from glucose-1-P stimulates glycolysis, producing energy
for muscle contraction.
c. Protein kinase A activates phosphorylase kinase, which transfers a phosphate
that activates glycogen phosphorylase. This leads to the breakdown of
glycogen and the production of glucose-1-phosphate.
d. The second messenger, cAMP, activates protein kinase A which inhibits
glycogen synthetase blocking glycogen synthesis.
e. All of the above.

67. Which of the following statements is true about caffeine?


a. Caffeine primarily inhibits phospholipase C
b. Caffeine primarily inhibits adenylyl cyclase
c. Caffeine primarily inhibits phosphodiesterase
d. Caffeine primarily activates phosphodiesterase
e. Caffeine primarily activates adenylyl cyclase

68. Which of the following statements is true?


a. Binding of norepinephrine to the-adrenergic receptors stimulate
contraction of smooth muscles that regulate blood flow to visceral organs.
b. Binding of EGF to the-adrenergic receptors stimulate contraction of
smooth muscles that regulate blood flow to visceral organs.
c. Binding of norepinephrine to the-adrenergic receptors stimulate
contraction of smooth muscles that regulate blood flow to visceral organs.
d. Binding of EGF to the-adrenergic receptors stimulate contraction of
smooth muscles that regulate blood flow to visceral organs.
e. None of the above

69. Norepinephrine stimulation of G proteins results in an increased intracellular


concentration of calcium. This increase is not reversed by the extracellular calcium
chelator EGTA. Which of the following is the most likely source of the increased
intracellular calcium concentration?
a. Golgi complex
b. Mitochondria
c. Plasma membrane
d. Endoplasmic reticulum
e. Nucleus

70. How does cholera toxin induce the cell to secrete salts and fluid into the gut?
a. It inhibits Gi
b. Chemically modifies adenylyl cyclase directly so that it can no longer
synthesize cAMP
c. Chemically modifies Gs so that it can no longer hydrolyze GTP
d. Cholera toxin mimics cAMP and binds to cAMP targets

71. The phosphorylated form of glycogen phosphorylase


a. can be activated in muscle in the presence of calcium
b. is inactivated by secretin
c. is inactive
d. does not exist
e. is active

72. Which of the following diseases is caused by a mutation of the fibroblast growth
factor receptor 3 (FGFR-3) gene?
a. Cholera
b. Achondroplasia
c. Sickle cell anemia
d. Whooping cough
e. Tay-Sachs

73. Prostaglandins act on G protein-linked receptors.


a. inhibit phospholipase C.
b. stimulate adenylate cyclase.
c. both A and B
d. choices A, B, and C
74. Signaling between cells usually results in the activation of protein _______.
a. lipases
b. nucleases
c. proteases
d. kinases

75. Which G protein below is associated with the prevention of cAMP synthesis?
a. Gi
b. Gp
c. Gt
d. Gs
e. None of the above

76. For a target receptor located within the cytoplasm of a particular cell, the ligand that
binds the receptor must be
a. an ion.
b. nonpolar.
c. polar.
d. hydrophilic.
e. large.

77. Hemidesmosomes are structures found between


a. two adjacent plant cells
b. two adjacent animal cells
c. cell and extracellular matrix
d. within a bacteria

78. An important link between receptor tyrosine kinases and the activated intracellular
signaling cascades is provided by
a. Cyclic AMP
b. Ras protein
c. CREB protein
d. Inositol triphosphate
e. Diacylglycerol

79. Kinases and phosphatases are essential in the cell because they
a. help turn proteins "on and off" through changes in phosphorylation status.
b. are enzymes that destroy damaged proteins.
c. destroy second messengers, thereby turning off a signal transduction
pathway.
d. sense short poly A tails in mRNA as a signal to degrade the mRNA.
e. all of the above
80. Which of the following proteins can directly interact with epidermal growth factor
(EGF) receptors AFTER autophosphorylation?
a. GRB2
b. Ras
c. Phospholipase C
d. (A & C) Both GRB2 and Phospholipase C
e. None of the above

81. The role of the intracellular protein calmodulin in various human diseases such as
glioblastoma, macular disease, Alzheimer's disease, Huntington's disease, and
Parkinson's disease has been investigated. The major function of calmodulin in the
cell is to act as a
a. Modulator of cAMP binding
b. Calcium binding protein
c. Regulator of adenylate cyclase
d. Calcium pump
e. Protein phosphatase

82. Nitroglycerin causes the relaxation of vascular smooth muscle through


a. Cyclic AMP
b. Guanylate cyclase
c. G-protein linked receptors
d. Phospholipase C
e. Ras

83. G protein-adenyl cyclase activity has been shown, in some cases, to be associated
with certain diseases. In the case of cholera, which of the following is NOT correct?
a. Cholera toxin alters Gs so that it no longer hydrolyzes GTP.
b. Vibrio cholera bacteria colonize the gut.
c. Cholera toxin is secreted by these bacteria.
d. The cholera toxin causes the cells of the gut to take in salts and fluids.
e. The toxin, by altering Gs, keeps intracellular cAMP levels high and doesn't
allow them to decrease.

84. How is nitic oxide synthase activated to produce nitric oxide?


a. by guanyly cyclase
b. by activated calmodulin
c. by protein kinase A
d. by tyrosine kinase receptor
e. by protein kinase C

85. ERK is a member of ------ family


a. Src
b. MAPK
c. PKB
d. GSK3
86. What is the energy source for transport of molecules into and out of the nucleus?
a. ATP hydrolysis within the cytoplasm
b. GTP hydrolysis within the cytoplasm
c. ATP hydrolysis within the nucleus
d. GTP hydrolysis within the nucleus

87. Which among the following statements is incorrect in the context of transcription factor
NFkB?
a) It mediates antiviral response
b) It resides in the nucleus
c) It is a member of the Rel family of proteins
d) Its activity is blocked by IkB

88. Identify the protein that is functionally different from the other three
a. HADC
b. HAT
c. P300
d. JNK

89. Which of the following is not an immediate event after stimulation of a transmembrane
receptor by a growth factor?
a) GTP hydrolysis
b) Activation of tyrosine kinase
c) Receptor dimerisation
d) Ubiquitination and degradation of targeted proteins

90. When a platelet contacts a damaged blood vessel, it is stimulated to release thromboxane
A2. Thromboxane A2 in turn stimulates vascular spasm and attracts additional platelets to
the injured site. In this example thromboxane A2 is acting as a
a) Neurotransmitter
b) Transcription factor
c) Protein kinase
d) Local regulator

91. Testosterone and estrogen are lipid soluble signal molecules that cross the plasma
membrane by simple diffusion. If these molecules can enter all cells, why do only specific
cells respond to their presence?
a. Non target cells possess enzymes that immediately degrade the molecules as they
enter the cell
b. Non target cells lack the intracellular receptors that, when activated by the signal
molecule, can interact with genes in the cells nucleus
c. Non target cells lack the inactive enzymes that the signal molecules activate
d. In nontarget cells these signal molecules cross the membranes of the endoplasmic
reticulum and are captured by vescicles.
92. If a modified form of GTP that cannot be enzymatically converted to GDP were added to a
culture of cells, the likely result would be
a. The inactivation of ligand ion channels
b. The inactivation of G-protein linked signalling pathways
c. That the activated G protein would remain locked in the on position,
transmiting signal even in the absence of signalling molecule.
d. The inhibition of pathways stimulated by tyrosine-kinase receptors

93. During the transduction of a signal, one molecule or ion may be closely associated with
the activity of another. Select the pair that is correctly combined
a. Calcium-IP3
b. cAMP-Adenylyl cyclase
c. cAMP protein kinase A
d. all the above

94. The hormone glucagon induces liver cells to produce more glucose by breaking down
glycogen. However, it is known that the hormone never enters the liver cells. Which
among the following best describes the mode by which glucagon exerts its action on the
liver cell?
a. The hormone is highly charged. When it binds to the plasma membrane of a liver
cell, it can exert its influence on intracellular proteins by means of long range
electrostatic interaction.
b. The hormone is degraded into small peptides by proteases on the surface of the
plasma membrane. These fragments then penetrate the cell membrane and activate
the cytoplasmic glycogen degrading enzymes.
c. The hormone binds to specific receptors on the plasma membrane which in turn
releases second messenger molecules in the cytoplasmic side, that then triggers
further downstream activity of the hormone.
d. Glycogen degrading enzymes are membrane bound, and reside on the outer
surface of the plasma membrane where they are accessible to glucagon to
penetrate the cell membrane

PART C

95. You have isolated a factor from the blood of a space alien and named it XFF (for X-
files factor). Based on preliminary data, you think that XFF could be one of three
compounds normally found in human blood: epidermal growth factor (EGF),
epinephrine, or estrogen.

It would then be useful to determine if the mechanism of action for the XFF - XFF
receptor is similar to that of estrogen or epinephrine. To test this, you use a
nonhydrolyzable analog of GTP in the reaction mixture. If the mechanism is still similar
to that of epinephrine, you would expect the cellular responses would be
a. prolonged.
b. blocked.
c. delayed.
d. both B and C
e. You cannot determine this from the given information.

96. You add radioactive XFF to cultured liver cells. Which of the following might you
observe if XFF can bind to human epinephrine receptors?
a. increased cell division
b. decreased protein synthesis
c. glucose 6-phosphate polymerization
d. glycogen breakdown
e. all of the above

97. You argue with one of your fellow students that other features of the protein suggest
that the protein is much like EGF. To discredit this, you would show the data obtained
with
a. amino acid phosphorylation.
b. tyrosine kinase inhibition studies.
c. binding assay.
d. radioactive XFF.
e. use of nonhydrolyzable GTP analogs.

98. After determining which preparation contained the XFF receptor, you wish to
determine the properties of the ligand receptor. Using a radioactive XFF, you wish to
determine if the properties are similar to that of epinephrine and the epinephrine
receptor. If so, you expect to find the radioactive XFF
a. within the cell.
b. associated with the ER.
c. on the cell surface.
d. in the nucleus.
e. associated with mitochondria.

99. Growth hormone releasing hormone produced by hypothalamus binds to its pituitary
receptor and leads to the production of growth hormone because of increase in cAMP.
Certain pituitary tumors result in hypersecretion of GH because of a mutation that
produces a Gs- protein with a greatly diminished GTPase activity
1) Elements leading to include cyclic AMP in response to GHRH binding to its
receptor include
a. Activation of monomeric G protein
b. Activation of adenylate cyclase by s subunit of a Gs protein
c. Activation of cyclic nucleotide phosphodiesterase
d. Activation of protein kinase A
e. All of the above
2) Low GTPase activity in mutated protein results in constitutive activation of
Gs and adenylate cyclase because
a. GTP bound subunit does not reform the trimer
b. GTP bound G protein binds more strongly to membrane receptor
c. GTP reacts directly with adenylate cyclase to activate it
d. Trimeric form of G protein is stabilized
e. Adenylate cyclase is phosphorylated more readily

100.Endothelial cells that line the internal surfaces of all blood vessels are source of NO
which activates guanylate cyclase. cGMP is a second messenger leading to
vasodilation of blood vessels and increased blood flow to nearby tissues. Therapeutic
drugs to increase blood flow to particular tissues can either increase NO production
or inhibit vascular isoforms of phosphodiesterase
1) Guanylate cyclase that responds to NO
a. Is in the catalytic domain of a membrane receptor
b. Ia also activated when atrial natriuretic factor (ANF) binds to its
receptor
c. Increase activity because of a conformational change when NO binds
to its heme
d. Is a monomeric enzyme
e. Is found only in smooth muscle cells
2) Viagra used to treat erectile dysfunction inhibits, selectively, the PDE
isoforms highly expressed in vascular smooth muscle. Viagra
a. Increases production of NO
b. Activates guanylase cyclase
c. Increases the rate of synthesis of cyclic GMP
d. Has the same effect in all tissues using cGMP
e. Inhibits the conversion of cGMP to inactive GMP

101.Initiation of mitogenesis by epidermal growth factor and depolarization of the


membrane of a skeletal muscle cell by acetylcholine are similar in that each
a. involves, as an essential early step, an ion flux across the plasma-membrane
receptor of the responding cell
b. requires a ligand-mediated conformational change in a plasma-membrane
receptor of the responding cell
c. requires activation of a G protein on the cytoplasmic face of the plasma
membrane in the responding cell
d. is mediated by phosphorylation of the ligand receptor in the responding
cell
e. completes its primary task by direct activation of specific regulatory DNA
sequences in the nucleus of the responding cell

102.cAMP signaling plays a very important role in the development and differentiation of
Dictyostelium discoideum. This morphogen is synthesized by different adenylyl
cyclases expressed at different stages of its life cycle. The following statements (A-D)
refer to the effect of mutations in different adenylcyclase genes:
A. aca deficient cells can be allowed to aggregate by exposing them to pulses of
cAMP.
B. Acb deficient cells would form normal fruiting bodies and the spores can
germinate when exposed to favourable conditions.
C. Acg deficient cells develop normally and the spores germinate in the spore
head itself.
D. Spores formed from the acg deficient cells will germinate irrespective of the
osmotic conditions.
Which of the above statements are correct?
a) A and D
b) A only
c) A and B
d) C and D

103.In an experiment

i. Electrical stimulation of an area in the brain (A) increased a function (F) which
was prevented by systemic injection of adrenergic antagonistic, prazosin.
ii. Injection of carbachol (cholinergic agonist) into A also increased function F
which was, however, not prevented by systemic injection of adrenergic
antagonistic, prazosin.

The results are likely to be due to the stimulation of


a) Nonadrenergic and cholinoceptive neurons
b) Cholinergic and non-adrenoceptive neurons
c) Adrenergic terminals in A
d) Both neurons and fibres passing through A

104.Genetic engineers can introduce various mammalian genes into the fly embryos. In
studies designed to investigate the development of R7 cells in fly eyes, they have
introduced various genes into fly embryios that were homozygous for the sevenless
mutation. What effect would expression of a mammalian gene have on development
of R7 cells if the introduced gene encode a ras protein with enhanced GTPase activity
a. R7 cells would develop in sev- embryos expressing normal mammalian ras
protein
b. R7 cells would not develop in sev - embryos expressing modified mammalian
ras protein
c. R7 cells would not develop in sev - embryos expressing normal mammalian
ras protein
d. None of the above
105.Some receptors for growth factors activate a protein kinase cascade, with the
participation of multiple enzymes to effect a change in gene expression. Which of the
following statements about a protein kinase cascade are true?
a. Multiple steps allow the amplification of the signal.
b. External signals can lead to changes in gene expression.
c. Multiple steps leading to kinase activations can result in cells having different
responses, depending on the presence or absence of target proteins.
d. Multiple steps in an activation mean that abnormal stimulation of a cell
response such as growth can occur with mutations in more than one gene.
e. All of the above are true.

106.The specificity of the steroid-receptor interaction is exploited in the use of the drug
tamoxifen. In some types of breast cancer, division of the cancerous cells depends on
the continued presence of the hormone estrogen. Tamoxifen competes with estrogen
for binding to the estrogen receptor, but the tamoxifen-receptor complex has little or
no effect on gene expression; tamoxifen is an antagonistof estrogen. tamoxifen
administered
a. slows or stops the growth of remaining cancerous cells
b. increases the growth of cancerous cells
c. unregulated growth
d. no effect on the cancerous cells but effects normal cells

107. Mutations in the R or C subunit of cAMP-dependent protein kinase (PKA) might lead
to
a. mutation in the R subunit makes it unable to bind to the C subunit, the C
subunit is never inhibited; it is constantly active.
b. mutation prevents the binding of cAMP to the R sub unit but still allows
normal R-C interaction, the inhibition of C by R cannot be relieved by elevated
cAMP so constantly inactive.
c. Both a and b
d. None

108.Two protein kinases, K1 and K2 function sequentially in regulating intracellular


pathway in response to extracellular signal. The following observations are made:
(i) Response is observed even in the absence of extracellular signal when a
mutation permanently activates K1.
(ii) Response is observed even in the absence of extracellular signal when K1
contains an activating mutation and K2 with inactivating mutation.
(iii) No response in the cells is detected even in the presence of extracellular
signal when both kinases are inactivated by mutation.
Which one of the following is correct?
a. K1 activates K2
b. K2 activates K1
c. K1 inhibits K2
d. K2 inhibits K1

109. In the yeast signal-transduction pathway, after both types of mating cells have
released the mating factors and the factors have bound to specific receptors on the
correct cells,
a. the cell membranes fall apart, releasing the mating factors that lead to
new yeast cells.
b. the cells then produce the a-factor and the alpha-factor.
c. one cell nucleus binds the mating factors and produces a new nucleus in
the opposite cell.

d. binding induces changes in the cells that leads to cell fusion.


Answer key Cell Signaling and Cell communication

1.B
2.C
3.C
4.C
5.E
6.A
7.A
8.A
9.D
10.A
11.D
12.D
13.A
14.A
15.D
16.A
17.B
18.B
19.D
20.E
21.B
22.A
23.D
24.E
25.E
26.B
27.C
28.B
29.A
30.B
31.E
32.B
33.E
34.E
35.D
36.B
37.C
38.A
39.B
40.E
41.D
42.D
43.B
44.D
45.E
46.D
47.A
48.C
49.D
50.C
51.B
52.B
53.E
54.D
55.D
56.A
57.C
58.E
59.A
60.A
61.A
62.D
63.B
64.B
65.D
66.E
67.C
68.C
69.D
70.C
71.E
72.B
73.A
74.D
75.A
76.B
77.C
78.B
79.A
80.A
81.B
82.B
83.D
84.B
85.B
86.B
87.B
88.D
89.D
90.D
91.B
92.C
93.C
94.C
95.A
96.D
97.B
98.C
99.(1) B, (2) A
100.(1)A, (2) E
101.D
102.A
103.B
104.B
105.E
106.A
107.C
108.A
109.d

Part B questions on cancer.

1. Taxol, an anti cancer drug is obtained from the plant taxus bocata which belongs to
a. Bryophytes
b. Pteridophytes
c. Gymnosperms
d. Angiosperms.

2. Cancerous cells differ from the normal cells in the amount of


a. Genetic damage
b. Organic molecules
c. Cytoplasm
d. Proteases

3. Which of the following is a characteristic of the precancer cells


a. irregular cell shape
b. large nuclei
c. increased amount of DNA
d. All of the above

4. The products of exactly which genes assess cell damage and coordinate the repair process?
a. Tumour suppressor genes
b. Proto-oncogenes
c. Activating genes
d. Oncogenes.

5. What is the first thing that happens when a healthy cell detects damage to DNA?
a. It stops the cell cycle.
b. It becomes cancerous.
c. It divides quickly.
d. it dies.
6. What is the normal role of telomerase?
a. builds new DNA in cancerous cells
b. builds telomeres in embryonic cells
c. repairs broken strands of chromosomes
d. builds and repairs proto-oncogenes

7. Which cells normally kill defective or mutant cells?


a. cytotoxic T and natural killer cells
b. no normal cells can kill a mutant
c. antibody-producing cells
d. chemokiller cells

8. What is the most common cancer among men?


a. Colon
b. Leukemia
c. Rectal
d. Prostate

9. What is cancer of the bone marrow stem cells that produce white blood cells called?
a. Sarcoma
b. Lymphoma
c. Leukemia
d. carcinoma

10. Which of the following statements is NOT true regarding gene therapy use for cancer
treatment?
a. It can insert a healthy p53 gene into a mutant cell.
b. It is being used in clinical trials on humans.
c. It is approved by the FDA.
d. It uses viruses to insert genes into normal and cancerous cells.

11. Cancer causing agents are called:

a. Carcinogens

b. Mutagens

c. Teratogens

d. Tumorgens

12. Most human cancers are caused by:


a. Cancer viruses
b. Chromosomal arrangements
c. Inherited disorders
d. Environmental factors

13. Cancer cells:

a. Divide uncontrollably and then die

b. Are particularly sensitive to extracellular messages

c. Divide uncontrollably and are immortal

d. Are impossible to grow in culture


14. In normal differentiated somatic cells, telomerase:

a. Actively adds material to the ends of chromosomes with each cell division

b. Is not expressed and telomere tips erode with each division

c. Removes telomere tips with each division

d. Is overexpressed and cells undergo apoptosis

15. Cancer cells are not:

a. Contact inhibited

b. Transplantable

c. Invasive

d. De-differentiated

16. A cancer cell is said to be _______ if it is shown that the disease will spread when injected into
a healthy, susceptible animal.

a. Contact inhibited

b. Transplantable

c. Benign

d. Invasive

17. A cancer that spreads is termed:

a. Benign

b. Carcinogenic

c. Metastatis
d. Mutagenic

18. proto-oncogene can become an oncogene when:

a. It is shut off

b. It is translocated next to a highly expressed gene

c. Growth factors decrease cell division rate

d. A person is exposed to pesticides

19. A(n) _______ is a type of cancer-causing gene that promotes cancer by activating cell division
at an inappropriate time or place.
a. Mutated DNA repair gene
b. Tumor suppressor gene
c. Oncogene
d. Proto-oncogene
20. The oncogene that causes Burkitt's lymphoma results from:

a. A translocation that moves a proto-oncogene next to an antibody gene

b. An inversion that places a proto-oncogene next to a transcription factor gene

c. A point mutation in a proto-oncogene

d. A virus that inserts next to a proto-oncogene

21. The Philadelphia translocation involves:

a. An exchange between chromosomes 9 and 22

b. An exchange between chromosomes 8 to 14

c. Translocation between chromosome 15 and 17

d. A fusion between chromosomes 14 and 21

22. Chronic myeloid leukemia is caused by a translocation that creates:

a. A proto-oncogene

b. A fusion protein that acts like a transcription factor

c. A protein that increases growth factor production

d. A fusion protein that deregulates the cell cycle of myeloid white blood cells

e. A deletion of a whole chromosome

23. Genes that normally prevent cell division are:

a. Tumor suppressors

b. Transcription factors

c. Proto-oncogenes
d. Growth factors

24. Loss of tumor suppression in a cell usually results from:

a. Cytokine activation of a tumor suppressor gene

b. A translocation of a tumor suppressor gene

c. An inversion involving a tumor suppressor gene


d. A deletion of a tumor suppressor gene
e. Activation of a proto-oncogene by a virus

25. The childhood kidney cancer Wilms' tumor is caused by:

a. Activation of an oncogene

b. Translocation of an oncogene

c. Loss of a tumor suppressor gene

d. A transposon

26. Formation of a retinoblastoma eye tumor requires:

a. A germinal mutation in one RB allele

b. A somatic mutation that turns on the RB gene

c. A germinal mutation in one RB allele, then a somatic mutation in the other allele

d. A germinal mutation in the dominant RB allele

27. A cytogenetic diagnosis of chronic myeloid leukemia is made by identification of:

a. Barr bodies

b. Viral infection

c. Promyelocytes

d. The Philadelphia chromosome

28. With a germ-line mutation:

a. An oncogene is activated in some cells and a tumor suppressor deleted in others


b. Only some cells are affected

c. Two somatic mutations occur

d. All cells are affected

29. The main gatekeeper for FAP colon cancer is most likely:

a. APC

b. TGF

c. P53

d. PRL-3

30. Which of the following is a tumor suppressor gene instead of an oncogene?

a. Ras

b. Myc

c. P53

d. Abl

31. Which statement is true?

a. All people who inherit proto-oncogenes develop cancer

b. All people who inherit the p53 gene develop cancer

c. Most cancers are caused by a series of genetic changes

d. Oncogenes and tumor suppressors act by the same mechanism

32. Growth of new blood vessels in and around tumors is called:


a. Invasiveness

b. Angiogenesis

c. Metastasis

d. Dedifferentiatio

33. BRCA1 and BRCA2 mutations:

a. Are X-linked

b. Are incompletely penetrant

c. Result from translocations

d. Occur in malignant breast tumors

34. Which type of study would compare the incidence of colon cancer among Japanese and
Americans of Japanese descent?

a. Clinical

b. Population

c. Prospective

d. Case-control

35. Which of these are thought to have anti-cancer benefits?

a. Heterocyclic aromatic amines (HAs)

b. Cruciferous vegetables such as broccoli


c. Red meats

d. Baked potatoes

36. The BRAC1 gene is involved in regulating:

a. Cell division

b. Cell death

c. DNA repair

d. DNA replication

37. Although BRAC2 was initially found associated with breast cancer, it also increases the risk
for cancers in the _______.

a. Colon

b. Pancreas

c. Stomach

d. All of these

38. DNA microarrays and human genome data are being used to diagnose and manage cancer.

a. True

b. False

39. Hippocrates is credited with first using the term cancer.

a. True
b. False

40. Which of the following are known to cause cancer?

a. Viruses

b. Radiation

c. Chemicals

d. All of these

41. A benign tumor is one that invades locally and metastasizes to distant sites.

a. True

b. False

42. Most carcinogens are also mutagens.

a. True

b. False

43. Sporadic cancers result from

a. Somatic mutation

b. Germline mutation

c. Inherited mutation

d. All of these
44. Cancer cells probably arise frequently but are detected and eliminated by our immune
system.

a. True

b. False

45. _______ cells are commonly used today in research laboratories. They are from a cervical
cancer patient who died in 1951.

a. HeLa

b. PSA

c. AML

d. CML

46. Cancer cells are considered to be

a. Transplantable

b. Heritable

c. Dedifferentiated

d. Invasive

e. All of these

47. Researchers have identified the biochemical reversine that stimulates cancer stem cells to
differentiate.

a. True

b. False
48. All of the following are correct except:

a. Gatekeeper genes regulate mitosis and apoptosis

b. Caretaker genes control mutation rates of gatekeeper genes

c. Proto-oncogenes normally regulate the cell cycle

d. Oncogenes are often overexpressed in cancer cells

e. Tumor suppressor genes are often overexpressed leading to cancer

49. Which of the following are thought to lower the risk of developing cancer?
a. Avoiding cigarette smoke
b. Avoiding excess exposure to the sun
c. Eating less meat and more whole grains and vegetables
d. Eating cruciferous vegetables
e. All of these

50. The term cancer means


a. cell division.
b. out of control.
c. crab.
d. lobster.

51. Cancer is often the result of activation of ____ to ____ and the inactivation of ____ genes.
a. oncogenes, tumor-suppressor genes, proto-oncogenes
b. proto-oncogenes, oncogenes, tumor-suppressor genes
c. oncogenes, proto-oncogenes, tumor-suppressor genes
d. proto-suppressor genes, suppressors, oncogenes

52. About 50% of all human cancers may involve an abnormal or missing
a. oncogene.
b. proto-oncogene.
c. p53 gene.
d. BRCA-1 gene.

53. Inherited retinoblastoma requires ____ mutation(s) or deletion(s).


a. one
b. two
c. three
d. four

54. In which of the human cells listed below is telomerase active?


a. blood
b. bone
c. muscle
d. sperm

55. Which of the following is not a characteristic of cancer cells?


a. loss of cell cycle control
b. transplantability
c. loss of contact inhibition
d. all are characteristic

56. The Philadelphia chromosome is associated with which type of cancer?


a. breast
b. thyroid
c. nerve
d. leukemia

57. BRCA-1 is associated with which cancer?


a. breast
b. thyroid
c. nerve
d. leukemia

58. If 85% of lung cancer cases occur in heavy smokers, can lung cancer still have a genetic
origin?
a. yes
b. no

59. An increasing number of women in the rural south die from ____ cancer.
a. breast
b. colon
c. lung
d. mouth

60. Which of the following statements about telomerase is incorrect?


a. It is an enzyme that adds DNA to telomeres.
b. It serves as the template for telomeres lengthening.
c. It is not activated in cancer cells.
d. Its activity continually resets the cellular clock.

61. Familial cancer is caused by


a. a mutation in somatic cells only.
b. a mutation in germline cells only.
c. a germline mutation plus a somatic mutation in affected tissue.
d. two germline mutations.

62. A cancerous tumor eventually grows faster than surrounding tissues because a greater
proportion of its cells are dividing. True or false?
a. true
b. false
63. Which type of cancer in humans is directly caused by a viral infection?
a. acute T cell leukemia
b. Wilms' tumor
c. Burkitt's lymphoma
d. Rous sarcoma

64. An oncogene transcribed and translated with another gene produces a


a. transcribed protein.
b. fusion protein.
c. fusion cell.
d. cancer protein.

65. The P53 protein normally promotes


a. DNA repair.
b. tumor formation.
c. cell division.
d. apoptosis.

66. The P53 gene is especially prone to


a. point mutation.
b. chromosomal rearrangement.
c. loss.
d. none of the above.

67. FAP colon cancer results from ____ mutation(s).


a. one
b. two
c. three
d. four or more

68. Which type of study compares the incidence of a type of cancer among very different groups
of people?
a. population
b. case-control
c. prospective
d. empiric

69. Which type of study would compare cancer rates seen in a group of individuals who take
herbal supplements versus those in a control group who do not use the supplement?
a. population
b. case-control
c. prospective
d. empiric
70. Which of the following cancers develops from loss of tumor suppression?
a. cute T cell leukemia
b. Wilms' tumor
c. Burkitt's lymphoma
d. Rous sarcoma
71. Why is genetic counseling for familial breast cancer difficult?
a. BRCA1 and BRCA2 are incompletely penetrant.
b. Breast cancer can occur in other ways.
c. Not all mutations are associated with disease.
d. all of the above

72. Which of the following is not a traditional cancer treatment?


a. blocking telomerase
b. inhibiting angiogenesis
c. stimulating specialization
d. none of the above

73. A mutation in which gene makes nearby DNA more susceptible to replication errors?
a. APC
b. BRCA1
c. P53
d. RB

74. Which of the following may contribute to causing cancer?


a. a mutation in a gene that slows the cell cycle
b. faulty DNA repair
c. loss of control over telomere length
d. all of the above
e. Question 01

75. Risk factors for prostate cancer include which one of the following?
a. Castration before age 40 years
b. Black race
c. Asian race
d. Young age

76. Which of the following is true regarding prostate cancer?


a. The lifetime risk of developing it is 10%
b. It is the most common cause of cancer death in men
c. Incidence is falling
d. It accounts for approximately 30% of all cancers in men

77. In diagnosis of prostate cancer:


a. all patients should have a digital rectal examination (DRE)
b. most patients with a PSA>4ng/ml will have prostate cancer
c. urinary obstructive symptoms are highly suggestive
d. biopsy is usually by fine needle aspiration (FNA)

78. Which one of the following statements is true of prostate cancer?


a. Gleason score 3 describes a poorly differentiated tumour
b. 65% of patients with Gleason score 5-7 will die from prostate cancer within 15 years
c. PSA >50ng/ml is often associated with distant bone metastases
d. T2 describes a tumour that has extended through the capsule
79. Which of the following statements is true in management of prostate cancer which is
confined to the prostate gland?
a. Active surveillance is a valid approach
b. Prostatectomy has been shown to be superior to radiotherapy
c. Radiotherapy causes impotence in 50% of patients
d. Prostatectomy leads to long term incontinence in 5% of patients

80. In metastatic prostate cancer:


a. median survival is 18 months
b. hormone therapy will produce responses in around 70% of men with bone
metastases
c. the median response duration of hormone therapy is 8 months
d. chemotherapy is the most commonly used systemic treatment used

81. Side effects of hormone treatment rarely include which one of the following?
a. Hot flushes
b. Loss of libido and potency
c. Osteoporosis
d. Myelopsuppression

82. In treatment of hormone refractory prostate cancer:


a. docetaxel and prednisolone have been shown to increase median survival by 2
months
b. mitoxantrone and prednisolone have been shown to increase median survival by 1
month
c. withdrawing anti-androgen therapy confers no benefit
d. intravenous radio-isotope therapy has been shown to increase median survival by 3
months

83. Which one of the following statements is true of prostate bone metastases and their
complications?
a. Most fractures occur as the result of trauma
b. Spinal cord compression is usually managed surgically
c. Prophylactic orthopaedic intervention may be required for bone lesions at high risk
of fracture
d. Bisphosphonates may help to reduce bone pain but do not reduce the number of
cancer-associated skeletal events

84. Which one of the following symptoms is rare in patients with progressive prostate cancer?
a. Symptomatic anaemia
b. Urinary retention
c. Sexual dysfunction
d. Haematemesis

85. The translocation of chromosomes 9 and 22 is characteristic of:


a. Retinoblastoma
b. Li-Fraumeni syndrome
c. Chronic myelogenous leukemia
d. Soft tissue sarcoma
86. Which of the following is not a characteristic of tumor suppressor genes?
a. Transmitted as dominant traits in families.
b. May be associated with loss of heterozygosity in tumors.
c. Act dominantly in tumor cells.
d. Associated with "two-hit" model of carcinogenesis.

87. Which of the following statements about proto-oncogenes is true?


a. They are originally derived from RNA tumor viruses.
b. Loss of function mutations tend to occur in tumor cells.
c. Mutation underlies most cases of familial predisposition to cancer.
d. Some are activated by gene amplification.

88. Which of the following types of genetic change is least likely to be found in an oncogene in a
tumor?
a. Gene amplification
b. Chromosome translocation
c. Missense mutation
d. Nonsense mutation

89. Double minute chromatin bodies are indicative of which of the following?

a. Translocation
b. Inactivation of a tumor suppressor gene
c. Apoptosis
d. Gene amplification

90. Genes responsible for hereditary nonpolyposis colon cancer encode proteins with which of
the following function?
a. Cell membrane receptor
b. Transcription factor
c. DNA mismatch repair
d. Cell cycle control

91. The 8;14 translocation in Burkitt lymphoma activates an oncogene that encodes a protein
with which of the following function?
a. Cell membrane receptor
b. Transcription factor
c. DNA mismatch repair
d. Cell cycle control

92. Microsatellite instability is associated with:


a. Hereditary nonpolyposis colon cancer
b. Familial adenomatous polyposis
c. Neurofibromatosis
d. Hereditary breast cancer

93. Response of non-small cell lung cancer to gefitinib is predicted by mutation in:
a. EGF receptor gene
b. Abl oncogene
c. Myc oncogene
d. NF1 gene

94. Which of the following would be an effective approach to a new cancer therapy?
a. finding a way to stabilize p53 specifically in tumor cells
b. preventing nucleotide synthesis in tumor cells
c. inactivating the HER2 receptor on tumor cells
d. inhibiting growth of new blood vessels with endostatin
e. All of the above would help to fight cancer.

95. How would the cell cycle be affected if you removed the phosphorylation sites in the Rb
protein?
a. The cell cycle would not be affected because pRb is not phosphorylated normally.
b. The cell cycle would be blocked in G1.
c. The cell cycle would be blocked in G2.
d. The cell cycle would be shorter.

96. Embryonic stem (ES) cells are an attractive source of material for therapeutic cloning
because
a. they can be induced to assume any cell fate.
b. ES cells are not targets for the host immune response, so tissue rejection is not an
issue.
c. there are no other sources of stem cells to use for therapeutic cloning, so ES cells are
the only solution.
ES cells will not work as a source of tissue for cloning.

97. How would growing cells in the presence of methyladenosine affect the mismatch repair
system?
a. The repair system would only repair half of the errors introduced by DNA
polymerase.
b. There would be no repair of mismatched DNA.
c. Mismatch repair would be normal, but excision repair would fail.
d. Methyladenosine would prevent DNA replication, so there would be no need for
mismatch repair.

98. Too much time in a tanning booth probably causes DNA damage to epithelial cells. The most
likely effect would be
a. depurination.
b. pyrimidine dimers.
c. deamination.
d. single-stranded nicks in the phosphodiester backbone.

99. Mutations that impact evolution most occur in


a. somatic cells
b. brain cells
c. germ-line cells
d. sperm cells

100. Ultraviolet light can cause the formation of double bonds between adjacent pyrimidines,
resulting in a formation called a
a. pyrimidine mispairing
b. pyrimidine-pyrimidine pairing
c. pyrimidine dimer
d. pyrimidine pseudopairing

101. Chemical modification of DNA may be caused by


a. chemicals that resemble DNA nucleotides
b. chemicals that remove amino acids from adenine or cytosine
c. chemicals that add hydrocarbon groups to nucleotide bases
d. all of the above

102. Chromosomal rearrangement may result in


a. monoploidy
b. aneuploidy
c. diploidy
d. statiploidy

103. Carcinomas are tumors arising from


a. epithelial tissue
b. bone
c. muscle
d. connective tissue

104. Pseudogenes are important to evolution because


a. they are silent genes that have been activated by mutations
b. they did not exist until recently
c. they occur entirely within multigene families
d. mutational changes may convert them to active genes, encoding a protein with
different properties
e. they tend to be transcribed more frequently

105. The ras-induced bladder cancer is caused by


a. inducing the transcription of p21 proteins
b. inducing p53 mutations
c. a single DNA base change in the normal ras protein
d. preventing phosphorylation of Rb protein
e. tying up the transcription factor E2F

106. Transposition can facilitate


a. insertional activation
b. gene amplification
c. gene mobilization
d. insertional reversion
e. gene immobilization
107. Unequal crossing over tends to cause
a. an increase in the number of copies of a gene
b. insertional inactivation in the region of crossing over
c. loss of genes excised by correction mechanisms
d. conversion of the excess DNA into heterochromatin
e. polyploidy

108. The virus known as RSV is associated with


a. a chicken sarcoma
b. a bovine cancer
c. breast cancer
d. liver sarcomas
e. human carcinoma

109. Double-strand breaks in DNA are lethal to the descendants of


a. bacteria
b. fungi
c. insects
d. algae
e. reptiles

110. Crossing over, as a mode of genetic recombination, is important to evolution because it


a. tends to purify traits
b. brings about segment segregation
c. is normal and doesn't cause cancer
d. can bring together mutations occurring on different chromosomes
e. tends to eliminate mutations

111. Next to breast cancer, the most common form of cancer in the United States is
a. breast
b. leukemia
c. colon & rectum
d. bladder
e. prostate

112. Transponsons
a. are more numerous than satellite DNA, but are shorter in length
b. appear to play no functional role in a cell
c. are more diverse in mammalian genomes than in other eukaryotes
d. account for at least 20% of Drosophila DNA
e. include the many ULA elements of the human genome

113. Cells that leave a tumor and spread throughout the body, forming tumors at distant sites are
called
a. sarcomas
b. lymphomas
c. carcinomas
d. metamorphoses
e. metastases
114. Cancer is more common in older people because
a. their immune systems have degenerated
b. the supply of certain hormones declines with age
c. a change in the rate of cell replacement takes place
d. they have accumulated more mutations
e. their bodies are unable to adjust to the changing environment

115. Tandem clusters occur most commonly in


a. satellite DNA
b. centromere
c. the nuclear organizer regions
d. plasmids
e. the area of the synaptonemal complex

116. The price of smoking a pack of cigarettes is _______________ hours of one's life.
a. 24
b. 16
c. 12
d. 7
e. 3.5

117. Slipped mispairing may cause deletions resulting in


a. single nucleotide substitution
b. translocations
c. frameshift mutation
d. insertion inactivation
e. errors in nucleotide choice

118. Plasmids are transferred between bacteria by way of a


a. conjugator
b. pilus
c. transposer
d. snorkel
e. trichogyne

119. The primary effect of ultraviolet radiation is the production of


a. two-stranded breaks in DNA
b. pyrimidine dimers
c. base analogue mispairing
d. isomerization of a base
e. slipped mispairing

120. Gene conversion refers to


a. a DNA segment being reversed in a chromosome
b. insertion of a transposon that alters the reading frame
c. the formation of plasmids from a bacterial genome
d. single nucleotide substitution caused by a base analogue mispairing
e. alterations in homologues as mismatch pair errors are corrected

121. Inversions in a chromosome


a. are a form of transposition
b. cause aneuploidy
c. are important in eukaryotes if in somatic cells
d. have an effect on recombination
e. alter gene expression

122. Which of the following is not a form of genetic recombination


a. plasmid formation
b. chromosome assortment
c. reciprocal recombination
d. gene transfer
e. none of the above

123. Tumor-suppressor genes


a. are involved in the cellular response to EGF
b. encode proteins that prevent binding of cyclins
c. stimulates the binding of GTP
d. includes the widely studied myc gene
e. are genetically recessive

124. Mutations always cause detrimental effects.


a. True
b. False

125. A sexual-like process in some bacteria that results in exchange of genetic material between
two cells that are temporarily joined, best defines:
a. conjugation.
b. complementation.
c. lysogeny.
d. transformation.
e. transduction.

126. F+ bacteria can construct which of the following that allow the bacteria to join together to
transfer genes?
a. gap junctions
b. pili
c. connecting channels
d. porins
e. plasmodesmata

127. Tumor promoting genes are called proto-oncogenes.


a. True
b. False

128. Changes that alter the sequence of one or a few DNA nucleotides are called point mutations.
a. True
b. False
129. Genes that can move about from one location on a chromosome to another are called
transposons.
a. True
b. False

130. The p53 gene is responsible for


a. triggering cells to grow uncontrollably
b. damaging DNA
c. over riding the G1 checkpoint
d. initiating transcription of p21 which binds to cyclins
e. degrading damaged DNA

131. Incidence of cancer increases with age because several mutations must accumulate to
trigger cancerous growths.
a. True
b. False

132. Potential cancer therapies are being developed to target all of the following except:
a. inhibiting the release of a signal (i.e. growth factor) from neighboring cells
b. using monoclonal antibodies to "tag" cancerous cells
c. inactivating Ras proteins in the cell
d. using anti-sense RNA to inhibit Src
e. using viruses that don't elicit an immune response to attack and kill cancer cells

133. Which definition of cancer is the most accurate?


a. A group of diseases characterized by uncontrolled cell division.
b. A disorder in which cells produce a localized tumor.
c. A disorder in which tissues of the body are replaced by nonfunctional cells.
d. All of these.
e. None of these.

134. Genes which promote cancer are called:


a. Tumor suppressors.
b. Oncogenes.
c. Growth factors.
d. Malignancy enhancers.

135. The first oncogene to be identified:


a. Was c-src.
b. Causes sarcomas in chickens.
c. Is normally expressed at high levels in eukaryotic cells.
d. All of these.
e. None of these.

136. Rous sarcoma virus (RSV) causes cancer by integrating into genes that normally control cell
division and eliminating their function.
a. True
b. False
137. When a cell is stimulated by a growth hormone:
a. The presence of the hormone directly activates gene expression.
b. A cascade of protein interactions leads to changes in gene expression.
c. Cell surface receptors are deactivated.
d. All of these.
e. None of these.

138. The role of a tumor suppressor gene:


a. Is to encode a protein that stimulates cell division.
b. Is to prevent mistakes in DNA replication.
c. Is to allow cell division only under appropriate conditions.
d. All of these.
e. None of these.

139. Which of the following is a common type of tumor suppressor genes?


a. Genes that increase the rate of the cell cycle or stimulate cell division.
b. Genes involved in performing DNA synthesis.
c. Genes involved in maintenance of cell cycle checkpoints.
d. All of these.
e. None of these.

140. Why are cancer cells likely to accumulate mutations, including changes in chromosome
number, over time?
a. They are less likely to attach chromosomes correctly to the mitotic spindle.
b. They are more likely to make mistakes in DNA synthesis.
c. They are allowed to divide despite potential errors in DNA synthesis or spindle
assembly.
d. All of these.
e. None of these.

141. chemical or physical factor that causes cancer is called a(n) _____.
a. virus
b. carcinogen
c. oncogene
d. mutagen

142. Malignant tumors:


a. are surrounded by connective tissue
b. can travel and begin growing in distant body locations
c. can be easily removed surgically
d. remain in one place as a well-defined mass of cells.

143. Benign tumors:


a. remain in one place as a well-defined mass of cells
b. have abnormalities in DNA but not in cell division
c. metastasize, but not as extensively as cancer cells
d. are always pre-cancerous
144. Which of the following indicates the progressive stages of cancer development in the correct
order?
a. in situ cancer ( dysplasia ( malignant tumor ( metastasis
b. in situ cancer ( hyperplasia ( metastasis ( malignant tumor
c. hyperplasia ( dysplasia ( in situ cancer ( malignant tumor
d. dysplasia ( hyperplasia ( metastasis ( malignant tumor

145. The leading cause of cervical cancer is:


a. Human papillomavirus (HPV)
b. radiation exposure
c. exposure to carcinogens in the diet
d. HIV

146. Metastasis is the process of:


a. destroying cancer cells with radiation
b. cancer cells breaking free of the main tumor and spreading to other locations
c. changes accumulating in the structure of a cancer cell
d. naturally limiting cancer spread with the body's immune system

147. When a normal regulatory gene that promotes cell growth is mutated or damaged, it may
become a/an:
a. Oncogene
b. proto-oncogene
c. growth factor
d. tumor suppressor gene

148. One cancer treatment involves "starving" the cancer cells by inhibiting:
a. digestion
b. immune function
c. metastasis
d. angiogenesis

149. The immune system can help to defend against cancer cells because:
a. cancer is able to hide inside body organs
b. cancer cells metastasize
c. cancer antigens attack T cells
d. cancer cells may stop displaying normal "self" proteins on the cell surface

150. The p53 gene is an example of a/an:


a. mutator gene.
b. oncogene
c. tumor suppressor gene
d. proto-oncogene

151. Which type of genes are involved in the repair of DNA damage?
a. mutator genes
b. tumor suppressor genes
c. proto-oncogenes
d. oncogenes
152. Which of the following is NOT a classic treatment for cancer?
a. surgery
b. physical therapy
c. chemotherapy
d. radiation

153. Which of the following is NOT a risk factor for breast cancer?
a. taking estrogen supplements after menopause
b. smoking
c. age
d. early menstruation

154. Which of the following is the most common cancer in the United States?
a. breast cancer.
b. lung cancer
c. skin cancer
d. colon cancer

155. You can inherit a susceptibility to cancer because:


a. cancer begins as a weakness in the tissues of an organ
b. cancer involves abnormal genes, some of which may be inherited
c. cancer genes develop primarily during meiotic cell divisions
d. cancer abnormalities may be passed in the cytoplasm of an oocyte.

156. Oncogenes __________.


a. are both derived from proto-oncogenes and induce uncontrolled cell proliferation
b. are tumor suppressor genes
c. are derived from proto-oncogenes
d. induce uncontrolled cell proliferation
e. are viral genes

157. The function of the ras gene family involves __________.


a. signal transduction
b. tumor suppression
c. chromosome segregation
d. cell cycle control
e. chromatin condensation

158. Cancer is best described as __________.


a. a genetic disorder at the cellular level
b. a genetic disease at the somatic cell level
c. a genetic disease at the gametic cell level
d. an environmental disease at the gametic cell level
e. a bacterial disease at the somatic cell level

159. Apoptosis represents genetically programmed __________.


a. determination
b. tumor suppression
c. cell death
d. proto-oncogene conversion
e. differentiation

160. Ionizing radiation, viruses, and chemicals are all examples of __________.
a. carcinogens
b. precursors to oncogenes
c. tumor suppressors
d. environmental hazards

161. __________ and __________ are two classes of proteins known to regulate the cell cycle
checkpoints.
a. Protein kinases/cyclins
b. Cyclin-dependent kinases/cyclins
c. p53/polymerases
d. Ligase/polymerases
e. Protein kinases/ATP synthase

162. Why is familial retinoblastoma known to be inherited as an autosomal dominant trait when
the responsible gene is recessive?
a. This recessive gene needs only one copy to express this phenotype.
b. People with this mutation are always homozygous for this allele.
c. If you are born with one allele mutated in every cell, the chance that the other allele will
be mutated in one of the retinal cells is extremely high.
d. Retinoblastoma was mischaracterized originally when it was called an autosomal
dominant trait.

163. The abnormal growth and spread of abnormal cells is known as:
a. Malignant
b. Biopsy
c. Carcinogen
d. Cancer

164. Cigarette smoke is considered a cancer-causing agent, which could also be referred to as:
a. Benign
b. Carcinogen
c. Malignant
d. Oncogenes

165. Noncancerous tumors are called:


a. Metastasis
b. Oncogenes
c. Benign
d. Malignant

166. The surgical procedure used to determine the type of tumor one has is called a:
a. Angiogenesis
b. Carcinogen
c. Metastasis
d. Biopsy

167. The spread of cancerous tissue from one location to another is called:
a. Malignant
b. Migration
c. Metastasis
d. Angiogenesis

168. The process that allows for tumors to develop a blood supply to provide the tumor with
oxygen for survival and further growth is referred to as:
a. Metastasis
b. Carcinoma
c. Angiogenesis
d. Oncogenes

169. Tumors that develop in the lymph nodes are called:


a. Leukemias
b. Sarcomas
c. Carcinomas
d. Lymphomas

170. The type of cancerous tumors that are characterized as being nonsolid is:
a. Lymphomas
b. Sarcomas
c. Leukemias
d. Carcinomas

171. Doctors that specialize in cancer are called:


a. Oncologists
b. Cardiologists
c. Neurologists
d. Obstetricians

172. The most common form of cancerous tumors are:


a. Sarcomas
b. Leukemias
c. Carcinomas
d. Lymphomas

173. Which of the following statements is true regarding cancer?


a. It only occurs in middle age to older age adults
b. It is characterized by controlled cell growth
c. Most cancers are unpreventable
d. Stage 1 tumors are localized and often curable

174. All of the following are lifestyle risk factors for cancer except:
a. Alcohol consumption
b. Overweight
c. Regular exercise
d. Tobacco use

175. Identify the correct statement regarding cancer:


a. Stress increases susceptibility to cancer.
b. Physical inactivity does not increase susceptibility to cancer.
c. Cancer death rates are higher for African Americans than Caucasians.
d. Delayed menarche increases a woman's risk for breast cancer.

176. A woman's risk of cancer development is reduced under which condition:


a. Hormone replacement therapy
b. Early menarche
c. Having children after age 30
d. Having many children

177. Genes that play a role in uncontrolled cell growth are known as:
a. Oncogenes
b. Growth genes
c. Protogenes
d. Tumor genes

178. Cervical cancer is associated with which of the following viruses?


a. PIV
b. HPV
c. HCV
d. HSV

179. The most serious form of skin cancer is:


a. Basal
b. Malignant
c. Squamous
d. Benign

180. In the "ABCD" guidelines to melanoma the "D" stands for:


a. Diameter
b. Depth
c. Dimension
d. Darkness

181. At what age is it recommended that women start getting mammograms?


a. 30
b. 25
c. 40
d. 35

182. Colorectal cancer risk can potentially be reduced by:


a. Limiting fruit intake
b. Limiting physical activity
c. Eating a high fat diet
d. Eating a high fiber diet

183. The early warning signs of cancer are summarized by the acronym:
a. CAUTION
b. CAUSE
c. COVER
d. CHECK

184. All of the following are risk factors for pancreatic cancer except:
a. Smoking
b. Having diabetes
c. High fat diet
d. Being female

185. Hair loss is common with this form of cancer treatment:


a. Gene therapy
b. Radiation
c. Chemotherapy
d. Immunotherapy

186. Cancer is the ______ leading cause of death in the U.S.


a. 2nd
b. 4th
c. 3rd
d. 5th

187. A Pap test is effective for detecting ________ cancer.


1. Breast
2. Skin
3. Cervical
4. Lung

188. Which cellular component is most likely coded for by proto-oncogenes?


a. signal transduction proteins
b. histone proteins
c. mitochondrial proteins
d. ribosomal proteins
e. nuclear membrane proteins

189. p53 can be best classified as


a. a tummor suppressor gene
b. an oncogene

190. A study of malignant neoplasms reveals that some of them have a greater potential for
invasion and metastases. Analysis of the cells of cancers that have metastasized reveals a
mutation which results in decreased cell surface expression of E-cadherin. Which of the
following genes is most likely mutated to produce these findings?
a. Beta-catenin
b. Ras
c. BRCA-2
d. Rb
e. Cyclin D

191. 5'-ACG_____-3' is half of a palindromic restriction site. What is the complete sequence?
a. 5'-ACGCGT-3'
b. 5'-ACGACG-3'
c. 5'-ACGTGC-3'
d. 5'-ACGGCA-3'

192. Identify the appropriate chromosomal transloction that is most frequently found with
Chronic myelogenous leukemia
a. t(8;14)(q24;q32)
b. t(14;18) (q32;q21)
c. t(11;14)(q13;q32)
d. t(9;22) (q34;q11)

193. A 28-year-old woman goes to her physician for a routine examination. She is sexually active.
Pelvic examination reveals no abnormalities. A Pap smear is obtained. The cytopathology
report indicates the presence of severely dysplastic cells. A biopsy of the cervix is
performed, and on microscopic examination shows cervical intraepithelial neoplasia III (CIN
III). Infection with which of the following organisms is most likely to cause her disease?
a. Epstein-Barr virus
b. Herpes simplex virus infection
c. Human papillomavirus

194. The sputum (fluid coughed up from the lungs) of many smokers contain cells with
mutations (errors) in the genes for p53. The smoking induced mutations appear to be an
early signal showing that cancer of the lungs will follow. What is the likely relationship
between early p53 mutation and the development of lung cancer?
a. Mutations in p53 would prevent abnormal cells from dying by apoptosis.
b. Mutant p53 triggers the M phase of the cell cycle leading to abnormal cell division
c. p53 causes a cell to enter G2, blocking cell division.
d. p53 with a mutation directly stimulates the growth of cancer cells.

195. What method is employed to verify the transcript size of a gene?


a. Allele-specific oligonucleotide (ASO) hybridization
b. Polymerase chain reaction
c. Southern blot analysis
d. Western blot analysis
e. Northern blot analysis

196. Southern Blotting is a technique that:


a. Uses antibodies to detect proteins
b. Sorts RNA molecules by size
c. Uses lectins to detect carbohydrates
d. Uses DNA probes to detect DNA fragments
197. Identify the appropriate chromosomal transloction that is most frequently found with
Follicular B-cell lymphoma.
a. t(8;14)(q24;q32)
b. t(9;22) (q34;q11)
c. t(14;18) (q32;q21)
d. t(11;14) (q13;q32)

198. A loss of heterozygosity in a cell that gives rise to a tumor


a. Refers to activation of an oncogene
b. Refers to loss of hybrid vigor
c. Refers to homozygous presence of a mutant allele in tumor cells that is heterozygous in
normal cells
d. Refers to uniparental disomy
e. Refers to the loss of mis-match repair in DNA

199. You have referred one of your patients to an oncologist because of a breast tumor.The
oncologist decides to treat the patient with a combination of 5-fluorouracil and Taxol. The
mechanism of action of the antitumor drug Taxol involves
a. Binding to intermediate filament dimers
b. Specific binding to actin filaments
c. Interference with the folic acid pathway
d. Alkylation of DNA
e. Stabilization of microtubules

200. A husband (H2) knows that his wife (W2) had an affair with her best friend's husband (H1).
Husband (H2) fears that he might not be the biological father of their 7-month-old child
(C2). Restriction fragment length polymorphism (RFLP) analysis is performed on DNA
samples from the two couples and from their two children (each couple has one child). The
results are shown below. Which conclusion can be drawn from these results?
a. Husband 1 is the father of child 1
b. Husband 2 is the father of child 2
c. Husband 1 is the father of child 2
d. Husband 1 is the father of both children
e. Husband 2 is father of child 1

201. Many different classes of drugs are used as anti-tumor agents. The rationale for each
different drug takes advantage of a particular property of tumor cells compared to normal
cells. Recently, inhibition of angiogenesis has become an important potential avenue of
cancer chemotherapy. Inhibition of angiogenesis could result in inhibiting tumor growth by
blocking
a. metastasis directly
b. new blood vessel formation
c. telomerase activity
d. tumor cell proliferation directly
e. stimulation of the immune system

202. A polymerase chain reaction can increase the sensitivity of certain genetic tests. Which of
the following characteristics applies to polymerase chain reaction?
a. Multiple cycles lead to the rapid linear amplification of the DNA
b. Addition of a heat-resistant restriction endonuclease is required
c. The DNA to be amplified is denatured in the presence of an equimolar ratio of primers

203. The sequence of the segment of DNA to which the primers will bind must be known Identify
the gatekeeper gene:
a. Rb
b. PMS2
c. MLH1
d. MSH2

204. A woman was diagnosed with breast cancer at the age of 40. Her father had breast cancer
and herpaternal grandmother had ovarian cancer. Which gene is most likely to be mutated
in affected members of the family?
a. APC
b. BRCA2
c. PTEN
d. FAS
e. MSH2

205. A 26-year-old female is found positive for occult blood in her stool sample. A colonoscopy is
performed. It shows numerous polyps, numbering well over 200, ranging in size from 1-5
cms. One polyp is resected. Histologically, the polyp shows the features of tubular adenoma.
There is no invasion. Molecular analysis of patients fibroblasts shows mutation in APC gene.
The most likely diagnosis in this case is
a. Hereditary nonpolyposis colon cancer
b. Familial gastric cancer
c. Familial adenomatous polyposis
d. Melanoma
e. Peutz-Jeghers

206. A particular RFLP is diagrammed below. 'E' represents invariant EcoRI restriction sites. '*'
represents polymorphic EcoRI sites. The dark box represents the location of a particular
DNA probe 'A'. What are all the possible alleles (i.e. size of bands) seen on a Southern blot
probed with 'A'?
a. 1 kb, 2 kb, 3 kb, 4 kb and 6 kb
b. 3 kb, 4 kb, 6 kb
c. 2 kb, 3 kb and 6 kb
d. 1 kb, 3 kb, 4 kb and 6 kb
e. 1 kb

207. In the Sanger method of DNA sequencing, incorporation of the nucleotides stops because
a. the dideoxynucleotide analogs have no 2'-OH group to continue DNA synthesis.
b. the dideoxynucleotide analogs have no 5'-OH group to continue DNA synthesis.
c. the dideoxynucleotide analogs are in greater concentration than the actual nucleotides.
d. the mixture is heated to 92C to stop DNA synthesis at specific time intervals.
e. the dideoxynucleotide analogs have no 3'-OH group to continue DNA synthesis.

208. A collection of plasmids containing foreign DNA fragments representing the entire genome
of an organism is called a(n)
a. cDNA
b. clone
c. vector
d. library
e. transgenic

209. Spinobulbar muscular atrophy (SMA) is an X-linked recessive motor neuron disease, arising
from a mutation in the androgen receptor gene. The normal allele of the gene has 10
adjacent glutamine codons (CAG) in exon 1, whereas the SMA allele has 40 of these CAG
repeats in the same region (a mutation referred to as triplet repeat expansion). The most
appropriate DNA-based test to detect this mutation would utilize PCR primers:
a. Flanking the CAG repeat followed by a dot-blot using a CAG repeat-specific DNA probe
b. Complementary to the CAG repeat followed by a dot-blot using a CAG repeat-specific DNA
probe
c. Flanking the CAG repeat followed by a Southern blot using a CAG repeat-specific DNA probe
d. Complementary to the CAG repeat followed by a Southern blot using a CAG repeat-specific
DNA probe.

210. Identify the caretaker gene:


a. BRCA1
b. Rb NF1
c. p53
d. MSH2

211. A patient has a disorder that is inherited as a dominant trait. Penetrance is incomplete. The
doctor discovers that the disease is caused by a mutation in a tumor suppressor gene. The
gene mutated regulates G1 to S phase transition.
What disorder does this patient have?
a. Familial Breast Cancer
b. Cowden syndrome
c. Retinoblastoma
d. Neurofibromatosis Type 1
e. Li-Fraumeni Syndrome

212. A 40-year-old male has started to notice he is becoming easily fatigued and frequently feels
weak. He has experienced weight loss as well. He sees his family doctor. Extensive history,
physical, and laboratory work is done. He is found to have the Philadelphia chromosome.
What is the most likely diagnosis?
a. Chronic myelogenous leukemia
b. Follicular B-cell lymphoma
c. Cowden syndrome
d. Burkitt's lymphoma
e. Wilm's tumor

213. Which of the following sequences is recognized and cleaved by the restriction enzyme EcoR
I?
a. 5'-CACACA-3'
b. 5'-CGCGAA-3'
c. 5'-CCTTAA-3'
d. 5'-CGTGAA-3'
e. 5'-GAATTC-3'
214. A proto-oncogene may be activated by any of the following mechanisms EXCEPT
a. point mutation
b. chromosome rearrangement
c. placing it under the control of another promoter
d. deletion
e. amplification

215. Marked microsatellite instability is a feature of:


a. Familial adenomatous polyposis.
b. Von Hippel-Lindau
c. Hereditary non-polyposis colon cancer (HNPCC).
d. Neurofibromatosis 1.
e. Multiple endocrine adenomatosis type 2.

216. Western Blotting is a technique that:


a. Sorts DNA fragments by size
b. Uses antibodies to detect proteins
c. Uses DNA probes to detect DNA fragments
d. Uses DNA probes to detect RNA fragments

217. Dideoxynucleotides, such as dideoxycytidine and dideoxyinosine, directly inhibit 30)


a. cDNA synthesis
b. mRNA synthesis
c. poliovirus genomic replication
d. tRNA synthesis
e. rabies virus genomic replication

218. A cell culture of a tumor biopsy is suspected of containing a ras oncogene. Which of the
following proteins should an analyed to confirm this diagnosis? (What type of protein is ras?)
a. Nuclear transcription factor
b. Cyclin
c. Growth factor receptor
d. Growth factor
e. G protein

219. Which primers will amplify my favorite gene if it is in the context below? 5'-GGTACAGTC-MY
FAVORITE GENE-CTAGATCAT-3'
a. 5'-GGTACAGTC-3' and 5'-CTAGATCAT-3'
b. 5'-GACTGTACC-3' and 5'-CTAGATCAT-3'
c. 5'-GGTACAGTC-3' and 5'-ATGATCTAG-3'
d. 5'-GACTGTACC-3' and 5'-ATGATCTAG-3'
e. 5'-GGTACAGTC-3' and 5'-GACTGTACC-3'

220. Tumors originating in the mesenchymal tissue, such as bone muscle or connective tissue are
referred to as:
a. Sarcomas
b. Lymphomas
c. Carcinomas
d. Leukemias
221. Which of the following vectors can accept the largest DNA fragments in cloning experiments?
a. Bacteriophage lambda
b. Cosmids
c. Plasmid
d. Phage P1
e. YACs

222. The mold Aspergillus flavus oryzae produces aflatoxin B1, one of the most potent carcinogenic
compounds known. What form of cancer is linked to dietary exposure of aflatoxin B1 from
Aspergillus flavus growing on grains and nuts in tropical climates?
a. Autoimmune lymphoproliferative syndrome
b. Burkitt lymphoma
c. Nonpolyposis colon cancer
d. Hepatocellular carcinoma
e. Wilms' tumor

223. Chromosome abnormalities that develop in association with neoplasms often involve all of
the following EXCEPT
a. oncogene activation
b. clonal evolution
c. loss of tumor suppressor genes
d. transmission to the offspring of an affected individual

224. Which of the following enzymes synthesizes DNA from a single-stranded RNA template?
a. Endonuclease
b. DNA polymerase
c. Reverse transcriptase
d. Ligase
e. RNA polymerase

225. The rate at which a DNA fragment moves in an electrophoretic gel is primarily a function of
the fragment's
a. degree of methylation
b. adenine content
c. radioactivity
d. double helical structure
e. length

226. Cancer cells undergo many phenotypic and genetic changes compared to their normal
counterparts. Immortalization of cells is specifically associated with the tumor cell phenotype. A
specific enzyme, telomerase, has been shown to be involved in this immortalization. The
enzyme telomerase has catalytic properties similar to the enzyme
a. DNA polymerase
b. Reverse transcriptase
c. Restriction endonuclease
d. Topoisomerase
e. DNA methylase

227. Breast cancer caused by mutations in the BRCA1 gene is characterized by: 40)
a. relatively early age of onset of the disease
b. increased incidence of breast cancer in males
c. increased risk of ovarian cancer
d. increased risk of small cell lung cancer
e. Both (A) and (C)

228. A renal transplantation patient returns for followup renal biopsy. The medical team wishes
to conduct gene expression analysis of the biopsy specimen. Which process do they use?
a. Linkage analysis
b. DNA sequencing
c. DNA microarray analysis
d. Polymerase chain reaction
e. Comparative genomic hybridization

229. Evidence that the development of malignancy is a multistep process includes 42)
a. the occurrence of retinoblastoma at a younger age when the disease is sporadic rather than
when it is inherited
b. the observation that certain tumor suppressor genes and oncogenes are involved in a
sequential manner in the development of colon cancer
c. the fact that c-oncogenes are widely conserved through evolution
d. the observation that normal tissues are usually of clonal origin, whereas, adjacent tumor
tissue develops from multiple progenitor cells
e. the development of neoplasia as a result of activation of a single oncogene by any of a
variety of different mechanisms

230. A gene product thought to be involved in the down-regulation of fetal hemoglobin


expression is being investigated. Samples of primary tissue cultures of fetal, neonatal, and adult
liver and bone marrow, as well as adequate amounts of a DNA probe believed to contain the
studied gene, are provided. The best method for determining which of the tissue culture
samples expresses the studied gene is
a. DNA sequencing
b. Southern Blot
c. Western Blot
d. Polymerase chain reaction (PCR)
e. Northern Blot

231. Metastasis involves:


a. adding telomeres to the center of the chromosomes
b. the ability of cancer cells to enter body fluids (blood) and travel to distant sites, where they
c. may form tumors
d. the need for a surface in order to proliferate and survive
e. the tendency to grow in a confined local area
f. increased normal cellular differentiation of the primary tumor

232. A 28-year-old woman and a 25-year-old man present for genetic counseling. Both are white
and have one sibling affected with cystic fibrosis. The most appropriate method to assess the
risk of transmitting cystic fibrosis to a potential child would be
a. Karyotype analysis
b. Western Blot
c. Biochemical testing
d. Fluorescence in situ hybridization (FISH)
e. Polymerase chain reaction (PCR)

233. You have just started a rotation in a genetics lab focused on screening DNA samples from
patients of Eastern European Jewish decent for the most common mutation in the
Hexosaminidase A gene. The most common mutation in this population is a four base pair
insertion which causes a frameshift (see sequences below). The mutation can easily be detected
by PCR amplification followed by gel electrophoresis. You need to design PCR primers to
selectively amplify this region of the Hexosaminidase A gene to detect the insertion.
Wild Type:
5'-CTGCCCCCTGGTACCTGAACCGTATATCCTATGGCCCTGACTGGAAGGATTTCTACGTAG
-3'
4 Base pair frameshift mutation (underlined and bold):
5'-CTGCCCCCTGGTACCTGAACCGTATATCTATCCTATGGCCCTGACTGGAAGGATTTCTAC
GTAG-3'
Which pair of primers should you use for PCR amplification of this region?
a. 5'-GACGGGGGACCATGGACTTG-3' and 5'-CTGGAAGGATTTCTACGTAG-3'
b. 5'-GTTCAGGTACCAGGGGGCAG-3' and 5'-CTGGAAGGATTTCTACGTAG-3'
c. 5'-CTGCCCCCTGGTACCTGAAC-3' and 5'-CTACGTAGAAATCCTTCCAG-3'
d. 5'-CTGCCCCCTGGTACCTGAAC-3' and 5'-CTGGAAGGATTTCTACGTAG-3'
e. 5'-GTTCAGGTACCAGGGGGCAG-3' and 5'-CTACGTAGAAATCCTTCCAG-3'

234. Which of the following best describes the "soil and seed" hypothesis? 47)
a. the method in which blood transports cancerous cells
b. the method in which some tissues provide optimal growth conditions for cancerous cell
c. metastasis
d. a description of the spread of malignant tumors through the lymph system

235. This technique is used usually to detect an allelic version that is a common cause of the
disease and results from a point mutation in the gene. What is the method?
a. Northern blot analysis
b. Allele-specific oligonucleotide (ASO) hybridization
c. Western blot analysis
d. Polymerase chain reaction
e. Southern blot analysis

236. A 30-year-old woman is diagnosed with cervical intraepithelial neoplasia with a previous
viral infection. Which of the following viral products is (are) implicated in producing this type of
dysplasia?
a. EBNA proteins
b. Large tumor antigen
c. E6 and E7 proteins
d. E1A and E1B proteins

237. The Ames test is a screening test used to predict whether a chemical is likely to cause
__________ in human cells.
a. recombination
b. cancer
c. resistance to antibiotics
d. auxotrophic mutations
238. A 45-year-old male patient previously treated for colon cancer, was recently diagnosed with
metasized tumors in his liver.
Which drug can he take that will selectively interfere with angiogenesis by targeting VEGF?
a. Interleukin-2
b. Herceptin
c. Avastin
d. Fluorouracil
e. Gleevec

239. N-myc can be best classified as


a. a tummor suppressor
b. gene an oncogene

240. Which of the following disorders involve an altered interaction of gene product with beta-
catenin?
a. Sickle-cell disease
b. Tay-Sachs disease
c. Alpha-1 antitrypsin deficiency
d. Familial adenomatous polyposis coli
e. Obesity

241. The characteristic translocation in Burkitt lymphoma 54)


a. involves the abl oncogene and bcr (break cluster region)
b. involves the myc oncogene and an immunoglobulin locus.
c. is t(8;22)
d. involves the ras oncogene and bcr (break cluster region)
e. is t(9;22)

242. Which of the following inducers of angiogenesis is both an endothelial mitogen and an
endothelial motogen?
a. Transforming growth factor beta
b. Vascular endothelial growth factor-A
c. Fas ligand
d. Platelet-derived endothelial cell growth factor Angiogenin

243. Which of the following genes is responsible for Familial Polyposis Coli? 56)
a. APC
b. BRCA1
c. NF2
d. MLH1

244. Identify the appropriate chromosomal transloction that is most frequently found with
Burkitt lymphoma.
a. t(11;14) (q13;q32)
b. t(14;18) (q32;q21)
c. t(9;22) (q34;q11)
d. t(8;14)(q24;q32)

245. Restriction enzymes have which one of the following characteristics? They 58)
a. can cleave only circular DNA
b. cleave different DNA's randomly
c. can cleave different DNAs only once
d. generate either staggered (sticky) or blunt ends upon cleaving DNA
e. can cleave both DNA and RNA

246. Which one of the following statements best describes most malignant neoplasms? 59)
a. They are due to an inherited mutation of an oncogene
b. Most are of multifactorial origin
c. They are associated with constitutional chromosomal abnormalities
d. They result from activation of tumor suppressor genes
e. None of the above

247. Restriction fragment length polymorphisms are caused by variations in 60)


a. the methylation pattern of the DNA
b. the splicing of mRNA
c. which nucleotide changes eliminate restriction sites in the genome
d. which the promoter affinities are affected
e. amino acid sequences within populations

248. An enzyme used to covalently join DNA segments to form recombinant DNA molecules is
called a
a. Helicase
b. Reverse transcriptase
c. Restriction endonuclease
d. DNA polymerase
e. Ligase

249. DNA fragments migrate in an electric field because DNA molecules 62)
a. Are neutral in charge
b. Are double stranded
c. Are positively charged
d. Are negatively charged
e. Contain numerous covalent bonds
250. In order for a tumor to grow beyond a certain size, new blood vessels must be formed to
provide the tumor cells with the required nutrients. many new antitumor strategies are being
developed to try to block the formation of these new blood vessels. The process by which new
blood vessels form from existing capillaries is called
a. Ametastasis
b. capillophoresis
c. differentiation
d. angiogenesis
e. hematopoiesis

251. Which of the following accounts for the increased risk for skin cancer in patients with
Xeroderma Pigmentosum?
a. Their cells are deficient in mismatch repair.
b. Their cells are deficient in nucleotide excision repair of UV-induced DNA lesions.
c. Their cells are deficient in transcription-coupled repair of UV-induced DNA lesions.
d. Their cells are sensitive to crosslinking agents.
e. They inherit one mutation in a DNA repair gene and there is a high rate of loss of the second
copy.

252. A researcher is trying to identify a specific protein within a mixture. He subjects the mixture
to gel electrophoresis and then transfers the separation to nitrocellulose filters. The filters are
incubated with antibody to the specific protein, and the excess antibody is washed off. The
antibody-protein complex is then incubated with a radiolabeled protein that binds to the
antibody. Autoradiography is performed to detect the presence of the protein. The technique
represents
a. Northern blotting
b. Western blotting
c. Southwestern blotting
d. Southern blotting

253. Which of the following is true about PTEN Hamartoma Tumor Syndrome? 66)
a. The mutation in PTEN prevents PI3 kinase activation
b. The mutation in PTEN inhibits its phosphatase activity
c. The mutation in PTEN prevents AKT activation
d. The mutation in PTEN causes the lipid PIP3 levels to decrease rapidly

254. Recent evidence indicates that there are a number of mechanisms which can transform a
normal cell into a cancer cell. Many of these involve genetic mutations leading to altered
expression or function of a protein. Proteins can also mutate and lose functions. Among these
proteins are a class called tumor suppressor genes. These tumor suppression genes function as
inhibitors of
a. Protein synthesis
b. Cell proliferation
c. Retroviral replication
d. RNA transcription
e. Cellular differentiation

255. Sickle cell anemia is caused by a missense mutation in codon 6 of the -globin gene.
Normal allele: 5'- CCT GAG GAG AAG -3'
Mutant allele: 5'- CCT GTG GAG AAG -3'
A man with sickle cell disease and his phenotypically normal wife request genetic testing because
they are concerned about the risk for their unborn child. DNA samples from the men and the
woman and from fetal cells obtained by amniocentesis are analyzed using the PCR to amplify exon
1 of the -globin gene.
You need to analyze the amplified material under stringent conditions using allele-specific
oligonucleotide probes.Which 12-base oligonucleotide sequence will you most likely use as a
specific probe complementary to the coding strand of the sickle cell allele?
a. 5'-GGACACCTCTTC-3'
b. 5'-CCTCACCTCAGG-3'
c. 5'-CCTGTGGAGAAG-3'
d. 5'-CTTCTCCACAGG-3'
e. 5'-CTTCTCCTCAGG-3'

256. Columnar epithelial cells from the colonic mucosa are studied to identify abnormalities in
cell signaling pathways. Abnormal epithelial cells from colonic adenocarcinoma are shown to
have a mutation that blocks hydrolysis of GTP-bound active RAS. Normal columnar cells have
active RAS protein that undergoes hydrolysis to the inactive GDP-bound form. Which of the
following signaling pathways is most likely abnormally stimulated in the carcinoma cells?
a. Cyclic AMP
b. BCR-ABL
c. Nf-KB
d. MAP kinase
e. JAK/STAT

257. The most common genetic change that occurs in the development of tumors is 70)
a. inactivation or loss of the p53 gene by somatic mutation
b. activation of the c-myc gene by chromosome rearrangement
c. inherited inactivation or loss of the p53 gene
d. activation of the c-myc gene by somatic mutation
e. inherited activation of the c-myc gene

258. Adult glandular tissues, such as those found in the breast, prostate, and intestine, tend to
develop cancer more often than certain other tissues like muscle and brain. Why?
a. Brain and muscle cells divide more often than glandular cells.
b. DNA of brain and muscle cells is more intrinsically resistant to damage.
c. Normal glandular cells lack DNA repair systems.
d. Normal brain and muscle cells have more DNA repair systems than do glandular cells.
e. Glandular cells replicate their DNA more often than do brain and muscle cells.

259. Analysis of a tumor biopsy revealed the absence of any Rb protein. However, genetic analysis
of the tumor confirms that the both Rb alleles are normal and are transcriptionally active with
normal levels of wild-type mRNA produced.What is the most likely reason that this patient's
tumor lacks Rb protein but has normal Rb mRNA levels?
a. Overexpression of oncomir miR-106a
b. p53 induced gene expression of the p21 inhibitor
c. The Rb gene locus at 13q14 is transcriptionally inactivated by spontaneous methylation of
the local chromatin
d. Decreased expression of oncomir miR-21
e. The histones associated with the Rb gene locus are deacetylated

260. The p53 gene


a. is the most frequently mutated gene in human cancer
b. can lead to cell cycle arrest at the G1 checkpoint
c. can trigger apoptosis.
d. all of the above

261. A benign tumor is one in which the cancerous cells:


a. have an unusual number of chromosomes
b. can divide indefinitely if an adequate supply of nutrients is available
c. migrate from the initial site of transformation to other organs or
d. remain confined to their original site

262. During the Ras pathway:


a. cytoplasmic protein kinases are activated
b. the growth factor receptor is dephosphorylated
c. growth factors bind to receptors in the cytoplasm
d. leads to the production of translation factors

263. Which of the following would be most likely to lead to cancer?


a. hyperactivity of both a proto-oncogene and a tumor-suppressor
b. amplification of a proto-oncogene and the failure of a tumor-suppressor gene
c. hyperactivity of bith a proto-oncogene and a tumor suppressor gene
d. failure of a proto-oncogene to produce a protein and amplification of tumor suppressor
gene

264. What is true of proto-oncogenes?


a. cells produce proto-oncogenes as a by-product of mitosis
b. proto-oncogenes are necessary for normal control of cell division
c. proto-oncogenes are genetic junk that has not yet been eliminated by natural
selection
d. proto-oncogenes are unavoidable environmental carcinogens.

265. Cancers cell are monoclonal, are characterized by uncontrolled growth, invasion of other
tissues and dissemination to other tissues. The phenomenon of invasion to other tissues is
termed as-
a. Angiobiogenesis
b. Metastasis
c. Diapedesis
d. Transformation

266. Cancer causing genes can be functionally classified into mainly three types:
(i) genes that induce cellular proliferation,
(ii) tumor suppressor genes,
(iii) genes that regulate apoptotic pathway.
Epstein-Barr virus that causes cancer by modulating apoptotic pathway, contains a gene
having sequence homology with which of the following genes?
a. bax
b. bcl-2
c. p53
d. caspase-3

Part C
267. What is the most likely sentence from the following 4 alternatives?
A. The cellular proto oncogene originated from viral oncogene
B. The viral oncogene originated from cellular proto-oncogene
C. The c-myc has anti apoptotic activity
D. Bcl-2 has anti apoptotic activity
a. Only A
b. Only B
c. Both A and D
d. Both B and D
268. The c-ras gene products are involved in cancer development and it acts as a proto oncogene,
causing colon cancer. The homology analysis has shown that the oncogene c-ras differs from the
proto oncogene by just one base pair. The possible kind of mutation might have occurred here is
a. Base substitution
b. Frame shift mutation
c. Homologous recombination
d. None of the above
269. The majority of the c-ras oncogene have alterations in one of the 3 codons in the c-ras mRNA
that lead to the substitution of amino acids in the position 12, 59 and 61 from the amino
terminus. How can the clustering of mutational events to these 3 sites in the 3 c-ras genes are
explained?
a. The genes are the part of promoter and when the promoter is not under control, the
c-ras gets over expressed
b. The 3 regions are in the terminator locations, so when terminator is non functional
the c-ras protein mutated
c. The 3 locations code for amino acids, which are required for the normal functioning
of c-ras proteins
d. The mutation in these 3 locations produces proteins which enhance the protein
activity.
270. Which of the following is NOT a possible way of converting a cellular proto oncogene into a
cellular oncogene?
a. Base pair substitution mutations such as those observed in the c-ras gene
b. Insertion of a stron enhancer or promoter sequence adjascent to the protooncogene,
thus leading to over production
c. Amplification of the protooncogene leading to overproduction of its product
d. Deletion of promoter, leading loss of control on RNA polymerase binding, resulting
in mutated and over produced proteins
271. Methotrexate is a drug used to block DNA synthesis. It blocks the enzyme Dihydrofolate
reductase, and thus suppresses dTMP synthesis. The mutant cells that have become cancerous
show tolerance to the drug and hence continue to divide even in the presence of Methotrexate.
Chromosomal analysis of these mutants shows the formation of extra very smallc chromosomes
called double minutes, DM. These DM are found in extrachromosomal DNA and show
homogenous staining pattern. Which of the following can be the reason for the tolerance to this
drug?
a. The cells mutate the receptor which binds to the drug, so that no more drug can bind
to cells
b. The cells amplify the gene , resulting in 1000s of copies of gene coding for
Dihydrofolate reductase,
c. The cells mutate the Dihydrofolate reductase, so that the drug cannot bind to it
d. The cells show resistance allele for the drug.
272. Philadelphia chromosome is a chromosomal abnormality associated with chronic
myelogenous leukemia. Which of the following is the correct definition of Philadelphia
chromosome?
a. Translocated chromosome 9 from 22
b. Translocated chromosome 22 from 9
c. Base substitution in chromosome 22
d. Base substitution in chromosome 9

273. Consider a family where a father and son both have retinoblastoma. DNA analysis from the
child's tumor shows only a single allele from the Rb locus on chromosome 13. Both parents are
heterozygous in blood, as is the child. Which allele would you expect is preserved in the tumor?
a. Mother's
b. Father's
c. There is a 50:50 chance that it is the mother's or father's allele.
d. Both alleles would be abnormal due to genetic rearrangement.

274. In lab, you are studying cell cycle control in the fission yeast, S. pombe. A student finds a new
mutant that she wants to call giant because the cells are much larger than normal (suggesting
that it is not dividing normally). What type of mutation do you think your student has isolated?
a. a loss-of-function mutation in a tumor-suppressor gene
b. a loss-of-function mutation in a cellular proto-oncogene
c. a gain-of-function mutation in a tumor-suppressor gene
d. Both a and d are possible.
e. Both b and c are possible.

275. Using a car and driver analogy, which of the following accurately describes the role of
tumor-suppressor genes and proto-oncogenes in normal cells?
a. Tumor-suppressor genes are the gas pedal, while proto-oncogenes are the brakes.
b. Tumor-suppressor genes are the brakes while proto-oncogenes are the gas.
c. Both tumor-suppressor genes and proto-oncogenes are like the gas, but tumor-suppressors
are like turbo and proto-oncogenes are like a regular carburetor.
d. Tumor-suppressor genes are like the steering wheel, and proto-oncogenes are like the turn
signals.

276. During the early years of cancer research, there were two schools of thought regarding the
causes of cancer: 1) that cancer was caused entirely by environmental factors, and 2) that
cancer was caused by genetic factors. Which was correct?
a. #1 because we have identified many potential carcinogens
b. #2 because we know of many proto-oncogenes
c. #2 because we know of many tumor-suppressor genes
d. Both were correct; most chemical carcinogens function by altering genes.

277. If you found a specific chromosomal deletion in the genome from a tumor, what could be the
cause of this specific cancer?
a. The deletion likely affected a tumor-suppressor gene, leading to a loss of function in the
tumor cells.
b. The deletion likely affected a proto-oncogene, leading to a loss of function in the tumor
cells.
c. The deletion likely affected a tumor-suppressor gene, leading to a gain of function in the
tumor cells.
d. The deletion likely affected a proto-oncogene, leading to a gain of function in the tumor
cells.
278. A 44-year-old woman notes a lump in her left breast while taking a shower. Her physician
notes a 3 cm firm, irregular, non-movable mass located in the upper outer quadrant of her left
breast on physical examination. A fine needle aspiration of this mass is performed. Cells
obtained from the mass are examined cytologically and are consistent with infiltrating ductal
carcinoma. The mass is removed with lumpectomy along with an axillary lymph node
dissection. Which of the following findings will best predict a better prognosis for this patient?
a. The tumor cells are strongly estrogen receptor positive
b. No metastases are found in the sampled lymph nodes
c. Flow cytometric analysis demonstrates aneuploidy and a high S-phase
d. She has one relative who had a similar type of breast cancer

279. A change in bowel habits prompts a 53-year-old woman to see her physician. On physical
examination there are no lesions noted on digital rectal examination, but her stool is positive for
occult blood. A colonoscopy is performed and reveals a 6 cm friable mass located in the cecum.
A biopsy of this mass is performed and microscopic examination shows a moderately
differentiated adenocarcinoma. Which of the following findings is most likely to be present in
this patient?
a. k-ras mutation in the neoplastic cells
b. An immunoperoxidase stain positive for vimentin in the neoplastic cells
c. A stool culture positive for Shigella flexneri
d. A plasma HIV-1 RNA level of 40,000 copies/mL

280. A 45-year-old healthy woman has a routine check of her health status. She has no chest pain,
cough, or fever. A chest x-ray taken and shows a peripheral 2.5 cm diameter "coin lesion" in the
right mid-lung field. Which of the following biologic characteristics best distinguishes this
lesion as a neoplasm, rather than a granuloma?
a. Recurrence following excision
b. Rapid increase in size
c. Sensitivity to radiation or chemotherapy
d. Uncontrolled (autonomous) growth

281. A clinical study is performed on biopsy specimens obtained from patients who had a
malignant neoplasm. It is observed that a subset of these biopsies shows a neoplasm composed
predominantly of cells with a spindle shape and a high N/C ratio with marked pleomorphism.
These cells are vimentin positive, cytokeratin negative, and CD45 negative by
immunohistochemical staining. This type of neoplasm is most likely to have been diagnosed in
which of the following patients?
a. A 35-year-old woman with a left breast mass and enlarged axillary lymph nodes
b. 55-year-old woman with massive ascites and multiple peritoneal metastases
c. 25-year-old man with an enlarged left testis
d. 15-year-old man with a mass in the left femur and lung metastases

282. A 48-year-old woman goes to her physician for a routine physical examination. A 4 cm
diameter non-tender mass is palpated in her right breast. The mass appears fixed to the chest
wall. Another 2 cm non-tender mass is palpable in the left axilla. A chest radiograph reveals
multiple 0.5 to 2 cm nodules in both lungs.mWhich of the following classifications best indicates
the stage of her disease?
a. T1 N1 M0
b. T1 N0 M1
c. T2 N1 M0
d. T4 N1 M1
283. A 51-year-old man has worked for 10 years in a factory producing plastic pipe. He has noted
weight loss, nausea, and vomiting worsening over the past 5 months. On examination he is
afebrile. There is generalized muscle wasting. Laboratory studies show the serum alkaline
phosphatase is 405 U/L with AST 67 U/L, ALT 55 U/L, and total bilirubin 1.2 mg/dL. An
abdominal CT scan reveals a 12 cm right liver lobe mass. Liver biopsy reveals a neoplasm
composed of spindle cells forming irregular vascular channels. The cells demonstrate vimentin
positivity and cytokeratin negativity with immunohistochemical staining. Exposure to which of
the following substances most likely led to development of this neoplasm?
a. Benzene
b. Radon
c. Asbestos
d. Vinyl chloride

284. A child is born with a single functional allele of a tumor suppressor gene. At the age of five
the remaining normal allele is lost through a point mutation. As a result, the ability to continue
the transition from G1 to the S phase of cell cycle is lost. Which of the following neoplasms is
most likely to arise via this mechanism?
a. infiltrating ductal carcinoma of breast
b. Small cell anaplastic carcinoma of the lung
c. Retinoblastoma of eye
d. Cerebral astrocytoma

285. A 22-year-old woman goes to her physician for a routine examination. A palpable nodule is
found in the right lobe of her thyroid gland. No lymphadenopathy is noted. A chest x-ray shows
no masses. A fine needle aspirate of the nodule is performed and cytologic examination reveals
cells present consistent with a papillary carcinoma of the thyroid. There are no other family
members affected by this disorder. She works as a secretary for an accounting firm part time
and is earning a college degree. Which of the following findings would you consider most
relevant in her past history to indicate a risk factor for this neoplasm?
a. Chronic alcoholism
b. Radiation therapy in childhood
c. Ataxia telangiectasia
d. Blunt trauma from a fall

286. A 52-year-old man has had increasing fatigue for the past 6 months. On physical
examination he has a palpable spleen tip. Laboratory studies show a WBC count of
189,000/microliter. The peripheral blood smear shows many mature and immature myeloid
cells present. Cytogenetic analysis of cells obtained via bone marrow aspiration reveals a t(9:22)
translocation. This translocation leads to formation of a hybrid gene that greatly increases
tyrosine kinase activity. Which of the following genes is most likely translocated to cause these
findings?
a. p53
b. Rb
c. c-abl
d. NF-1

287. A 40-year-old woman has had a feeling of abdominal discomfort for the past 8 months. On
pelvic examination, there is a right adnexal mass. An abdominal CT scan demonstrates a 7 cm
cystic mass involving the right ovary with small areas of calcification. The uterus is normal in
size. The right fallopian tube and ovary are removed surgically. Grossly, the mass on sectioning
is filled with abundant hair and sebum. Microscopically, the mass has glandular spaces lined by
columnar epithelium, squamous epithelium with hair follicles, cartilage, and dense connective
tissue. Which of the following statements regarding this type of neoplasm is most appropriate?
a. A sarcomatous element is usually present.
b. human papillomavirus infection preceded development of this mass.
c. Metastases are unlikely to be present.
d. Primitive immature tissues resembling those from an embryo can usually be seen.

288. A 29-year-old woman with a history of multiple sexual partners over the last 15 years has a
routine physical examination with no abnormal findings. On pelvic examination, the cervix
shows no abnormalities, but a Pap smear is taken and dysplastic cells are reported to be
present. A cervical biopsy is performed and shows microscopic features of a minimal dysplasia
(CIN 1) involving the cervical squamous epithelium. Which of the following is the most
appropriate statement to make to the patient regarding these findings?
a. No further treatment is indicated
b. Antibiotic therapy will be necessary
c. You probably have widespread metastases
d. Complete excision of the lesion is required

289. A 64-year-old man has noted a 5 kg weight loss along with increasing fatigue over the past
year. He has experienced dull abdominal pain for the past week. He has developed abdominal
distention with lack of stools in the past two days. On physical examination, bowel sounds are
reduced. An abdominal CT scan reveals a mass involving the descending colon. At laparotomy, a
partial resection of the left colon is performed, with removal of an encircling mass in the
descending colon. Microscopically, the mass is found to be a moderately differentiated
adenocarcinoma. Which of the following laboratory test findings is most likely to be present in
this man?
a. Microcytic hypochromic anemia
b. Positive antinuclear antibody test
c. Hyperglycemia
d. Elevated alpha-fetoprotein

290. A a 27-year-old woman in excellent health has a routine health maintenance examination. A
2 cm firm, rounded mass is palpable beneath the skin of the left forearm. She has no difficulty
using the arm and there is no associated pain with the mass, either in movement or on
palpation. The overlying skin appears normal. The mass does not change in size over the next
year. Which of the following neoplasms is she most likely to have?
a. Metastatic carcinoma
b. Melanoma
c. Rhabdomyosarcoma
d. Lipoma

291. A 41-year-old woman has noted a foul-smelling vaginal discharge for 3 weeks. On physical
examination there is an exophytic 3 cm mass involving the ectocervix. Pap smear testing is
performed; she has never had a previous Pap smear. Cytologic changes are present on this Pap
smear that are consistent with squamous cell carcinoma. She is found to have a positive
serologic test for syphilis. Her serum glucose is 157 mg/dL. She has been a commercial sex
worker in the past. Which of the following is the most likely risk factor for her cervical
carcinoma?
a. Human papillomavirus infection
b. Diabetes mellitus, type II
c. Heavy cigarette smoking
d. Pelvic inflammatory disease
292. A 59-year-old man went to his physician for a routine health maintenance examination. The
only abnormal physical examination finding is a positive stool guaiac test. Laboratory studies
show a CBC with Hgb 10.0 g/dL, Hct 29.8&, MCV 73 fL, platelet count 300,000/microliter, and
WBC count 8700/microliter. He is found on colonoscopy to have a 4 cm mass lesion arising on
the mucosal surface of the transverse colon. Which of the following statements is most
appropriate regarding these findings?
a. This is an uncommon site for a primary malignant neoplasm
b. Microscopically, the mass is probably an adenocarcinoma
c. Persons who are smokers are at much greater risk to develop this lesion
d. Hypercalcemia is a frequent paraneoplastic complication
293. A 66-year-old man has noted darker urine for the past 2 weeks. A urinalysis shows
hematuria. He is referred to a urologist who performs cystoscopy. A 3 cm mass is found in the
dome of the bladder. Biopsies of the mass are taken and on microscopic examination show a
urothelial carcinoma. Cells of this neoplasm demonstrate a single mutation causing cellular
inability to hydrolyze GTP, thus resulting in cellular transformation. Which of the following
oncogenes is most likely implicated in this case?
a. c-abl
b. c-neu
c. c-sis
d. k-ras
294. An 8-year-old boy has complained of difficulty swallowing for the past month. On physical
examination he has a palpable, firm mass in the right neck. A head CT scan reveals a 7 cm solid
soft tissue mass lateral to the esophagus on the right. A biopsy of this mass demonstrates
pleomorphic spindle cells that are cytokeratin negative, CD45 negative, and vimentin positive.
Further history indicates that this boy was affected by fetal alcohol syndrome and he does
poorly in school. Which of the following neoplasms is he most likely to have?
a. Wilms tumor
b. Rhabdomyosarcoma
c. Neuroblastoma
d. Malignant lymphoma

ANSWER KEYS:

1.c
2.a
3.d
4.a
5.a
6.b
7.a
8.d
9.c
10.c
11.a
12.d
13.c
14.b
15.a
16.b
17.c
18.b
19.b
20.a
21.a
22.d
23.a
24. b
25.c
26.c
27.d
28.d
29.a
30.c
31.c
32.b
33.d
34.b
35.b
36.c
37. d
38. a
39. a
40. d
41. b
42. b
43. a
44. a
45. a
46. e
47. a
48. e
49. e
50.a
51.b
52.c
53.b
54.d
55.d
56.d
57.a
58.a
59.d
60.c
61.c
62.a
63.a
64.b
65.d
66.a
67.d
68.a
69.c
70.b
71.d
72.d
73.a
74.d
75.b
76.d
77.a
78.c
79.d
80.b
81.d
82.a
83.c
84.d
85.c
86. c
87.d
88.d
89.d
90.c
91.b
92.a
93.a
94.e
95.b
96.a
97.a
98.b
99.c
100.c
101.d
102.b
103.a
104.d
105.c
106.c
107.a
108.a
109.b
110.d
111.e
112.b
113.e
114.e
115.c
116.e
117.c
118.b
119.b
120.e
121.d
122.a
123.b
124.b
125.a
126. b
127. a
128.a
129.a
130.d
131.a
132.e
133.a
134.b
135.b
136.b
137.b
138.c
139.c
140.c
141.b
142.b
143.a
144.c
145.a
146.b
147.a
148.d
149.d
150.c
151.a
152.b
153.b
154.c
155.b
156.a
157.a
158.a
159.c
160.a
161.b
162.c
163.d
164.b
165.c
166.d
167.c
168.c
169.d
170.c
171.a
172.c
173.d
174.c
175.a
176.d
177.a
178.b
179.b
180.a
181.c
182.d
183.a
184.d
185.c
186.a
187.c
188.a
189. a
190. a
191. a
192. d
193. a
194. A
195.E
196.D
197.C
198.C
199.E
200.A
201.B
202.D
203.A
204.B
205.C
206.D
207.E
208.D
209.C
210.E
211.C
212.A
213.E
214.D
215.C
216.B
217.A
218.E
219.C
220.A
221.E
222.D
223.D
224.C
225.E
226.B
227.E
228.C
229.B
230.E
231.B
232.E
233.C
234.B
235.B
236.C
237.B
238.C
239.B
240.D
241.B
242.B
243.A
244.D
245.D
246.B
247.C
248.E
249.D
250.D
251.B
252.B
253.B
254.B
255.D
256.D
257.A
258.E
259.A
260.D
261.D
262.b
263.B
264.B
265.B
266.B
267.d
268.a
269.c
270.d
271.b
272.b
273.b
274.e
275.b
276.d
277.a
278.b
279.a
280.d
281.d
282.d
283.d
284.c
285.b
286.c
287.c
288.d
289.a
290.d
291.a
292.b
293.d
294.d
Immune system
Part B

1. An antigen is?
a. a protein molecule that helps defend the body against disease.
b. a type of white blood cell.
c. an invading virus or bacterium.
d. a foreign molecule that evokes a specific response by a lymphocyte.

2. How do memory cells differ from effector cells?


a. Memory cells are more numerous.
b. Memory cells are responsible for the primary immune response.
c. Memory cells attack invaders; effector cells do not.
d. Memory cells live longer.

3. Following tissue damage or the entry of microorganisms, an inflammatory response


may be initiated by
a. the accumulation of phagocytes in an injured area.
b. the release of interferon by infected cells.
c. an increased blood flow in an infected or injured area.
d. the release of chemicals such as histamine by damaged cells.

4. Which of the following could be considered a nonspecific defense?


a. Intact skin creates a barrier that cannot normally be penetrated by bacteria or
viruses.
b. Secretions from sebaceous and sweat glands give the skin an acidic pH that
prevents bacterial colonization.
c. Tears, saliva, and mucous secretions contain lysozome, an enzyme that digests
the bacterial cell wall.
d. all of the above

5. During a secondary immune response


a. selected B generate antibody-producing effector B cells called plasma cells.
b. the stricken individual may become ill.
c. about 10 to 17 days are required from exposure to maximum effector response.
d. the generation of effector cells begins with memory cells produced during the
primary immune response.

6. Most individuals infected with HIV


a. can live for 15 to 20 years.
b. never develop AIDS.
c. die from autoimmune reactions.
d. die from other infections or cancer.

7. Tissues are typed before an organ transplant to make sure that the _____ of donor
and recipient match as closely as possible.
a. T cells
b. Antibodies
c. MHC (major histocompatibility complex) proteins
d. histamines

8. A vaccine contains
a. white blood cells that fight infection.
b. antibodies that recognize invading microbes.
c. inactivated disease-causing microbes.
d. a hormone that boosts immunity.

9. When you are immune to a disease,


a. antibodies against the disease are constantly circulating in your blood.
b. certain lymphocytes are able to make the proper antibodies quickly.
c. your nonspecific defenses are strengthened.
d. B cells are stimulated to quickly engulf invaders.

10. In a series of immune system experiments, the thymus glands were removed from
baby mice. Which of the following would you predict as a likely result?
a. The mice suffered from numerous allergies.
b. The mice never developed cancerous tumors.
c. The mice suffered from autoimmune diseases.
d. The mice readily accepted tissue transplants.

11. The body produces antibodies complementary to foreign antigens. The process by
which the body comes up with the correct antibodies to a given disease is most like
a. going to a tailor and having a suit made to fit you.
b. ordering the lunch special at a restaurant without looking at the menu.
c. going to a shoe store and trying on shoes until you find a pair that fits.
d. selecting a lottery prize-winner by means of a random drawing.

12. The antigen-binding sites of an antibody molecule are formed from the molecules
variable regions. Why are these regions called variable?
a. They can change their shapes on command to fit different antigens.
b. They change their shapes when they bind to an antigen.
c. They can be different shapes on different antibody molecules.
d. Their sizes vary considerably from one antibody to another.

13. The biggest difference between cell-mediated immunity and humoral immunity is
a. how long their protection lasts.
b. whether a subsequent secondary immune response can occur.
c. whether clonal selection occurs.
d. how they respond to and dispose of invaders.

14. Viruses and bacteria in body fluids are attacked by


a. antibodies from B cells.
b. cytotoxic T cells.
c. complement proteins.
d. helper T cells.

15. What do the antibodies secreted by plasma cells (the effector cells of humoral
immunity) do to attack their targets?
a. activate complement to punch holes in them
b. clump cells together so that phagocytes can ingest them
c. cause antigen molecules to settle out of solution
d. all of the above

16. Tissue macrophages


a. begin their lives as neutrophils.
b. have short life spans because they self-destruct after engulfing foreign invaders.
c. originate from monocytes that leave the circulation and enter the tissues.
d. are most effective against parasites.

17. The idea behind vaccination is to induce _____ without the vaccinated individual
having to get sick.
a. passive immunity
b. the primary immune response
c. anaphylactic shock
d. nonspecific defenses

18. A group of researchers have tested many chemicals and found several that have
potential for use in modifying the action of the immune system. Which of the following
would seem to have the most promise as a drug for inhibiting transplant rejection?
a. Compound A13: acts like histamine
b. Compound Q6: suppresses cytotoxic T cells
c. Compound N98: a potent allergen
d. Compound M31: stimulates helper T cells

19. Which category of hypersensitivity BEST describes hemolytic disease of the newborn caused
by Rh incompatibility?
a. atopic or anaphylactic
b. cytotoxic
c. immune complex
d. delayed

20. The principal difference between cytotoxic (type II) and immune complex (type III)
hypersensitivity is
a. the class (isotype) of antibody.
b. the site where antigen-antibody complexes are formed.
c. the participation of complement.
d. the participation of T cells.
21. A child stung by a bee experiences respiratory distress within minutes and lapses into
unconsciousness. This reaction is probably mediated by
a. IgE antibody
b. IgG antibody
c. sensitized T cells
d. complement
e. IgM antibody

22. A kidney biopsy specimen taken from a patient with acute glomerulonephritis and stained
with fluorescein-conjugated anti-human IgG antibody would probably show
a. no fluorescence.
b. uniform fluorescence of the glomerular basement membrane.
c. patchy, irregular fluorescence of the glomerular basement membrane.
d. fluorescent B cells.
e. fluorescent macrophages.

23. A patient with severe asthma gets no relief from antihistamines. The symptoms are MOST
likely to be caused by
a. interleukin-2.
b. slow-reacting substance A (leukotrienes).
c. serotonin.
d. bradykinin.

24. Hypersensitivity to penicillin and hypersensitivity to poison oak are both


a. mediated by IgE antibody.
b. mediated by IgG and IgM antibody.
c. initiated by haptens.
d. initiated by Th-2 cells.

25. A recipient of a 2-haplotype MHC-matched kidney from a relative still needs


immunosuppression to prevent graft rejection because
a. graft-versus-host disease is a problem.
b. minor histocompatibility antigens will not be matched.
c. minor histocompatibility antigens will not be matched.
d. complement components will not be matched.

26. Bone marrow transplantation in immunocompromised patients presents which major


problem?
a. potentially lethal graft-versus-host disease
b. high risk of T cell leukemia
c. inability to use a live donor
d. delayed hypersensitivity

27. What is the role of class II MHC proteins on donor cells in graft rejection?
a. They are the receptors for interleukin-2, which is produced by macrophages when they
attack the donor cells.
b. They are recognized by helper T cells, which then activate cytotoxic T cells to kill the
donor cells.
c. They induce the production of blocking antibodies that protect the graft.
d. They induce IgE which mediates graft rejection.

28. Grafts between genetically identical individuals (i.e., identical twins)


a. are rejected slowly as a result of minor histocompatibility antigens.
b. are subject to hyperacute rejection.
c. are not rejected, even without immunosuppression.
d. are not rejected if a kidney is grafted, but skin grafts are rejected.

29. Penicillin is a hapten in both humans and mice. To explore the hapten-carrier relationship, a
mouse was injected with penicillin covalently bound to bovine serum albumin and, at the
same time, with egg albumin to which no penicillin was bound. Of the following, which one
will induce a secondary response to penicillin when injected into the mouse 1 month later?
a. Penicillin
b. penicillin bound to egg albumin
c. egg albumin
d. bovine serum albumin

30. AIDS is caused by a human retrovirus that kills


a. B lymphocyte
b. lymphocyte stem cells.
c. CD4-positive T lymphocytes.
d. CD8-positive T lymphocytes.

31. Chemically-induced tumors have tumor-associated transplantation antigens that


a. are always the same for a given carcinogen.
b. are different for two tumors of different histologic type even if induced by the same
carcinogen.
c. are very strong antigens.
d. do not induce an immune response.

32. Polyomavirus (a DNA virus) causes tumors in "nude mice" (nude mice do not have a thymus,
because of a genetic defect) but not in normal mice. the BEST interpretation is that
a. macrophages are required to reject polyomavirus-induced tumors.
b. natural killer cells can reject polyomavirus-induced tumors without help from T
lymphocytes.
c. T lymphocytes play an important role in the rejection of polyomavirus-induced tumors.
d. B lymphocytes play no role in rejection of polyomavirus-induced tumors.

33. C3 is cleaved to form C3a and C3b by C3 convertase. C3b is involved in all of the following
EXCEPT
a. altering vascular permeability.
b. promoting phagocytosis.
c. forming alternative-pathway C3 convertase.
d. forming C5 convertase.

34. After binding to its specific antigen, a B lymphocyte may switch its
a. immunoglobulin light-chain isotype.
b. immunoglobulin heavy-chain class.
c. variable region of the immunoglobulin heavy chain.
d. constant region of the immunoglobulin light chain.
35. Diversity is an important feature of the immune system. Which one of the following
statements about it is INCORRECT?
a. Humans can make antibodies with about 108 different VH X VL combinations.
b. A single cell can synthesize IgM antibody, then switch to IgA antibody.
c. The hematopoietic stem cell carries the genetic potential to create more than 104
immunoglobulin genes.
d. A single B lymphocyte can produce antibodies of many different specificities, but a
plasma cell is monospecific.

36. C3a and C5a can cause


a. bacterial lysis.
b. vascular permeability.
c. phagocytosis of IgE-coated bacteria.
d. aggregation of C4 and C2.

37. Neutrophils are attracted to an infected area by


a. IgM.
b. vascular permeability.
c. phagocytosis of IgE-coated bacteria.
d. aggregation of C4 and C2.

38. Complement fixation refers to


a. the ingestion of C3b-coated bacteria by macrophages.
b. the destruction of complement in serum by heating at 56C for 30 minutes.
c. the binding of complement components by antigen-antibody complexes.
d. the interaction of C3b with mast cells.

39. The classic complement pathway is initiated by interaction of C1 with


a. antigen.
b. factor B.
c. antigen-IgG complexes.
d. bacterial lipopolysaccharides.

40. Patients with severely reduced C3 levels tend to have


a. increased numbers of severe viral infections.
b. increased numbers of severe bacterial infections.
c. low gamma globulin levels.
d. frequent episodes of hemolytic anemia.

41. Individuals with a genetic deficiency of C6 have


a. decreased resistance to viral infections.
b. increased hypersensitivity reactions.
c. increased frequency of cancer.
d. decreased resistance to Neisseria bacteremia.

42. Natural killer cells are


a. B cells that can kill without complement.
b. cytotoxic T cells.
c. increased by immunization.
d. able to kill virus-infected cells without prior sensitization.

43. A positive tuberculin skin test (a delayed hypersensitivity reaction) indicates that
a. a humoral immune response has occurred.
b. a cell-mediated immune response has occurred.
c. both the T and B cell systems are functional.
d. only the B cell system is functional.

44. Reaction to poison ivy or poison oak is


a. an IgG-mediated response.
b. an IgE-mediated response.
c. a cell-mediated response.
d. an Arthus reaction.

45. A child disturbs a wasp nest, is stung repeatedly, and goes into shock within minutes,
manifesting respiratory failure and vascular collapse. This is MOST likely to be due to
a. systemic anaphylaxis.
b. serum sickness.
c. an Arthus reaction.
d. cytotoxic hypersensitivity.

46. "Isotype switching" of immunoglobulin classes by B cells involves


a. simultaneous insertion of VH genes adjacent to each CH gene.
b. successive insertion of a single VH gene adjacent to different CH genes.
c. activation of homologous genes on chromosome 6.
d. switching of light-chain types (kappa and lambda).

47. Which one of the following pairs of genes is linked on a single chromosome?
a. V gene for lambda chain and C gene for kappa chain
b. C gene for gamma chain and C gene for kappa chain
c. V gene for lambda chain and V gene for heavy chain
d. C gene for gamma chain and C gene for alpha chain

48. Idiotypic determinants are located within


a. hypervariable regions of heavy and light chains.
b. constant regions of light chains.
c. constant regions of heavy chains.
d. the hinge region.

49. A primary immune response in an adult human requires approximately how much time to
produce detectable antibody levels in the blood?
a. 12 hours
b. 3 days
c. 1 week
d. 3 weeks

50. The membrane IgM and IgD on the surface of an individual B cell
a. have identical heavy chains but different light chains
b. are identical except for their CH regions
c. are identical except for their VH regions
d. have different VH and VL regions

51. During the maturation of a B lymphocyte, the first immunoglobulin heavy chain synthesized
is the
a. Mu chain.
b. gamma chain.
c. epsilon chain.
d. alpha chain.

52. In the immune response to a hapten-protein conjugate, in order to get anti-hapten antibodies
it is essential that
a. the hapten be recognized by helper T cells.
b. the protein be recognized by helper T cells.
c. the protein be recognized by B cells.
d. the hapten be recognized by suppressor T cells.

53. In the determination of serum insulin levels by radioimmunoassay, which one of the
following is NOT needed?
a. isotope-labeled insulin
b. anti-insulin antibody made in goats
c. anti-goat gamma globulin made in rabbits
d. isotope-labeled anti-insulin antibody made in goats

54. Which one of the following sequences is appropriate for testing a patient for antibody against
the AIDS virus with the ELISA procedure? (The assay is carried out in a plastic plate with an
incubation and a wash step after each addition except the final one.)
a. patient's serum/enzyme substrate/HIV antigen/enzyme-labeled antibody against HIV
b. HIV antigen/patient's serum/enzyme-labeled antibody against human gamma
globulin/enzyme substrate
c. enzyme-labeled antibody against human gamma globulin/patient's serum/HIV
antigen/enzyme substrate
d. enzyme-labeled antibody against HIV/HIV antigen/patient's serum/enzyme substrate

55. The BEST method to demonstrate IgG on the glomerular basement membrane in a kidney
tissue section is the
a. precipitin test.
b. complement fixation test.
c. agglutination test.
d. indirect fluorescent-antibody test.

56. In pancreatic carcinoma the ras gene:


a. Is absent.
b. Is normal but is overexpressed.
c. Has a large deletion.
d. Contains a single point mutation, always at the same position.
e. Contains a single point mutation, but not always at the same position.

57. A patient with a central nervous system disorder is maintained on the drug methyldopa.
Hemolytic anemia develops, which resolves shortly after the drug is withdrawn. This is MOST
probably an example of
a. atopic hypersensitivity.
b. cytotoxic hypersensitivity.
c. immune-complex hypersensitivity.
d. cell-mediated hypersensitivity.

58. Which one of the following substances is NOT released by activated helper T cells?
a. interleukin-1
b. gamma interferon
c. interleukin-2
d. interleukin-4

59. A delayed hypersensitivity reaction is characterized by


a. edema without a cellular infiltrate.
b. an infiltrate composed of neutrophils.
c. an infiltrate composed of helper T cells and macrophages
d. an infiltrate composed of eosinophils.

60. Two dissimilar inbred strains of mice, A and B, are crossed to yield an F1 hybrid strain, AB. If
a large dose of spleen cells from an adult A mouse is injected into an adult AB mouse, which
one of the following is MOST likely to occur?
a. The spleen cells will be destroyed.
b. The spleen cells will survive and will have no effect in the recipient.
c. The spleen cells will induce a graft-versus-host reaction in the recipient.
d. The spleen cells will survive and induce tolerance of strain A grafts in the recipient.

61. This question is based on the same strains of mice described in the previous question. If
adult AB spleen cells are injected into a newborn B mouse, which one of the following is
MOST likely to occur?
a. The spleen cells will be destroyed.
b. The spleen cells will survive without any effect on the recipient.
c. The spleen cells will induce a graft-versus-host reaction in the recipient.
d. The spleen cells will survive and induce tolerance of strain A grafts in the recipient.

62. The minor histocompatibility antigens on cells


a. are detected by reaction with antibodies and complement.
b. are controlled by several genes in the major histocompatibility complex.
c. are unimportant in human transplantation.
d. induce reactions that can cumulatively lead to a strong rejection response.

63. Which one of the following is NOT true of class I MHC antigens?
a. They can be assayed by a cytotoxic test that uses antibody and complement.
b. They can usually be identified in the laboratory in a few hours.
c. They are controlled by at least three gene loci in the major histocompatibility complex.
d. They are found mainly on B cells, macrophages, and activated T cells.

64. An antigen found in relatively high concentration in the plasma of normal fetuses and a high
proportion of patients with progressive carcinoma of the colon is
a. viral antigen.
b. carcinoembryonic antigen.
c. alpha-fetoprotein.
d. Heterophil

65. An antibody directed against the idiotypic determinants of a human IgG antibody would react
with
a. the Fc part of the IgG.
b. an IgM antibody produced by the same plasma cell that produced the IgG.
c. all human kappa chains.
d. all human gamma chains.

66. Which one of the following is NOT true of the gene segments that combine to make up a
heavy-chain gene?
a. Many V region segments are available.
b. Several J segments and several D segments are available.
c. V, D, and J segments combine to encode the antigen-binding site.
d. A V segment and a J segment are preselected by an antigen to make up the variable-
region portion of the gene.

67. When immune complexes from the serum are deposited on glomerular basement membrane,
damage to the membrane is caused mainly by
a. gamma interferon.
b. phagocytosis.
c. cytotoxic T cells.
d. enzymes released by polymorphonuclear cells.

68. If an individual was genetically unable to make J chains, which immunoglobulin(s) would be
affected?
a. IgG
b. IgM
c. IgA
d. IgG and IgM
e. IgM and IgA

69. The antibody-binding site is formed primarily by


a. the constant regions of H and L chains.
b. the hypervariable regions of H and L chains.
c. the hypervariable regions of H chains.
d. the variable regions of H chains.
e. the variable regions of L chains.

70. The class of immunoglobulin present in highest concentration in the blood of a human
newborn is
a. IgG.
b. IgM.
c. IgA.
d. IgD.
e. IgE.

71. Individuals of blood group type AB


a. are Rh(d)-negative.
b. are "universal recipients" of transfusions.
c. have circulating anti-A and anti-B antibodies.
d. have the same haplotype.

72. Cytotoxic T cells induced by infection with virus A will kill target cells
a. from the same host infected with any virus.
b. infected by virus A and identical at class I MHC loci of the cytotoxic T cells.
c. infected by virus A and identical at class II MHC loci of the cytotoxic T cells.
d. infected with a different virus and identical at class I MHC loci of the cytotoxic cells.
e. infected with a different virus and identical at class II MHC loci of the cytotoxic cells.

73. Antigen-presenting cells that activate helper T cells must express which one of the following
on their surfaces?
a. IgE
b. gamma interferon
c. class I MHC antigens
d. class II MHC antigens

74. Which one of the following does NOT contain C3b?


a. classic-pathway C5 convertase
b. alternative-pathway C5 convertase
c. classic-pathway C3 convertase
d. alternative-pathway C3 convertase

75. Which one of the following is NOT true regarding the alternative complement pathway?
a. It can be triggered by infectious agents in absence of antibody.
b. It does not require C1, C2, or C4.
c. It cannot be initiated unless C3b fragments are already present.
d. It has the same terminal sequence of events as the classic pathway.

76. In setting up a complement fixation test for antibody, the reactants should be added in what
sequence? (Ag = antigen; Ab = antibody; C = complement; EA = antibody-coated indicator
erythrocytes.)
a. Ag + EA + C/wait/ + patient's serum
b. C + patient's serum + EA/wait/ + Ag
c. Ag + patient's serum + EA/wait/ + C
d. Ag + patient's serum + C/wait/ + EA

77. Proteins from two samples of animal blood, A and B, were tested by the double-diffusion
(Ouchterlony) test in agar against antibody to bovine albumin. Which sample(s) contain
horse blood?
a. sample A
b. sample B
c. both samples
d. neither sample

78. Complement lyses cells by


a. enzymatic digestion of the cell membrane.
b. activation of adenylate cyclase.
c. insertion of complement proteins into the cell membrane.
d. inhibition of elongation factor 2.
79. Graft and tumor rejection are mediated primarily by
a. non-complement-fixing antibodies.
b. phagocytic cells.
c. helper T cells.
d. cytotoxic T cells.

80. Which one of the following properties of antibodies is NOT dependent on the structure of the
heavy-chain constant region?
a. ability to cross the placenta
b. isotype (class)
c. ability to fix complement
d. affinity for antigen

81. In which one of the following situations would a graft-versus-host reaction be MOST likely to
occur? (Mouse strains A and B are highly inbred; AB is an F1 hybrid between strain A and
strain B.)
a. newborn strain A spleen cells injected into a strain B adult
b. x-irradiated adult strain A spleen cells injected into a strain B adult
c. adult strain A spleen cells injected into an x-irradiated strain AB adult
d. adult strain AB spleen cells injected into a strain A newborn

82. In a mixed-lymphocyte culture, lymphocytes from person X, who is homozygous for the HLA-
Dw7 allele, are irradiated and then cultured with lymphocytes from person Z. It is found that
DNA synthesis is NOT stimulated. The proper conclusion to be drawn is that
a. person Z is homozygous for HLA-Dw7.
b. person Z is homozygous or heterozygous for HLA-Dw7.
c. person Z is heterozygous for HLA-Dw7.
d. person Z does not carry the HLA-Dw7 allele.

83. A patient skin-tested with purified protein derivative (PPD) to determine previous exposure
to Mycobacterium tuberculosis develops induration at the skin test site 48 hours later.
Histologically, the reaction site would MOST probably show
a. eosinophils.
b. neutrophils.
c. helper T cells and macrophages.
d. B cells.

84. Hemolytic disease of the newborn caused by Rh blood group incompatibility requires
maternal antibody to enter the fetal bloodstream. Therefore, the mediator of this disease is
a. IgE antibody.
b. IgG antibody.
c. IgM antibody.
d. IgA antibody.

85. An Rh-negative woman married to a heterozygous Rh-positive man has three children. The
probability that all three of their children are Rh-positive is
a. 1:2.
b. 1:4.
c. 1:8.
d. zero.

86. Which one of the following statements BEST explains the relationship between inflammation
of the heart (carditis) and infection with group A beta-hemolytic streptococci?
a. Streptococcal antigens induce antibodies cross-reactive with heart tissue.
b. Streptococci are polyclonal activators of B cells.
c. Streptococcal antigens bind to IgE on the surface of heart tissue and histamine is
released.
d. Streptococci are ingested by neutrophils that release proteases that damage heart tissue.

87. Your patient became ill 10 days ago with a viral disease. Laboratory examination reveals that
the patient's antibodies against this virus have a high ratio of IgM to IgG. What is your
conclusion?
a. It is unlikely that the patient has encountered this organism previously.
b. The patient is predisposed to IgE-mediated hypersensitivity reactions.
c. The information given is irrelevant to previous antigen exposure.
d. It is likely that the patient has an autoimmune disease.

88. If you measure the ability of cytotoxic T cells from an HLA-B27 person to kill virus X-infected
target cells, which one of the following statements is CORRECT?
a. Any virus X-infected target cell will be killed.
b. Only virus X-infected cells of HLA-B27 type will be killed.
c. Any HLA-B27 cell will be killed.
d. No HLA-B27 cell will be killed.

89. You have a patient who makes autoantibodies against his own red blood cells, leading to
hemolysis. Which one of the following mechanisms is MOST likely to explain the hemolysis?
a. Perforins from cytotoxic T cells lyse the red cells.
b. Neutrophils release proteases that lyse the red cells.
c. Interleukin-2 binds to its receptor on the red cells, which results in lysis of the red cells.
d. Complement is activated, and membrane attack complexes lyse the red cells.

90. Your patient is a child who has no detectable T or B cells. This immunodeficiency is most
probably the result of a defect in
a. the thymus.
b. the bursal equivalent.
c. T cell-B cell interaction.
d. stem cells originating in the bone marrow.

91. The role of the macrophage during an antibody response is to


a. make antibody.
b. lyse virus-infected target cells.
c. activate cytotoxic T cells.
d. process antigen and present it.

92. The structural basis of blood group A and B antigen specificity is


a. a single terminal sugar residue.
b. a single terminal amino acid.
c. multiple differences in the carbohydrate portion.
d. multiple differences in the protein portion.
93. Complement can enhance phagocytosis because of the presence on macrophages and
neutrophils of receptors for
a. factor D.
b. C3b.
c. C6.
d. properdin.

94. The main advantage of passive immunization over active immunization is that
a. it can be administered orally.
b. it provides antibody more rapidly.
c. antibody persists for a longer period.
d. it contains primarily IgM.

95. On January 15, a patient developed an illness suggestive of influenza, which lasted 1 week.
On February 20, she had a similar illness. She had no influenza immunization during this
period. Her hemagglutination inhibition titer to influenza A virus was 10 on January 18, 40
on January 30, and 320 on February 20. Which one of the following is the MOST appropriate
interpretation?
a. The patient was ill with influenza A on January 15.
b. The patient was ill with influenza A on February 20.
c. The patient was not infected with influenza virus.
d. The patient has an autoimmune disease.

96. An individual who is heterozygous for Gm allotypes contains two allelic forms of IgG in
serum, but individual lymphocytes produce only one of the two forms. This phenomenon,
known as "allelic exclusion," is consistent with
a. a rearrangement of a heavy-chain gene on only one chromosome in a lymphocyte.
b. rearrangements of heavy-chain genes on both chromosomes in a lymphocyte.
c. a rearrangement of a light-chain gene on only one chromosome in a lymphocyte.
d. rearrangements of light-chain genes on both chromosomes in a lymphocyte.

97. Each of the following statements concerning class I MHC proteins is correct EXCEPT:
a. They are cell surface proteins on virtually all cells.
b. They are recognition elements for cytotoxic T cells.
c. They are codominantly expressed.
d. They are important in the skin test response to Mycobacterium tuberculosis.

98. Which one of the following is the BEST method of reducing the effect of graft-versus-host
disease in a bone marrow recipient?
a. matching the complement components of donor and recipient
b. administering alpha interferon
c. removing mature T cells from the graft
d. removing pre-B cells from the graft

99. Regarding Th-1 and Th-2 cells, which one of the following is LEAST accurate?
a. Th-1 cells produce gamma interferon and promote cell-mediated immunity.
b. Th-2 cells produce interleukin-4 and -5 and promote antibody-mediated immunity.
c. Both Th-1 and Th-2 cells have both CD3 and CD4 proteins on their outer cell membrane.
d. Before nave Th cells differentiate into Th-1 or Th-2 cells, they are double-positives; i.e.,
they produce both gamma interferon and interleukin-4.

100. Each of the following statements concerning the variable regions of heavy chains and the
variable regions of light chains in a given antibody molecule is correct EXCEPT:
a. They have the same amino acid sequence.
b. They define the specificity for antigen.
c. They are encoded on different chromosomes.
d. They contain the hypervariable regions.

101. Each of the following statements concerning class II MHC proteins is correct EXCEPT:
a. They are found on the surface of both B and T cells.
b. They have a high degree of polymorphism.
c. They are involved in the presentation of antigen by macrophages.
d. They have a binding site for CD4 proteins.

102. Which one of the following statements concerning immunoglobulin allotypes is CORRECT?
a. Allotypes are found only on heavy chains.
b. Allotypes are determined by class I MHC genes.
c. Allotypes are confined to the variable regions.
d. Allotypes are due to genetic polymorphism within a species.

103. Each of the following statements concerning immunologic tolerance is correct EXCEPT:
a. Tolerance is not antigen-specific; i.e., paralysis of the immune cells results in a failure to
produce a response against many antigens.
b. Tolerance is more easily induced in T cells than in B cells.
c. Tolerance is more easily induced in neonates than in adults.
d. Tolerance is more easily induced by simple molecules than by complex ones.

104. Each of the following statements concerning a hybridoma cell is correct EXCEPT:
a. The spleen cell component provides the ability to form antibody.
b. The myeloma cell component provides the ability to grow indefinitely.
c. The antibody produced by a hybridoma cell is IgM, because heavy-chain switching does
not occur.
d. The antibody produced by a hybridoma cell is homogeneous; i.e., it is directed against a
single epitope.

105. Each of the following statements concerning haptens is correct EXCEPT:


a. A hapten can combine with (bind to) an antibody.
b. A hapten cannot induce an antibody by itself; rather, it must be bound to a carrier protein
to be able to induce antibody.
c. In both penicillin-induced anaphylaxis and poison ivy, the allergens are haptens.
d. Haptens must be processed by CD8+ cells to become immunogenic.

106. Which of the following is a non-organ-specific (systemic) autoimmune disease:


a. Myasthenia gravis.
b. Systemic lupus erythematosus (SLE).
c. Hashimoto's thyroiditis.
d. Pernicious anemia.
e. Insulin-dependent diabetes mellitus
107. Which of the following antibodies are of most use for the diagnosis of pernicious anemia:
a. Anti-parietal cell.
b. Anti-thyroid peroxidase.
c. Anti-nuclear.
d. Anti-IgG Fc
e. Anti-TSH receptor.

108. The disease most frequently seen in association with pernicious anemia is;
a. Addison's disease of the adrenal.
b. Multiple sclerosis.
c. Autoimmune hemolytic anemia.
d. Rheumatoid arthritis.
e. Graves' disease.
109. The high concordance rate for monozygotic vs dizygotic twins in type 1 diabetes indicates:
a. A strong environmental element.
b. A strong genetic element.
c. A major influence of sex.
d. The influence of HLA.

110. The undue tendency for closely linked genes on a chromosome to remain associated rather
than undergo genetic randomisation, is termed:
a. Tandem duplication.
b. Meiotic crossover.
c. Relative risk.
d. Linkage disequilibrium.
e. Gene conversion

111. HLA-DR2 is a risk factor for:


a. Multiple sclerosis.
b. Insulin-dependent (type I) diabetes.
c. Ankylosing spondylitis.
d. Rheumatoid arthritis.
e. Myasthenia gravis.

112. HLA-DR2 is a risk factor for:


a. Multiple sclerosis.
b. Insulin-dependent (type I) diabetes.
c. Ankylosing spondylitis.
d. Rheumatoid arthritis.
e. Myasthenia gravis.

113. Cryptic T-cell epitopes capable of triggering autoimmune disease:


a. Dominate the normal immune response.
b. Negatively select T-cells.
c. Are unable to bind in the MHC groove.
d. Can be derived by antigen processing.
e. Are presented by MHC class I but not MHC class II molecules.

114. Which of the following is incorrect 'Natural antibodies' are often:


a. Autoreactive.
b. Polyspecific.
c. Reactive with bacterial carbohydrates.
d. High affinity IgG.
e. Produced by CD5+ B-cells.
115. An example of a known oncogenic virus is:
a. Herpes zoster.
b. HIV-2.
c. Epstein-Barr virus.
d. Vesicular stomatitis virus.
e. Proteus mirabilis.

116. Boon and colleagues showed that syngeneic transplantable tumors which mutate such that
they express strong transplantation antigens are rejected. They called these variants:
a. tum .
b. Xenogeneic.
c. tum + .
d. MCA.
e. Non-immunogenic.

117. Which of the following does not involve cell-mediated immunity?


a. contact sensitivity to lipstick
b. rejection of a liver graft
c. serum sickness
d. tuberculin reaction
e. immunity to chicken pox

118. A positive delayed-type hypersensitivity skin reaction involves the interaction of


a. antigen, complement, and cytokines.
b. antigen, antigen-sensitive lymphocytes, and macrophages.
c. antigen antibody complexes, complement, and neutrophils.
d. IgE antibody, antigen, and mast cells.
e. antigen, macrophages, and complement.

119. Cell-mediated immune responses are


a. enhanced by depletion of complement.
b. suppressed by cortisone.
c. Enhanced by depletion of T cells.
d. suppressed by antihistamine.
e. enhanced by depletion of macrophages.

120. Delayed skin reactions to an intradermal injection of antigen may be markedly decreased by
a. exposure to a high dose of X-irradiation.
b. treatment with antihistamines.
c. treatment with an antineutrophil serum.
d. removal of the spleen.
e. decreasing levels of complement.

121. Patients with DiGeorge syndrome who survive beyond infancy would be capable of
a. rejecting a bone marrow transplant.
b. mounting a delayed-type hypersensitivity response to dinitrochlorobenzene.
c. resisting intracellular parasites.
d. forming antibody to T-dependent antigens.
e. All of the above.
f. None of the above.

122. Which of the following statements is characteristic of contact sensitivity?


a. The best therapy is oral administration of the antigen.
b. Patch testing with the allergen is useless for diagnosis.
c. Sensitization can be passively transferred with serum from an allergic individual.
d. Some chemicals acting as haptens induce sensitivity by covalently binding to host
proteins acting as carriers.
e. Antihistamines constitute the treatment of choice.

123. Positive skin tests for delayed-type hypersensitivity to intradermally injected antigens
indicate that
a. a humoral immune response has occurred.
b. a cell-mediated immune response has occurred.
c. both T cell and B cell systems are functional.
d. the individual has previously made IgE responses to the antigen.
e. immune complexes have been formed at the injection site.

124. T cell-mediated immune responses can result in


a. formation of granulomas.
b. induration at the reaction site.
c. rejection of a heart transplant.
d. eczema of the skin in the area of prolonged contact with a rubberized undergarment.
e. All of the above.

125. Which of the following do not apply to CD4+ lymphocytes?


a. they account for two thirds of peripheral lymphocytes in blood and lymphoid organs.
b. they may have regulatory (suppressor) functions
c. they secrete IL2
d. they may be cytotoxic

126. Which of the following is correct regarding IL12?


a. Induces the formation of Th1 lymphocytes
b. Induces the formation of Th2 lymphocytes
c. Suppresses the formation of Th1 lymphocytes
d. Suppresses the formation of Th2 lymphocytes
e. Stimulates macrophages

127. Which of the following cells do not originate in the bone marrow?
a. T lymphocytes
b. B lymphocytes
c. Macrophages
d. follicular dendritic cells

128. Macrophages have Fc receptors for all of the following antibodies except?
a. IgM
b. IgG1
c. IgA
d. IgE

129. Of the following, which membrane proteins do not belong to the Ig superfamily?
a. IgM
b. ICAM 1
c. LFA1
d. Fc receptor
e. MHC Class I

130. In acute rejection of solid-organ transplants (e.g. kidney, liver, heart), graft damage is caused
a. due to pre-formed antibody
b. by recognition of donor peptides by cytotoxic T cells (CD8+)
c. by helper T cells (CD4+)
d. by transplant atherosclerosis

131. Immunoglobulins of M class (IgM)


a. cross the placenta
b. are characteristically produced in a secondary immune response
c. are found as a dimeric form linked by a J chain
d. can activate complement
e. are usually found on the surface of mast cells

132. All of the following statements regarding classical pathway of complement activation are
true except:
a. It starts with activation of C1q complement
b. It is activated by IgA immune complexes
c. It is activated by IgG which has bound to specific antigen
d. It is activated by IgM which is bound to specific antigen

133. The human major histocompatibility complex (correct statement is)


a. is situated on chromosome 4
b. is also known as the HLA complex
c. codes for blood group antigens
d. codes for two classes of antigens which are expressed on all nucleated cells

134. Hyperacute rejection of grafts (correct statement is)


a. occurs within 2 -4 days after transplantation
b. is a cell-mediated response
c. never occurs in autografts
d. may be reversed by cyclosporine

135. All the statements regarding Cortical epithelial cells in the thymus are true except:
a.
b. They express MHC molecules
c. They are responsible for positive selection of T cells
d. They are responsible for negative selection of T cells
e. They form Hassals corpuscles in the medulla when they become senescent
136. In type III (immune complex mediated) hypersensitivity tissue damage is mediated by
a. histamine
b. neutrophil activation
c. macrophage activation
d. T cell activation
e. NK cell activation

137. Which of the following is true of schistosomiasis?


a. is a liver fluke
b. causes rectal cancer
c. parasites are susceptible to IgE antibody
d. parasites are hermaphrodite

138. Infection by Herpes simplex virus is true for all following statements except
a. occurs via aerosol transmission
b. results in lifelong latent infection
c. can be treated with acyclovir
d. is controlled by mass vaccination

139. Live attenuated virus vaccines


a. are produced by inactivating virus particles using chemical or physical treatments
b. are used to immunise against measles
c. are used to immunise against hepatitis B
d. are used solely to protect the vaccinee from infection
e. do not induce a cytotoxic T-cell response

140. Which bacteria cannot survive inside phagocytes?


a. Streptococcus pneumoniae
b. Mycobacterium tuberculosis
c. Yersinia enterocolitica
d. Salmonella typhi

141. Survival of the host from Salmonella typhi infection (typhoid) depends upon:
a. IgA
b. Complement
c. Th1 lymphocytes
d. Th2 lymphocytes
e. IgM

142. Which of the follwing statements about physical and chemical defences is false?
a. Lysozyme has potent antibacterial activities
b. The skin is an effective barrier to microbial entry into deeper tissues
c. The high pH of the vagina inhibits microbial growth
d. The protective effects of mucus are limited to respiratory tract
e. The sebaceous glands of the skin produce antimicrobial fatty acids

143. What is true regarding antibiotic resistance in bacteria


a. Is always caused by degradation of the drug by bacteria
b. Is found only in Gram positive bacteria
c. Can be transferred between bacteria by conjugation
d. Is only encoded by plasmids
e. Is not transferred between bacterial species

144. Which of the following does not apply to "innate" immune mechanisms?
a. absence of specificity
b. activation by a stimulus
c. involvement of multiple cell types
d. a memory component

145. Removal of the bursa of Fabricius from a chicken results in


a. a markedly decreased number of circulating T lymphocytes.
b. anemia.
c. delayed rejection of skin graft.
d. low serum levels of antibodies in serum.
e. all of the above.
f. none of the above.

146. The germinal centers of lymph nodes


a. support the development of immature B and T cells.
b. function in the removal of damaged erythrocytes from the circulation.
c. act as the major source of stem cells and thus help to maintain hematopoiesis.
d. provide an infrastructure that on antigenic stimulation contains large populations of B
lymphocytes and plasma cells.
e. are the sites of NK-cell differentiation.

147. Which of the following statements is correct?


a. NK cells proliferate in response to antigen.
b. NK cells kill their target cells by phagocytosis and intracellular digestion.
c. NK cells are a subset of polymorphonuclear cells.
d. NK-cell killing is extracellular.
e. NK cells are particularly effective against certain bacteria

148. Converting a toxin to a toxoid


a. makes the toxin more immunogenic.
b. reduces the pharmacologic activity of the toxin.
c. enhances binding with antitoxin.
d. induces only innate immunity.
e. increases phagocytosis.

149. An adjuvant
a. reduces the toxicity of the immunogen.
b. enhances the immunogenicity of haptens.
c. enhances hematopoiesis.
d. enhances the immune response against the immunogen.
e. enchances immunologic cross-reactivity.

150. The idiotype of an antibody molecule is determined by the amino acid sequence of the
a. constant region of the L chain.
b. variable region of the L chain.
c. constant region of the H chain.
d. constant regions of the H and L chains.
e. variable regions of the H and L chains.

151. An individual was found to be heterologous for IgG1 allotypes IgG1m1 and IgG1m2. The
different possible IgG1 antibodies produced by this individual will never have
a. two H chains of allotype IgG1m1
b. two L chains of either or
c. two H chains of same allotype
d. two H chains, one of allotype IgG1m1 and one of allotype IgG1m2

152. Concentration of specific IgM antibodies can be of diagnostic significance because


a. IgM is easier to detect than the other isotypes.
b. viral infection often results in very high IgM responses.
c. IgM antibodies are more often protective against reinfections than are the other isotypes.
d. relatively high levels of IgM often correlate with a first recent exposure to the inducing
agent.

153. Serum from a patient is analyzed by serum protein electrophoresis. The gamma globulin
peak is twenty times lower than that of a normal healthy individual.
a. this result is normal because not all people have similar levels of IgG.
b. this result may indicate a B cell immunodeficiency
c. this patient probably has an autoimmune disease
d. the patient may have a myeloma protein

154. SCID mice have a genetic defect that prevents development of functional
a. hematopoietic cells.
b. B cells and T cells.
c. T cells and NK cells.
d. pluripotential stem cells.
e. myeloid cells.

155. Which of the following concerning Ig expression on a B cell is incorrect


a. The light chains of the IgM and IgD have identical amino acid sequences.
b. The constant parts of the heavy chains of the IgM and IgD have different amino acid
sequences.
c. The IgM and IgD have different antigenic specificities.
d. If the B cell is triggered by antigen and T-cell signals to proliferate and differentiate into
antibody secreting plasma cells, the cell can potentially secrete IgG, IgE, or IgA antibody.
e. The IgM on the surface will have either k light chains or l light chains, but not both.

156. Which of the following plays a role in changing the antigen binding site of a B cell after
antigenic stimulation?:
a. junctional diversity
b. combinatorial diversity
c. germ-line diversity
d. somatic hypermutation
e. differential splicing of primary RNA transcripts

157. In B cell development, heavy chain is expressed on the cell surface prior to light chain
rearrangement.
a. invariant chain
b. surrogate light chains
c. CLIP
d. b-2 microglobulin
e. it is expressed by itself

158. All the following are characteristics of both MHC class I and class II molecules except:
a. They are expressed codominantly.
b. They are expressed constitutively on all nucleated cells.
c. They are glycosylated polypeptides with domain structure.
d. They are involved in presentation of antigen fragments to T cells.
e. They are expressed on the surface membrane of B cells.

159. MHC class I molecules are important for which of the following?
a. binding to CD8 molecules on T cells
b. presenting exogenous antigen (e.g., bacterial protein) to B cells
c. presenting viral protein to antigen-presenting cells such as macrophages
d. binding to CD4 molecules on T cells
e. binding to Ig on B cells

160. When the sequences of different MHC class I molecules are compared, the variation between
molecules is concentrated within which of the following?
a. Areas of the molecule that bind CD4
b. Areas of the molecule that bind to the T cell receptor and to the antigenic peptide
c. -2 microglobulin
d. the transmembrane domain of the a chain
e. the -1 domain (the N terminal domain of the chain)

161. Which of the following statements is incorrect concerning TCR and Ig genes?
a. In both B- and T-cell precursors, multiple V-, D-, J-, and C-region genes exist in an
unrearranged configuration.
b. Rearrangement of both TCR and Ig genes involves specific recombinase enzymes that
bind to specific regions of the genome.
c. Both Ig and TCR are able to switch C-region usage.
d. Both Ig and TCR exhibit allelic exclusion.
e. Both Ig and the TCR use combinatorial association of V, D, and J genes and junctional
imprecision to generate diversity.

162. Which of the following statements is incorrect concerning antigen-specific receptors on


both B and T cells?
a. They are clonally distributed transmembrane molecules.
b. They have extensive cytoplasmic domains that interact with intracellular molecules.
c. They consist of polypeptides with variable and constant regions.
d. They are associated with signal transduction molecules at the cell surface.
e. They can interact with peptides derived from nonself antigens.

163. The development of self-tolerance in the T-cell compartment is important for the prevention
of autoimmunity. Which of the following results in T-cell self-tolerance?
a. allelic exclusion
b. somatic hypermutation
c. thymocyte proliferation
d. positive selection
e. negative selection

164. Bacterial lipopolysaccharide (LPS), a T-independent antigen, stimulates antibody


production in mice. Which of the following is incorrect?
a. The antibody produced will be predominantly IgM.
b. Memory B cells will not be induced.
c. IL-4 and IL-5 are required for the production of antibody during the response.
d. The polymeric nature of the antigen crosslinks B-cell surface receptors.
e. B cell activation involves phosphorylation of intracellular molecules.

165. Which of the following is incorrect concerning immune tolerance?


a. Tolerance induction is antigen-specific.
b. Tolerance results from inactivation and/or elimination of B and/or T cells.
c. Tolerance can be induced in both young and old individuals.
d. Immature neutrophils are more susceptible to tolerance than mature neutrophils.
e. The breakdown of tolerance can result in autoimmunity.

166. Which of the following statements is incorrect concerning the immune response to
antigens?
a. Reactivity is influenced by extremes of age.
b. Greater immune responses are produced when antigen is given with adjuvant.
c. Impaired nutrition depresses immunity.
d. The presence of preexisting antibody does not affect the subsequent response to antigen.
e. Different protein antigens stimulate different levels of antibody production.

167. Which one of the following factors is NOT indicative of renal graft rejection?
a. Fever
b. Increase in serum IgG level
c. Tenderness of the graft
d. Drop in urine volume
e. Rise in creatinine

168. Which of the following is associated with the development of systemic lupus erythematosus
(SLE)?
a. deficiencies in C1, C4, or C2
b. deficiencies in C5, C6, or C7
c. deficiencies in the late components of complement
d. increases in the serum C3 level
e. increases in the levels of C1, C4, or C2

169. Decay-accelerating factor (DAF) regulates the complement system to prevent complement-
mediated lysis of cells. This involves
a. dissociation of C4b2a or the C3bBb enzyme complex.
b. blocking the binding of C3 convertase to the surface of bacterial cells.
c. inhibiting the membrane attack complex from binding to bacterial membranes.
d. acting as a cofactor for the cleavage of C3b.
e. causing dissociation of C5 convertase.
170. Serum sickness occurs only
a. when anti-basement membrane antibodies are present.
b. in cases of extreme excess of antibody.
c. when IgE antibody is produced.
d. when soluble immune complexes are formed.
e. in the absence of neutrophils.

171. Serum sickness is characterized by


a. deposition of immune complexes in blood vessel walls when there is a moderate excess
of antigen.
b. phagocytosis of complexes by granulocytes.
c. consumption of complement.
d. appearance of symptoms before free antibody can be detected in the circulation.
e. All of the above.

172. Type II hypersensitivity


a. is antibody-independent.
b. is complement-independent.
c. is mediated by CD8+ T cells.
d. requires immune complex formation.
e. involves antibody-mediated destruction of cells.

173. Identifying an autoimmune disease in humans is often accomplished by


a. finding an antibody against self-components.
b. passively transferring specific T cells from a patient to a healthy individual.
c. showing that T cells or antibodies are the cause of the tissue damage.
d. circumstantial evidence, such as MHC association and clinical improvement, with
immunosuppressive drugs.
e. finding the definitive agent or agents responsible for the disease.

174. Rheumatoid factor, found in synovial fluid of patients with rheumatoid arthritis, is most
frequently found to be
a. IgM reacting with L chains of IgG.
b. IgM reacting with H-chain determinants of IgG.
c. IgE reacting with bacterial antigens.
d. antibody to collagen.
e. antibody to DNA.

175. Hashimoto's thyroiditis


a. is due primarily to antibodies formed to thyroid-stimulating hormone receptors.
b. mimics an animal model in which the disease is induced by immunization with
thyroglobulin.
c. can be transplacentally transmitted causing a neonatal form of the disease.
d. is an autoimmune disease which affects males and females equally.
e. is characterized by immune complex deposition in the thyroid.
176. The most common serologic test used for the detection of HLA antigens on lymphocytes is
a. the complement fixation test.
b. double gel diffusion.
c. complement-dependent cytotoxicity test.
d. mixed lymphocyte reaction.
e. radioimmunoassay.

177. Tumor antigens have been shown to cross-react immunologically in cases of


a. tumors induced by chemical carcinogens.
b. tumors induced by RNA viruses.
c. all tumors.
d. tumors induced by irradiation with ultraviolet light.
e. tumors induced by the same chemical carcinogen on two separate sites on the same
individual.

178. Immunotoxins are


a. toxic substances released by macrophages.
b. cytokines.
c. toxins completed with the corresponding antitoxins.
d. toxins coupled to antigen-specific immunoglobulins.
e. toxins released by cytotoxic T cells.

179. Which of the following is associated with passive immunity?


a. exposure to an antigen
b. infusion of weakened viruses
c. passage of IgG antibodies from a pregnant mother to her fetus
d. booster shot of vaccine

180. Which of the following is not a function of the inflammatory response?


a. prevents the spread of the injurious agent to nearby tissue
b. replaces injured tissues with connective tissue
c. disposes of cellular debris and pathogens
d. sets the stage for repair processes

181. Which of the following statements regarding NK cells is a false or incorrect statement?
a. NK cells are a type of neutrophil.
b. NK cells are present in the blood, spleen, lymph nodes, and red bone marrow.
c. NK cells attack cells that display abnormal MHC antigens.
d. NK cells attack cancer cells and virus-infected body cells.

182. Which of the following statements is incorrect or false?


a. Haptens lack immunogenicity unless attached to protein carriers.
b. Class 1 MHC molecules are built into the plasma membranes of all body cells.
c. Class II MHC molecules appear only on the surface of antigen-presenting cells, thymic
cells, and T cells that have been activated by exposure to antigens.
d. MHC proteins are the cells identity markers.

183. Which of the following statements is a false or incorrect statement?


a. The lymphoid organs where lymphocytes become immunocompetent are called primary
lymph organs. All other lymphoid organs are referred to as secondary lymphoid organs.
b. It is our genes, not antigens, that determine what specific foreign substances our immune
system will be able to recognize and resist.
c. After becoming immunocompetent, the naive T cells and B cells are exported to the bone
marrow where the encounters with antigens occur.
d. T cells and B cells become fully immunocompetent when they bind with recognized
antigens.

184. Cells of the innate immune system are specialized. Although several of them can
ingest microbes, one cell type is found both undifferentiated in the blood and
resident in tissues where they work as janitor cells eating particulate matter. What cell type
best fit this description?
a. Macrophages
b. Neutrophils
c. Natural Killer Cells
d. Dendritic Cells
e. Granulocytes

185. The internal immune system is mainly enforced by leukocytes, the leukocytes have different
half-lives and are constantly generated, about 1011 each day in a human adult. Where are they
generated?
a. All leukocytes originate from the bone marrow
b. All leukocytes except T cells originate from the bone marrow
c. B cells originates from the bone marrow, T cells from the thymus and innate immune
cells are formed throughout the body
d. Leukocytes originate from most tissues in the adult body
e. Innate immune cells originate from the bone marrow, lymphocytes from lymphoid tissue

186. An essential part of the innate immune system is phagocytosis followed by destruction of
the phagocytosed particle/microbe followed by antigen- presentation. What cells are
considered professional phagocytes?
a. Granulocytes, monocytes, macrophages, dendritic cells
b. Granulocytes, monocytes, macrophages, dendritic cells, B cells
c. Neutrophils, dendritic cells, monocytes, macrophages, mast cells
d. Antigen presenting cells
e. All innate immune cells

187. Which one of the following statements regarding graft-versus-host disease (GVHD) is NOT
true?
a. Occurs in only a minority of patients receiving allogeneic bone marrow transplants.
b. May occur mildly following bone marrow transplantation between HLA-identical siblings.
c. May occur following unirradiated blood transfusion in babies with severe combined
immune deficiency.
d. Is preventable by depleting T cells from donor bone marrow.
e. Clinically involves the skin, liver and intestine.

188. Natural Killer (NK) cells are innate immune cells with important functions against viral
infections. What is the main mechanism of recognition of infected cells?
a. Viral epitopes presented by non-classical MHC molecules
b. Through by-stander effects initiated by neighbouring activated dendritic cells
c. By cross-presentation of viral epitopes by follicular dendritic cells in the lymph nodes
d. By specific anti-viral antibodies on the infected cell surface
e. By binding of viral epitopes to Toll-like receptors (TLRs) on the NK cell

189. Which one of the following statements is true?


a. The thymus controls peripheral tolerance of T cells.
b. Self-reactive T cells are not found in healthy normal subjects.
c. Naive T cells need more than one signal in order to become activated.
d. B cell tolerance is more important than T-cell tolerance in the prevention of
autoimmunity.

190. Which one of the following statements about rheumatic fever is true?
a. It is a complication of beta-haemolytic streptococcal infection.
b. It only affects the elderly.
c. It complicates 3% of acute sore throats.
d. It shows no evidence of genetic susceptibility.
e. It is caused by immediate hypersensitivity to streptococci.

191. The most frequently diagnosed form of specific primary immunodeficiency is:
a. Severe combined immunodeficiency
b. X-linked agammaglobulinaemia
c. Chronic granulomatous disease
d. Selective IgA deficiency
e. DiGeorge anomaly

192. Which of the following is most pertinent to healthy individuals with selective IgA deficiency?
a. All individuals can be considered to be normal without risk of infections.
b. They are more likely to develop heart disease than those with normal IgA levels.
c. Will always have a high risk of HIV infection
d. They have a high risk of a recurrent serious bacterial infection.
e. They have a higher risk of developing an organ specific autoimmune disease than the
general population.

193. Transient hypogammaglobulinaemia of infancy usually:


a. Occurs at 3 months of age.
b. Is more pronounced in premature babies.
c. Is due to placental absorption of IgG.
d. Is linked with autoimmune disease in later life.
e. Involves only the IgA immunoglobulin class.

194. Which of the following is the primary defect in chronic granulomatous disease?
a. Neutrophil production in the bone marrow.
b. Neutrophil chemotaxis.
c. Neutrophil intracellular killing of organisms.
d. Opsonization
e. Cytotoxic T-cell activity.

195. Which of the following is commonly associated with marked secondary antibody deficiency?
a. Mild protein-energy malnutrition.
b. Multiple myeloma.
c. Non-Hodgkin's lymphoma.
d. HIV infection.
e. Hodgkin's disease.
196. Which one of the following sets of antigens is NOT important in relation to successful renal
grafting?
a. MHC class II antigens
b. Lewis red cell antigens
c. Sex antigens (products of X and Y chromosomes)
d. ABO red cell antigens

197. Which of the following statements best characterizes an antibody?


a. An antibody contains high molecular weight RNA as its basic structure.
b. An antibody is composed of protein and cannot be distinguished from the albumin
fraction of the serum proteins.
c. An antibody is composed of four identical protein subunits which may be caused to
dissociate by treatment with urea.
d. An antibody contains protein as its major chemical component and its synthesis may be
elicited by the administration of a foreign protein or polysaccharide.
e. An antibody contains mucopolysaccharides as its major chemical component and the
synthesis of these may be elicited by the administration of a foreign protein or
polysaccharide.

Part C:

198. Toll-like receptor 4 is associated with responsiveness to LPS, an endotoxin that causes lethal
endotoxic shock. The mice deficient in Toll-like receptor 4 and BALB/c mice were injected
with Escherichia coli. In addition, some BALB/b mice were also injected with the same
bacteria alone or with anti-interleukin- 10 (IL-10) antibody. The mice resistant to the lethal
effect of the bacteria were:
a. BALB/b mice receiving the bacteria.
b. BALB/b mice receiving the bacteria and the anti-IL-10antibody
c. Mice deficient In Toll-like receptor
d. BALB/c mice receiving the bacteria.

199. Macrophages were collected from BALB/c mice, CD40- deficient mice, CD86 deficient mice
and ICAM-1-deficient mice. These macrophages were co-cultured with LCMV peptidespecific
T cells in presence of the LCMV peptide for three days. The cells were recovered and co-
cultured with BALB/c-derived macrophages in presence of the peptide. During the last
twelve hour of the co-culture, 3H-thymidine was added to the cultures. The cells were
harvested and 3H- thymidine incorporation was assessed. The highest incorporation was
observed in
a. BALB/c macrophage-T cell co-culture.
b. CD40-deficient macrophage-T cell co-culture.
c. CD86-deficient macrophage-T cell co-culture.
d. ICAM-1-deficient macrophage-T cell co-culture.

200. Monoclonal antibodies (mAb) can be potentially used as therapeutic agents. The major
advantage is that they can specifically target aberrant cells. However, there is a practical
difficulty. Monoclonals are raised in mouse and therefore it is expected that an immune
reaction will develop if these are injected into humans. It is therefore necessary to 'humanize'
monoclonal antibody by
a. expressing the genes for the mAb in cultured human cells and isolating the mAb from
these cells
b. replacing the Fv region of a mAb with one derived from a human IgG
c. replacing CL and CH regions of the Mab with that obtained from a human IgG.
d. taking a human IgG and replacing the CDRs by those derived from the mouse mAb

201. Polyclonal antibodies are raised against bovine serum albumin in rabbit. Subsequently IgG
in the antiserum is purified and digested with either pepsin or papain. Out of the following
possibilities, which one is correct?
a. Pepsin-digested antibodies cannot precipitate the antigen
b. Papain-digested antibodies cannot precipitate the antigen
c. Pepsin digestion will produce two Fab molecules
d. Pepsin-digested antibodies will lose all interchain disulfide bonds

202. Opsonisation of a bacterium is a process by which specific antibody binds with the surface
molecule of the bacteria. In an experimental condition, macrophages were infected with
either Wild Type (WT): Mycobacteria or with opsonised: Mycobacteria for 2 hrs at 37C.
Subsequently, cells were washed and further incubated for 24 hrs at 37C. Finally, bacterial
load in macrophages were determined by colony forming unit (CFU). Which of the following
observation is true?
a. WT:Mycobacteria inhibits its transport to the lysosomes and survive in macrophages.
b. Opsonised: Mycobacteria inhibits its transport to the lysosomes and survive in
macrophages.
c. WT: Mycobacteria are targeted to the lysosomes and killed in macrophages.
d. Opsonised: Mycobateria are targeted to the lysosomes and survive in macrophages

203. A novel vaccine (against malaria liver stage specific) has been developed by an investigator.
This happened to be the parasites unique cell surface protein molecule (Mp). This Mp when
injected in mice elicits humoral antibody response (IgG in nature) and can efficiently
neutralize sporozoites by inhibiting their binding to the liver cells. Upon a pre-clinical trial
this vaccine failed universally despite a high titre IgG response. Which one out of the choices
below is the correct answer of this failure?
a. The liver cell surface receptor to which Mp binds is changed.
b. The malaria parasite is successful in changing the epitope in Mp to which IgG binds
c. IgG molecules change its binding sites for Mp.
d. The affinity of Ig to Mp epitopes is reduced.

204. During receptor-mediated endocytosis, ligand first binds with cell surface receptor, then
traffic through Rab5 positive early endosomal compartment. Finally, it moves to the Lamp1
positive lysosomes via Rab7 positive late compartment. In order to understand the
trafficking of ligand A in epithelia cells, cells were allowed to internalize ligand A for various
period of times at 37C. Finally, cells were stained with anti-ligand antibody and probed with
secondary antibody labelled with Alexa-Red fluorescence dye. Same cells were also co-
stained with anti-Rab5, anti-Rab 7 or anti-Lamp1 antibody and probed with appropriate
secondary antibody labelled with Alexa-green fluorescence dyes. Cells were viewed in
confocal microscope and observations are (I) 5 min internalize ligand (Red) in cells are
colocalize with anti Rab5 antibody but not with anti-Lamp1 antibody and (II) 90 min
internalized ligand are colocalized with anti-Lamp1 antibody but not with anti-Rab5
antibody. The following conclusions could be arrived at from the above observations.
(A) Ligand A travels to early endosomal compartment by 5 min.
(B) Ligand A travels to lysozomal compartment by about 90 min.
(C) Ligand A travels to lysozome by about 5 min.
(D) Ligand A travels to early endosome by about 90 min.
Identify the correct inferences.
a. (A) and (B)
b. (B) and (D)
c. (C) and (D)
d. (D) and (C)

205. A patient with rheumatic fever develops a sore throat from which beta-hemolytic
streptococci are cultured. The patient is started on treatment with penicillin, and the sore
throat resolves within several days. However, 7 days after initiation of penicillin therapy the
patient develops a fever of 103F, a generalized rash, and proteinuria. This MOST probably
resulted from
a. recurrence of the rheumatic fever.
b. a different infectious disease.
c. an IgE response to penicillin.
d. an IgG-IgM response to penicillin.
e. a delayed hypersensitivity reaction to penicillin.

206. A woman had a high fever, hypotension, and a diffuse macular rash. When all cultures
showed no bacterial growth, a diagnosis of toxic shock syndrome was made. Regarding the
mechanism by which the toxin causes this disease, which one of the following is LEAST
accurate?
a. The toxin is not processed within the macrophage.
b. The toxin binds to both the class II MHC protein and the T cell receptor.
c. The toxin activates many CD4-positive T cells, and large amounts of interleukins are
released.
d. The toxin has an A-B subunit structure--the B subunit binds to a receptor, and the A
subunit enters the cells and activates them.

207. A lymph node biopsy shows multiple necrotizing ("caseous") granulomas. A culture of
material obtained at the time of biopsy grows Mycobacterium tuberculosis. Which substance
produced locally is the principal activator or stimulator of the granuloma macrophages?
a. Bradykinin
b. interferon-
c. leukotriene B4
d. prostaglandin E2
e. the comlpement trimolecular complex (C567)

208. During the induction of type I (immediate) hypersensitivity response, which of the following
cells secrete cytokines that stimulate IgE production by B cells, promotes mast cell growth,
recruits and activates mast cells?
a. CD4+ lymphocytes
b. Natural killer (NK) cells
c. Macrophages
d. Dendritic cells
e. Neutrophils
209. In response to infection with Mycobacterium tuberculosis, a granuloma forms in the lung.
Within the granuloma are cells expressing class II MHC antigens. These cells elaborate
cytokines that promote fibroblastic proliferation. From which of the following peripheral
blood cells are these cells derived?
a. Neutrophil
b. Monocyte
c. B Cell
d. NK cell
e. Basophil

210. Following are statements regarding T lymphocytes. Which are the incorrect ones?
I. produce antibodies
II. mature in the thymus
III. are identified by the presence of surface immunoglobulin
IV. produce cytokines
V. have receptors for MHC molecules
a. I and II
b. II and III
c. I and III
d. II and IV

211. A lymph node biopsy from a 43 year old man with lymph node enlargement shows a
malignant tumour of lymphoid cells. Staining shows nuclei positive for BCL2. What is the
likely mechanism for this lymphoma?
a. Increased tyrosine kinase activity
b. Lack of apoptosis
c. Gene amplifications
d. Reduced DNA repair
e. Loss of cell cycle inhibition

212. An ELISA designed to test for the presence of serum antibody for a new strain of pathogenic
bacteria is under development. Initially, a monoclonal antibody specific for a single epitope of
the organism was used both to sensitize the wells of the ELISA plate and as the enzyme-
labeled detecting antibody in a conventional sandwich ELISA. The ELISA failed to detect the
antigen despite the use of a wide range of antibody concentrations. What is the most
probable cause of this problem?
a. The antigen is too large.
b. The antibody has a low affinity for the antigen.
c. The monoclonal antibody used to sensitize the wells is blocking access of the epitope,
thus when the same antibody is enzyme-labeled, it cannot bind to the antigen.
d. The enzyme-labeled antibody used should have been a different isotype than the
sensitizing antibody.
e. The monoclonal antibody used is probably unstable.

213. Infection with vaccinia virus results in the priming of virus-specific CD8+ T cells. If these
vaccinia virus-specific CD8+ T cells are subsequently removed from the individual, which of
the following cells will they kill in vitro?
a. vaccinia-infected cells expressing MHC class II molecules from any individual
b. influenza-infected cells expressing the same MHC class I molecules as the individual
c. uninfected cells expressing the same MHC class I molecules as the individual
d. vaccinia-infected cells expressing the same MHC class I molecules as the individual
e. vaccinia-infected cells expressing the same MHC class II molecules as the individual

214. A human volunteer agrees to be passively sensitized with IgE specific for a ragweed antigen
(allergen). When challenged with the allergen intradermally, he displayed a typical skin
reaction due to an immediate hypersensitivity reaction. If the injection with sensitizing IgE
was preceded by an injection (at the same site) of Fc fragments of human IgE followed by
intradermal injection with allergen, which of the following outcomes would you predict?
a. No reaction would occur because the Fc fragments would interact with the allergen and
prevent it from gaining access to the sensitized mast cells.
b. No reaction would occur because the Fc fragments would interact with the IgE antibodies
making their antigen-binding sites unavailable for binding to antigen.
c. No reaction would occur because the Fc fragments would interact with Fce receptors on
mast cells.
d. The reaction would be exacerbated due to the increased local concentration of IgE Fc
fragments.
e. The reaction would be exacerbated due to the activation of complement.

215. A patient is suspected of having farmer's lung. A provocation test involving the inhalation of
an extract of moldy hay is performed. A sharp drop in respiratory function is noted within 10
minutes and returns to normal in 2 hours, only to fall again in another 2 hours. The most
likely explanation is that
a. the patient has existing T cell-mediated hypersensitivity.
b. this is a normal pattern for farmer's lung.
c. the patient developed a secondary response after the inhalation of antigen.
d. the symptoms of farmer's lung are complicated by an IgE-mediated reactivity to the same
antigen.
e. All of the above.

216. A 50-year-old worker at an atomic plant who previously had a sample of his own bone
marrow cryopreserved was accidentally exposed to a minimal lethal dose of radiation. He
was subsequently transplanted with his own bone marrow. This individual can expect
a. to have recurrent bacterial infections.
b. to have serious fungal infections due to deficiency in cell-mediated immunity.
c. to make antibody responses to thymus-independent antigens only.
d. All of the above.
e. None of the above.

217. Kidney transplantation was performed using a kidney from a donor who was matched to the
recipient by serologic tissue typing. However, within a few months the kidney was rejected.
Assuming no technical problems with the surgical procedure, one reason for the rejection
may be that
a. there was insufficient blood supply to the graft.
b. there could have been a mismatch, which would have been detected by a mixed
lymphocyte reaction.
c. the recipient developed blocking antibodies.
d. the recipient also suffered from Wiskott Aldrich syndrome.
e. the donor was agammaglobulinemic.
218. A line of tumor cells prepared from one mouse is injected into an MHC-matched recipient.
The tumor was destroyed within 5 days. When tissues from the recipient were analyzed, the
animal had a negligible cytotoxic T lymphocyte (CTL) response against the tumor. Flow
cytometry revealed that the tumor cells expressed very low levels of MHC class I molecules.
The BEST explanation for the destruction of the tumor cells in the recipient is that:
a. CTL had an MHC-induced alloreaction against the tumor cells
b. CTL specific for the tumor antigens killed the tumor cells
c. Mast cells released histamine and other inflammatory mediators that clear the tumor
d. Natural killer cells bound anti-tumor antibodies and killed the tumor cells
e. Natural killer cells recognized the absence of MHC class I molecules and killed the tumor
cells

219. It has been shown that a B-cell lymphoma could be eliminated with anti-idiotypic serum.
The use of this approach to treat a plasma cell tumor would not be warranted because
a. plasma cell tumors have no tumor-specific antigens.
b. plasma cell tumors are not expected to be susceptible to ADCC.
c. plasma cell tumors can be killed in vivo only by cytotoxic T lymphocytes that bear the
same A, B, and C transplantation antigens.
d. the plasma cells do not have surface Ig.
e. the idiotype on the plasma-cell surface is different from that on the B-cell surface.

220. Freunds adjuvant is a commonly used adjuvant when immunizing mice to improve antibody
production. The so-called complete form (CFA) is composed of inactivated mycobacterial
components emulsified in mineral oil. What best describes the mechanism behind the
improved Ab production?
a. Effective antigen delivery to B-cells in the germinal center of lymph nodes
b. Improved degradation of the antigen by the cytosolic proteosome
c. Slow release of antigen and improvement of memory T-cell development
d. Slow release of antigen and TLR-initiated upregulation of co-stimulatory molecules of
antigen presenting cells (signal 2)

221. The lymphoid organs are organised tissue where lymphocytes (B-T cells) interact with non
lymphoid cells (APC) and trapped antigen. They are divided into primary central and
secondary peripheral organs and have different roles in immunity. What is the major function
of the peripheral organs?
a. Provide the microenvironment for maturation of T and B cells.
b. Maximize contact between antigen and lymphocytes.
c. Produce antigen-specific lymphocytes from stem cells in response to antigen
d. Provide a site where memory T-cells reside to ensure a rapid response to antigen
e. Sequester antigen to minimize its damage to the body.

222. Allelic exclusion is a process that is unique to B- and T-cells. Why is it important for B-cells
to exert (have) allelic exclusion?
a. Allelic exclusion ensures that a B cell can recognize only one antigen and give increased
antibody diversity
b. Allelic exclusion ensures that a B cell can recognize several antigens and give increased
antibody diversity
c. Allelic exclusion ensures that a B cell can recognize only one antigen and enables control
of antibody specificity
d. Allelic exclusion ensures that a B cell can produce high affinity antibodies towards
several antigens

223. B-cells can produce five different classes of antibodies. Antibodies belonging to the different
classes have different effector mechanisms. Transport of antibodies from mother to the fetus
(transcytosis) is important to protect the child against disease during the first months after
birth. Which one of the following sentences describes this situation most accurately?
a. Transportation of IgM and IgG to the fetus provides the child with antibodies that,
through opsonization and neutralization, may protect against several diseases
b. Transportation of IgG and IgE to the fetus leads to protection of the child through
different effector mechanisms towards the same antigens that the mother has generated
antibodies towards
c. Transportation of IgM to the fetus provides the child with antibodies that, through
opsonization and neutralization, may protect against several diseases
d. Transportation of IgG to the fetus leads to protection of the child through different
antibody effector mechanisms towards the same antigens that the mother has generated
antibodies towards

224. Immunological memory is important for an effective vaccine. Which one of the following
sentences describes the underlying mechanisms for this most accurately?
a. Macrophages and B cells have developed optimal effector mechanisms towards the
antigen contained in the vaccine and can during an infection with this antigen respond
faster and more effectively
b. T -and B-cells have developed effector mechanisms towards the antigen contained in the
vaccine and can during an infection with this antigen respond fast and effectively to
prevent disease
c. T -and B-cells have developed effector mechanisms towards a certain virus or bacteria
and can prevent us from infection
d. NK cells, T -and B-cells have developed effector mechanisms towards the antigen
contained in the vaccine and can during an infection with this antigen respond faster and
more effectively

225. In a stressful condition, ACTH secretion was increased and as a result glucocorticoid
concentration was elevated in blood. One or a combination of the following changes most
likely taking place in this condition
A. Decreased circulating eosinophils and basophils
B. Reduced IL-2 release
C. Potentiated inflammatory response to tissue injury
D. Increased mitotic activity of lymphocytes in lymph nodes
The correct answer is
a. B and C
b. A and B
c. B and D
d. C and D

226. In order to prevent tetanus in neonates, one of the following treatments can be adopted
A. Treatment of the infant with anti-toxin and the toxoid
B. Immunize the mother with the toxoid
In case of A, the treatment can be given
a) Immediately after birth
b) After the onset of the condition
In case of B, the immunization has to be done
c) Before pregnancy
d) Late in pregnancy
The correct combination is
a. A/a
b. A/b
c. B/c
d. B/d

227. A monoclonal antibody binds to G-actin but not to F-actin, what does this tell you about
epitope recognized by the antibody
a. The Epitope in F-actin induceds an enzymatic activity which inactivates mAb
b. The epitope in F-actin is surrounded by myosin thus no mAb interaction is observed
c. The epitope is likely to be a structure that is buried when G-actin polymerizes to form F-
actin
d. The peptide fragment release from F-actin blocks the mAb interaction with the epitope

228. What changes are made in B cells when they switch from producing IgM or IgD to IgG
immunoglobulins?
a. this change is accomplished by alternative splicing of the RNA transcripts
b. this change occurs in the proteome as the IgM/IgD constant regions are proteolytically
removed from the IgG protein
c. this change occurs in the genome as the genes encoding the constant regions for IgM and
IgD are deleted by the RAG1 and RAG2 proteins
d. this change occurs in the genome as the genes encoding the constant regions for IgM and
IgD are deleted independently of the RAG proteins

229. HAT medium is used for selecing the hybridomas based on the following
P. Only hybridoma will grow since it inherited HGPRT genes from B cells and can synthesize
DNA from hypoxanthine
Q. Myeloma cells will not grow in cultures since de novo synthesis is blocked by
aminopterine and due to the lack of HGPRT enzyme
a. Only P is true
b. Only Q is true
c. Both P and Q are true
d. P is true and Q is false
Answer key

1.d
2.d
3.d
4.d
5.d
6.b
7.c
8.c
9.b
10.c
11.c
12.c
13.d
14.c
15.d
16.c
17.b
18.b
19.d
20.b
21.a
22.c
23.b
24.c
25.c
26.a
27.b
28.c
29.d
30.c
31.b
32.c
33.a
34.b
35.d
36.b
37.c
38.c
39.c
40.b
41.d
42.d
43.b
44.c
45.a
46.b
47.d
48.a
49.c
50.b
51.a
52.b
53.d
54.b
55.d
56.e
57.b
58.a
59.c
60.c
61.d
62.d
63.d
64.b
65.b
66.d
67.d
68.e
69.b
70.a
71.b
72.b
73.d
74.c
75.c
76.d
77.b
78.c
79.d
80.d
81.c
82.b
83.c
84.b
85.c
86.a
87.a
88.b
89.d
90.d
91.d
92.a
93.b
94.b
95.a
96.a
97.d
98.c
99.d
100.a
101.a
102.d
103.a
104.c
105.d
106.b
107.a
108.e
109.b
110.d
111.a
112.a
113.d
114.d
115.c
116.a
117.c
118.b
119.b
120.a
121.f
122.d
123.b
124.e
125.d
126.a
127.d
128.a
129.c
130.b
131.d
132.b
133.b
134.c
135.d
136.b
137.c
138.d
139.b
140.a
141.c
142.d
143.c
144.d
145.d
146.d
147.d
148.b
149.d
150.e
151.d
152.d
153.b
154.b
155.c
156.d
157.b
158.b
159.a
160.b
161.c
162.b
163.e
164.c
165.d
166.d
167.b
168.a
169.a
170.d
171.e
172.e
173.d
174.b
175.b
176.c
177.b
178.d
179.c
180.b
181.a
182.c
183.c
184.a
185.a
186.c
187.a
188.a
189.c
190.a
191.d
192.e
193.b
194.c
195.b
196.b
197.d
198.a
199.a
200.d
201.b
202.a
203.b
204.a
205.d
206.d
207.b
208.a
209.b
210.c
211.b
212.c
213.d
214.c
215.d
216.e
217.b
218.e
219.d
220.d
221.b
222.c
223.d
224.b
225.b
226.c
227.c
228.d
229.c

S-ar putea să vă placă și